Вы находитесь на странице: 1из 127

K – Security – CNDI 2016

1NC
1NC – Security K
The affirmative’s security rhetoric creates threats and sanitizes global militaristic
violence – their approach to international relations makes extinction inevitable.
Ahmed 12 Dr. Nafeez Mosaddeq Ahmed is Executive Director of the Institute for Policy Research and Development (IPRD), an
independent think tank focused on the study of violent conflict, he has taught at the Department of International Relations, University of Sussex
"The international relations of crisis and the crisis of international relations: from the securitisation of scarcity to the militarisation of society"
Global Change, Peace & Security Volume 23, Issue 3, 2011 Taylor Francis 3. From securitisation to militarisation 3.1 Complicity

This analysis thus calls for a broader approach to environmental security based on retrieving the manner in which political actors
construct discourses of 'scarcity' in response to ecological, energy and economic crises (critical security
studies) in the context of the historically-specific socio-political and geopolitical relations of domination by
which their power is constituted, and which are often implicated in the acceleration of these very crises
(historical sociology and historical materialism). Instead, both realist and liberal orthodox IR approaches focus on different
aspects of interstate behaviour, conflictual and cooperative respectively, but each lacks the capacity to grasp that the unsustainable trajectory
of state and inter-state behaviour is only explicable in the context of a wider global system concurrently over-exploiting the biophysical
environment in which it is embedded. They are, in other words, unable to address the relationship of the inter-state
system itself to the biophysical environment as a key analytical category for understanding the
acceleration of global crises. They simultaneously therefore cannot recognise the embeddedness of the economy in society and the
concomitant politically-constituted nature of economics. Hence, they neglect the profound irrationality of collective
state behaviour, which systematically erodes this relationship, globalising insecurity on a massive scale
- in the very process of seeking security.85 In Cox's words, because positivist IR theory 'does not question
the present order [it instead] has the effect of legitimising and reifying it '.86 Orthodox IR sanitises
globally-destructive collective inter-state behaviour as a normal function of instrumental reason -thus
rationalising what are clearly deeply irrational collective human actions that threaten to permanently
erode state power and security by destroying the very conditions of human existence . Indeed, the prevalence of orthodox IR
as a body of disciplinary beliefs, norms and prescriptions organically conjoined with actual policy-making in the international system highlights
the extent to which both realism and liberalism are ideologically implicated in the acceleration of global
systemic crises. By the same token, the incapacity to recognise and critically interrogate how prevailing
social, political and economic structures are driving global crisis acceleration has led to the proliferation
of symptom-led solutions focused on the expansion of state/regime military-political power rather than
any attempt to transform root structural causes.88 It is in this context that, as the prospects for meaningful
reform through inter-state cooperation appear increasingly nullified under the pressure of actors with a vested
interest in sustaining prevailing geopolitical and economic structures, states have resorted progressively more to
militarised responses designed to protect the concurrent structure of the international system from dangerous new threats. In effect,
the failure of orthodox approaches to accurately diagnose global crises, directly accentuates a tendency
to 'securitise' them - and this, ironically, fuels the proliferation of violent conflict and militarisation responsible for
magnified global insecurity. 'Securitisation' refers to a 'speech act' - an act of labelling - whereby political authorities
identify particular issues or incidents as an existential threat which, because of their extreme nature, justify going beyond the
normal security measures that are within the rule of law. It thus legitimises resort to special extra-legal powers. By labelling issues a matter of
'security', therefore, statesare able to move them outside the remit of democratic decision-making and into
the realm of emergency powers, all in the name of survival itself. Far from representing a mere aberration from
democratic state practice, this discloses a deeper 'dual' structure of the state in its institutionalisation of the
capacity to mobilise extraordinary extra-legal military-police measures in purported response to an
existential danger. The problem in the context of global ecological, economic and energy crises is that
such levels of emergency mobilisation and militarisation have no positive impact on the very global
crises generating 'new security challenges', and are thus entirely disproportionate.90 All that remains to examine is
on the 'surface' of the international system (geopolitical competition, the balance of power, international regimes, globalisation and so
on), phenomena which are dislocated from their structural causes by way of being unable to recognise
the biophysically-embedded and politically-constituted social relations of which they are comprised. The
consequence is that orthodox IR has no means of responding to global systemic crises other than to
reduce them to their symptoms. Indeed, orthodox IR theory has largely responded to global systemic
crises not with new theory, but with the expanded application of existing theory to 'new security
challenges' such as 'low-intensity' intra-state conflicts; inequality and poverty; environmental
degradation; international criminal activities including drugs and arms trafficking; proliferation of
weapons of mass destruction; and international terrorism.91 Although the majority of such 'new security challenges' are
non-military in origin - whether their referents are states or individuals - the inadequacy of systemic theoretical frameworks
to diagnose them means they are primarily examined through the lenses of military-political power.92
In other words, the escalation of global ecological, energy and economic crises is recognised not as
evidence that the current organisation of the global political economy is fundamentally unsustainable ,
requiring urgent transformation, but as vindicating the necessity for states to radicalise the exertion of their
military-political capacities to maintain existing power structures , to keep the lid on.93 Global crises are thus
viewed as amplifying factors that could mobilise the popular will in ways that challenge existing political and economic structures, which it is
presumed (given that state power itself is constituted by these structures) deserve protection. This justifies the state's adoption of extra-legal
measures outside the normal sphere of democratic politics. In the context of global crisis impacts, this counter-democratic trend-line can
result in a growing propensity to problematise potentially recalcitrant populations - rationalising violence
toward them as a control mechanism. Consequently, for the most part, the policy implications of orthodox IR approaches
involve a redundant conceptualisation of global systemic crises purely as potential 'threat-multipliers' of
traditional security issues such as 'political instability around the world, the collapse of governments and the
creation of terrorist safe havens'. Climate change will serve to amplify the threat of international terrorism, particularly in regions with
large populations and scarce resources. The US Army, for instance, depicts climate change as a 'stress-multiplier'
that will 'exacerbate tensions' and 'complicate American foreign policy'; while the EU perceives it as a 'threat-multiplier which
exacerbates existing trends, tensions and instability'.95 In practice, this generates an excessive preoccupation not with the
causes of global crisis acceleration and how to ameliorate them through structural transformation, but with their purportedly
inevitable impacts, and how to prepare for them by controlling problematic populations. Paradoxically, this
'securitisation' of global crises does not render us safer. Instead, by necessitating more violence, while
inhibiting preventive action, it guarantees greater insecurity. Thus, a recent US Department of Defense report explores
the future of international conflict up to 2050. It warns of 'resource competition induced by growing populations and expanding economies',
particularly due to a projected 'youth bulge' in the South, which 'will consume ever increasing amounts of food, water and energy'. This will
prompt a 'return to traditional security threats posed by emerging near-peers as we compete globally for depleting natural resources and
overseas markets'. Finally, climate change will 'compound' these stressors by generating humanitarian crises, population migrations and other
complex emergencies.96 A similar study by the US Joint Forces Command draws attention to the danger of global energy depletion through to
2030. Warning of ‘the dangerous vulnerabilities the growing energy crisis presents’, the report concludes that ‘The implications for future
conflict are ominous.’97 Once again, the subject turns to demographics: ‘In total, the world will add approximately 60 million people each year
and reach a total of 8 billion by the 2030s’, 95 per cent accruing to developing countries, while populations in developed countries slow or
decline. ‘Regions such as the Middle East and Sub-Saharan Africa, where the youth bulge will reach over 50% of the population, will possess
fewer inhibitions about engaging in conflict.’98 The assumption is that regions which happen to be both energy-rich and Muslim-majority will
also be sites of violent conflict due to their rapidly growing populations. A British Ministry of Defence report concurs with this assessment,
highlighting an inevitable ‘youth bulge’ by 2035, with some 87 per cent of all people under the age of 25 inhabiting developing countries. In
particular, the Middle East population will increase by 132 per cent and sub-Saharan Africa by 81 per cent. Growing resentment due to
‘endemic unemployment’ will be channelled through ‘political militancy, including radical political Islam whose concept of Umma, the global
Islamic community, and resistance to capitalism may lie uneasily in an international system based on nation-states and global market forces’.
More strangely, predicting an intensifying global divide between a super-rich elite, the middle classes and an urban under-class, the report
warns: ‘The world’s middle classes might unite, using access to knowledge, resources and skills to shape transnational processes in their own
class interest.’99 Thus, the
securitisation of global crisis leads not only to the problematisation of particular
religious and ethnic groups in foreign regions of geopolitical interest, but potentially extends this problematisation
to any social group which might challenge prevailing global political economic structures across racial, national and class
lines. The previous examples illustrate how secur-itisation paradoxically generates insecurity by reifying a process
of militarization against social groups that are constructed as external to the prevailing geopolitical and
economic order. In other words, the internal reductionism, fragmentation and compartmentalisation that
plagues orthodox theory and policy reproduces precisely these characteristics by externalising global crises from one another,
externalising states from one another, externalising the inter-state system from its biophysical environment, and externalising new
social groups as dangerous 'outsiders*. Hence, a simple discursive analysis of state militarisation and the construction of new
"outsider* identities is insufficient to understand the causal dynamics driving the process of 'Otherisation' .
As Doug Stokes points out, the Western state preoccupation with the ongoing military struggle against
international terrorism reveals an underlying 'discursive complex", where representations about
terrorism and non-Western populations are premised on 'the construction of stark boundaries* that 'operate to
exclude and include*. Yet these exclusionary discourses are 'intimately bound up with political and
economic processes', such as strategic interests in proliferating military bases in the Middle East,
economic interests in control of oil, and the wider political goal of 'maintaining American hegemony * by
dominating a resource-rich region critical for global capitalism.100 But even this does not go far enough, for
arguably the construction of certain hegemonic discourses is mutually constituted by these geopolitical,
strategic and economic interests — exclusionary discourses are politically constituted . New conceptual
developments in genocide studies throw further light on this in terms of the concrete socio-political dynamics of securitisation processes. It is
now widely recognised, for instance, that the distinguishing criterion of genocide is not the pre-existence of primordial groups, one of which
destroys the other on the basis of a preeminence in bureaucratic military-political power. Rather, genocide is the intentional attempt to destroy
a particular social group that has been socially constructed as different. As Hinton observes, genocides precisely constitute a process of
'othering* in which an imagined community becomes reshaped so that previously 'included* groups become 'ideologically recast' and
dehumanised as threatening and dangerous outsiders, be it along ethnic, religious, political or economic lines — eventually legitimising
their annihilation.102 In other words, genocidal violence is inherently rooted in a prior and ongoing
ideological process, whereby exclusionary group categories are innovated , constructed and 'Otherised' in
accordance with a specific socio-political programme . The very process of identifying and classifying particular groups as
outside the boundaries of an imagined community of 'inclusion*, justifying exculpatory violence toward them, is itself a political act without
which genocide would be impossible.1 3 This recalls Lemkin's recognition that the intention
to destroy a group is integrally
connected with a wider socio-political project - or colonial project — designed to perpetuate the political,
economic, cultural and ideological relations of the perpetrators in the place of that of the victims, by
interrupting or eradicating their means of social reproduction. Only by interrogating the dynamic and
origins of this programme to uncover the social relations from which that programme derives can the
emergence of genocidal intent become explicable. Building on this insight, Semelin demonstrates that the process of
exclusionary social group construction invariably derives from political processes emerging from deep-seated sociopolitical crises that
undermine the prevailing framework of civil order and social norms; and which can, for one social group, be seemingly resolved by projecting
anxieties onto a new 'outsider' group deemed to be somehow responsible for crisis conditions. It is
in this context that various
forms of mass violence, which may or may not eventually culminate in actual genocide, can become
legitimised as contributing to the resolution of crises .105

The alternative is to reject the affirmative’s security discourse – only critically


interrogating discursive representations can produce successful action
Cheeseman & Bruce 96 (Graeme, Senior Lecturer at the University of New South Wales, and
Robert, Associate Professor in social sciences at Curtin university, “Discourses of Danger & Dread
Frontiers”, p. 5-8, MT)
This goal is pursued in ways which are still unconventional in the intellectual milieu of international relations in Australia, even though they are
gaining influence worldwide as traditional modes of theory and practice are rendered inadequate by global trends that defy comprehension, let
alone policy. The inability to give meaning to global changes reflects partly the enclosed, elitist world of
professional security analysts and bureaucratic experts, where entry is gained by learning and accepting
to speak a particular, exclusionary language. The contributors to this book are familiar with the discourse, but accord no
privileged place to its ‘knowledge form as reality’ in debates on defence and security. Indeed, they believe that debate will be
furthered only through a long overdue critical re-evaluation of elite perspectives. Pluralistic, democratically-oriented
perspectives on Australia’s identity are both required and essential if Australia’s thinking on defence and security is to be invigorated. This is
not a conventional policy book; nor should it be, in the sense of offering policy-makers and their academic
counterparts sets of neat alternative solutions, in familiar language and format, to problems they pose. This
expectation is in itself a considerable part of the problem to be analysed. It is, however, a book about policy, one
that questions how problems are framed by policy-makers. It challenges the proposition that irreducible bodies of real
knowledge on defence and security exist independently of their ‘context in the world’, and it demonstrates how security policy is
articulated authoritatively by the elite keepers of that knowledge, experts trained to recognize enduring,
universal wisdom. All others, from this perspective, must accept such wisdom or remain outside the
expert domain, tainted by their inability to comply with the ‘rightness’ of the official line. But it is
precisely the official line, or at least its image of the world, that needs to be problematised. If the critic responds
directly to the demand for policy alternatives, without addressing this image, he or she is tacitly
endorsing it. Before engaging in the policy debate the critics need to reframe the basic terms of
reference. This book, then, reflects and underlines the importance of Antonio Gramsci and Edward Said’s ‘critical intellectuals’.15 The
demand, tacit or otherwise, that the policy-maker’s frame of reference be accepted as the only basis for
discussion and analysis ignores a three thousand year old tradition commonly associated with Socrates and purportedly
integral to the Western tradition of democratic dialogue. More immediately, it ignores post-seventeenth century democratic
traditions which insist that a good society must have within it some way of critically assessing its knowledge and the
decisions based upon that knowledge which impact upon citizens of such a society. This is a tradition with a slightly different connotation in
contemporary liberal democracies which, during the Cold War, were proclaimed different and superior to the totalitarian enemy precisely
because there were institutional checks and balances upon power. In short, one of the major differences between ‘open societies’ and their
(closed) counterparts behind the Iron Curtain was that the former encouraged the critical testing of the knowledge and decisions of the
powerful and assessing them against liberal democratic principles. The latter tolerated criticism only on rare and limited occasions. For some,
this represented the triumph of rational-scientific methods of inquiry and techniques of falsification. For others, especially since positivism and
rationalism have lost much of their allure, it meant that for society to become open and liberal, sectors of the population must be independent
of the state and free to question its knowledge and power. Though we do not expect this position to be accepted by every reader, contributors
to this book believe that critical dialogue is long overdue in Australia and needs to be listened to. For all its liberal democratic trappings,
Australia’s security community continues to invoke closed monological narratives on defence and security. This book also questions the
distinctions between policy practice and academic theory that inform conventional accounts of Australian security. One of its major concerns,
particularly in chapters 1 and 2, is to illustrate how theory
is integral to the practice of security analysis and policy
prescription. The book also calls on policy-makers, academics and students of defence and security to think critically
about what they are reading, writing and saying; to begin to ask , of their work and study, difficult and searching
questions raised in other disciplines; to recognise, no matter how uncomfortable it feels, that what is involved in
theory and practice is not the ability to identify a replacement for failed models, but a realisation that
terms and concepts – state sovereignty, balance of power, security, and so on – are contested and
problematic, and that the world is indeterminate, always becoming what is written about it. Critical
analysis which shows how particular kinds of theoretical presumptions can effectively exclude vital areas of
political life from analysis has direct practical implications for policy-makers, academics and citizens who face the
daunting task of steering Australia through some potentially choppy international waters over the next few years. There is also much of interest
in the chapters for those struggling to give meaning to a world where so much that has long been taken for granted now demands imaginative,
incisive reappraisal. The contributors, too, have struggled to find meaning, often despairing at the terrible human costs of international
violence. This is why readers will find no single, fully formed panacea for the world’s ills in general, or Australia’s security in particular. There are
none. Every chapter, however, in its own way, offers something more than is found in orthodox literature, often by exposing ritualistic Cold War
defence and security mind-sets that are dressed up as new thinking. Chapters 7 and 9, for example, present alternative ways of engaging in
security and defence practice. Others (chapters 3, 4, 5, 6 and 8) seek to alert policy-makers, academics and students to alternative
theoretical possibilities which might better serve an Australian community pursuing security and prosperity in an
uncertain world. All chapters confront the policy community and its counterparts in the academy with a deep awareness of the intellectual and
material constraints imposed by dominant traditions of realism, but they avoid dismissive and exclusionary terms which often in the past
characterized exchanges between policy-makers and their critics. This is because, as noted earlier, attention needs to be paid to the
words and the thought processes of those being criticized. A close reading of this kind draws attention to underlying assumptions,
showing they need to be recognized and questioned. A sense of doubt (in place of confident certainty) is a necessary prelude to a
genuine search for alternative policies. First comes an awareness of the need for new perspectives, then
specific policies may follow. As Jim George argues in the following chapter, we need to look not so much at
contending policies as they are made for us but at challenging ‘the discursive process which gives [favoured
interpretations of “reality”] their meaning and which direct [Australia’s] policy/analytical/military responses’. This
process is not restricted to the small, official defence and security establishment huddled around the US-Australian War Memorial in Canberra.
It also encompasses much of Australia’s academic defence and security community located primarily though not exclusively within the
Australian National University and the University College of the University of New South Wales. These discursive processes are examined in
detail in subsequent chapters as authors attempt to make sense of a politics of exclusion and closure which exercises disciplinary power over
Australia’s security community. They also question the discourse of ‘regional security’, ‘security cooperation’, ‘peacekeeping’ and ‘alliance
politics’ that are central to Australia’s official and academic security agenda in the 1990s. This is seen as an important task especially when, as is
revealed, the disciplines of International Relations and Strategic Studies are under challenge from critical and theoretical debates ranging across
the social sciences and humanities; debates that are nowhere to be found in Australian defence and security studies. The chapters graphically
illustrate how Australia’s public policies on defence and security are informed, underpinned and legitimised by a narrowly-based intellectual
enterprise which draws strength from contested concepts of realism and liberalism, which in turn seek legitimacy through policy-making
processes. Contributors ask whether Australia’s policy-makers and their academic advisors are unaware of broader intellectual debates, or
resistant to them, or choose not to understand them, and why?
Links
Link – Apocalyptic Rhetoric
Apocalyptic predictions make serial policy failure inevitable
Kurasawa 4 (Fuyuki, Professor of Sociology, York University of Toronto, “Cautionary Tales: The Global
Culture of Prevention and the Work of Foresight”, Constellations Volume 11, No 4,
http://www.yorku.ca/kurasawa/Kurasawa%20Articles/Constellations%20Article.pdf)

Up to this point, I have tried to demonstrate that transnational socio-political relations are nurturing a thriving culture
and infrastructure of prevention from below, which challenges presumptions about the inscrutability
of the future (II) and a stance of indifference toward it (III). Nonetheless, unless and until it is substantively ‘filled in,’ the
argument is vulnerable to misappropriation since farsightedness does not in and of itself ensure
emancipatory outcomes. Therefore, this section proposes to specify normative criteria and participatory procedures through which citizens can
determine the ‘reasonableness,’ legitimacy, and effectiveness of competing dystopian visions in order to arrive at a socially self-instituting future.
Foremost among the possible distortions of farsightedness is alarmism, the manufacturing of
unwarranted and unfounded doomsday scenarios. State and market institutions may seek to
produce a culture of fear by deliberately stretching interpretations of reality beyond the limits of the
plausible so as to exaggerate the prospects of impending catastrophes, or yet again, by intentionally
promoting certain prognoses over others for instrumental purposes. Accordingly, regressive
dystopias can operate as Trojan horses advancing political agendas or commercial interests that
would otherwise be susceptible to public scrutiny and opposition. Instances of this kind of
manipulation of the dystopian imaginary are plentiful: the invasion of Iraq in the name of fighting
terrorism and an imminent threat of use of ‘weapons of mass destruction’ ; the severe curtailing of American civil
liberties amidst fears of a collapse of ‘homeland security’; the neoliberal dismantling of the welfare state as the only

remedy for an ideologically constructed fiscal crisis ; the conservative expansion of policing and incarceration due to supposedly
spiraling crime waves; and so forth. Alarmism constructs and codes the future in particular ways, producing or

reinforcing certain crisis narratives, belief structures, and rhetorical conventions . As much as alarmist ideas beget
a culture of fear, the reverse is no less true. If fear-mongering is a misappropriation of preventive foresight, resignation about the future represents a
problematic outgrowth of the popular acknowledgment of global perils. Some believe that the world to come is so uncertain and dangerous that we should not
attempt to modify the course of history; the future will look after itself for better or worse, regardless of what we do or wish. One version of this argument
consists in a complacent optimism perceiving the future as fated to be better than either the past or the present. Frequently accompanying it is a self-deluding
denial of what is plausible (‘the world will not be so bad after all’), or a naively Panglossian pragmatism (‘things will work themselves out in spite of everything,
because humankind always finds ways to survive’).37 Much
more common, however, is the opposite reaction, a fatalistic
pessimism reconciled to the idea that the future will be necessarily worse than what preceded it. This
is sustained by a tragic chronological framework according to which humanity is doomed to decay, or
a cyclical one of the endless repetition of the mistakes of the past. On top of their dubious
assessments of what is to come, alarmism and resignation would, if widely accepted, undermine a
viable practice of farsightedness. Indeed, both of them encourage public disengagement from
deliberation about scenarios for the future, a process that appears to be dangerous, pointless, or
unnecessary. The resulting ‘depublicization’ of debate leaves dominant groups and institutions(the
state, the market, techno-science) in charge of sorting out the future for the rest of us, thus
effectively producing a heteronomous social order. How, then, can we support a democratic process of prevention from below?
The answer, I think, lies in cultivating the public capacity for critical judgment and deliberation, so that participants in global civil society subject all claims about
potential catastrophes to examination, evaluation, and contestation. Two normative concepts are particularly well suited to grounding these tasks: the
precautionary principle and global justice.
Apocalyptic futures that lock in technocratic solutions breed apathy and depoliticize
agency
de Goede & Randalls 9 (Marieke, Department of European Studies, Univ of Amsterdam, and
Samuel, Dept of Geography, University College London, “Precaution, preemption: arts and technologies
of the actionable Future, Environment and Planning D: Society and Space 2009, volume 27, pages 859-
878)
If humans are the ones to blame, then humans can also attain control through reducing emissions or speculative geoengineering strategies.
Geoengineering approaches are claimed to be necessary as a precaution (eg Connor and Green, 2009), making available a set of emergency
technologies in case the unthinkable starts to occur (Fleming, 2007). Questions
of politics and ethics are frequently swept
aside by a banal, technocratic focus on risks. These `ultimate solutions' , whilst not new ideas especially in military
circles (Fleming, 2007), are enlivened by the dramatizations of apocalyptic futures in which the only way to act
seems to be to adopt spectacular techniques of/for control. The possibility of annihilation is made banal
through its institutional preparation and routinized imagination (Masco, 2006). This technological response
is also deeply redemptive in its potential to recover perfection (Noble, 1997). One of the political questions to be raised in
relation to this new banality of catastro- phe, as we will discuss below, is whether it is able to foster enchantment, understood as the
``profound and empowering attachment to life'' that, according to Bennett (2001, page 160), is required for ethical political engagement. The
apocalyptic futures are also stage-managed in practised responses to the threat of terrorism or climate change: terrorism
through, for example, London scenarios of a terrorist strike and climate change through the search for criteria to measure climate change
preparedness across the world. This is bolstered by, for example, the 1-in-1000- year tidal flood event exercise (Exercise Triton) held in the UK
in 2004 or the ATLANTIS project led by Thomas Downing at the Stockholm Environment Institute branch in Oxford. These calculations
and imaginations produce citizens sensitive to the concerns of the policy makers, but, at the same time, desensitized to
thinking critically about these issues. Local councils have seized on the opportunities to use dystopic images to engage citizens in
broader agendas of recycling and reductions in energy consumption.(7) Whilst there is not space in this paper to discuss in much detail the
ways in which subjects are constituted through contemporary terrorist and climate change policies, this is nonetheless a critical point. Isin
(2004), for example, discusses the `neurotic citizen' as one who is constituted as neurotic, insecure, and anxious by the rational and affective
governmental politics within a variety of domains including security and the environment. We become terrorist-conscious or climate-change-
conscious consumers, encouraged to eschew war diamonds, oil, or air-freighted goods (Campbell, 2005; Le Billon, 2006). The
neurotic
citizen is promised the impossible, absolute security from terrorism and climate change, but despite these active
attempts is unable to address his or her insecurities (indeed, Sutton Council also distributed leaflets to house- holds showing
local landmarks of ponds and trees with the admonishment to ``Protect Sutton, help Stop Climate Change'', with a list of `things you can do' on
the reverse). The implication of this is that the
neurotic citizen is no longer a rational subject amenable to rational
pleas, but, rather, always already an affective subject, where affec- tive politics are implicated through
and in the creation of these subjects (Isin, 2004). For our purposes, the troubling conclusion is the possibility of an
affective subject that is peculiarly susceptible to pleas to manage future climate and terror insecurities with
little consideration of the nature of political engagement and commitment that policies to manage these
insecurities might engender. Finally, it is easy to assume that disasters must always have negative repercussions, but disaster (and
disaster imaginaries) can be immensely profitable and not just for risk management consultancies.(8) Klein (2007), for example, illustrates
the ways in which natural and economic disasters offer opportunities for enforcement of and profits for `disaster
capitalism'. Climate change `events' (whether biophysical or, for example, business symposiums) offer marketable opportunities for
financial products such as catastrophe bonds or hurricane derivatives (Randalls, 2009) and advertising opportunities for companies as diverse as
oil giant BP and clothing company Diesel. Here, the imagination for disaster is used to sustain interest in and profit
from perceived future climatic risks, ones that must be not only anticipated but also exploited. Drawing authority from perceived
climate science (whether warranted or not), these crises become opportunities that must be continually
relegitimated by further warnings of crises to keep the market value of climate sufficiently high. In other words, these
imaginaries do not just generate environmental or government responses; they represent commercial
opportunities too. Whilst uncertainty may have been disabling in the sense of generating coordinated international action on reducing
emissions, climate uncertainties, as with terrorism, have successfully generated business innovation, governmental regulation, and manifold
surveillance practices. Politics We have argued that preemption in contemporary security practice, and precaution in contemporary
environmental practice display important affinities and historical entan- glements, through the ways in which they imagine apocalypse and
deploy arts and technologies that render this imagination banal.We now turn to examine more explic- itly the political implications of the
importance of precautionary principles and the resulting quests for knowledge. We argue that three broad political outcomes can be
considered. First, terrorist and climate change policies may be performative, bringing into being the very realities they seek to avoid. Second,
the imagination of apocalypse may depoliticize debates, smuggling other policies in under their rubric;
and, third, they may delegitimate positions in the debates. If apocalypse is also about the imagination of a
paradise (Enzensberger, 1978; Kumar, 1995), an emergent new order, then the irony of contemporary debates is
that they fail to engage in significant political imagination . Thus, we suggest that the banality of apocalypse in
these debates fosters a disenchantment that is itself depoliticizing . Masco writes: ``What does it mean when
the `state of emergency' has so explicitly become the rule when in order to prevent an apocalypse the
governmental apparatus has prepared so meticulously to achieve it?'' (2006, page 12, emphasis in original). First,
then, it is important to emphasize that governments not only are anticipating the worst, but also, in trying
to prevent that nightmare, act in ways that increase the possibility of its occurrence. This
phantasmagoria is thus imagined and made real . Thus, with regard to the politics of security preemption ,
Massumi (2007, ½16) recounts its logic as follows: ``It is not safe for the enemy to make the first move. You have to
move first, to make them move ... .You test and prod, you move as randomly and unpredictably and
ubiquitously as they do... .You move like the enemy, in order to make the enemy move .'' That such reasoning is
not purely theory was demonstrated by the events surrounding the arrest of six New Jersey men accused of plotting to kill soldiers at Fort Dix in
2007. Reports of the arrest uncovered that the `disrupted plot' was actively encouraged by a police informer, posing as an Egyptian radical. It
was the informer who offered to broker a planned weapons purchase, and who, according to New York Times journalist Kocieniewski (2007),
``seemed to be pushing the idea of buying the deadliest items, startling at least one of the suspects.'' In another example of the performativity
of security preemption, it is now widely acknowledged that the preemptive strike on Iraq fostered alliances
between al Qaeda and Iraqi violent groups that did not exist before the war . Indeed, terrorism expert Richardson
(2006, page 166) calls the discursive conflation of the threats of Saddam Hussein and bin Laden a ``self-fulfilling prophecy''.

Representations of catastrophe sanitize everyday violence and make it impossible to


resolve war
Borg 3 Mark B. Borg Jr., practicing psychoanalyst and community/organizational consultant working in
New York City, “Psychoanalytic Pure War: interactions with the post-apocalyptic unconscious”, Journal
of Psychoanalysis, questia

Paul Virilio
and Sylvere Lotringer's concept of "pure war" refers to the potential of a culture to destroy itself
completely (12). 2 We as psychoanalysts can—and increasingly must—explore the impact of this concept
on our practice, and on the growing number of patients who live with the inability to repress or dissociate their experience and awareness
of the pure war condition. The realization of a patient's worst fears in actual catastrophic events has always been a profound enough
psychotherapeutic challenge. These
days, however, catastrophic events not only threaten friends, family, and
neighbors; they also become the stuff of endless repetitions and dramatizations on radio, television, and
Internet. 3 Such continual reminders of death and destruction affect us all . What is the role of the analyst treating
patients who live with an ever-threatening sense of the pure war lying just below the surface of our cultural veneer? At the end of the First
World War, the first "total war," Walter Benjamin observed that "nothing [after the war] remained unchanged but the clouds, and beneath
these clouds, in a field of force of destructive torrents and explosions, was the tiny, fragile human body"(84). Julia
Kristeva makes a
similar note about our contemporary situation, "The recourse to atomic weapons seems to prove that
horror...can rage absolutely" (232). And, as if he too were acknowledging this same fragility and uncontainability, the French
politician Georges Clemenceau commented in the context of World War I that "war is too serious to be confined to the military" (qtd. in Virilio
and Lotringer 15). Virilio
and Lotringer gave the name "pure war" to the psychological condition that results
when people know that they live in a world where the possibility for absolute destruction (e.g., nuclear
holocaust) exists. As Virilio and Lotringer see it, it is not the technological capacity for destruction (that is,
for example, the existence of nuclear armaments ) that imposes the dread characteristic of a pure war
psychology but the belief systems that this capacity sets up . Psychological survival requires that a way be
found (at least unconsciously) to escape inevitable destruction—it requires a way out—but this enforces an
irresolvable paradox, because the definition of pure war culture is that there is no escape . Once people
believe in the external possibility —at least those people whose defenses cannot handle the weight of the dread that pure war
imposes—pure war becomes an internal condition , a perpetual state of preparation for absolute
destruction and for personal, social, and cultural death . The tragedy at the World Trade Center in New York City has given
us a bitter but important opportunity to study the effects of the pure war condition on individuals. It allows us to look at how this all-
encompassing state appears in psychoanalytic treatment and to observe its influence through the analysis of transference/countertransference
dynamics. The pure war condition has been brought grimly to consciousness. In this paper, I will explore how it manifests itself in society, in
character, and most specifically in the psychoanalytic treatment of one patient whose dynamics highlight significant aspects of the pure war
state. How does treatment happen when, at some level, we perceive ourselves as already dead? Whatever our individual differences, our
visions of the psychoanalytic endeavor arise out of the social defense of the culture within which we live and work (I have referred to this as
"community character," cf. Borg 350). And whatever our individual differences, in a pure war situation the primary
task is simply to sustain the dream of psychic survival. The case of Joyce, who saw the first explosion at the World Trade
Center as she rode down Fifth Avenue in a bus after her session with me, exemplifies this task. [End Page 57] The Pure Warrior The
philosophy (or practice) of "pure warriors," that is, of people who are preoccupied with the pure war
condition of their society, is based on the perpetual failure within them of the dissociation and repression that allow
others to function in a situation that is otherwise completely overwhelming. Joyce was one of those who lived on
the border of life and death; she could not escape awareness of that dread dichotomy that most of us are at great pains to dissociate . She
manifested the state of perpetual preparation that is the hallmark of pure war culture and of the
insufficiently defended pure warrior, and also a constant awareness of the nearness of death in all its
various forms. She understood quite well, for instance, that when people are institutionalize d (as she had been on numerous
occasions), "society is defining them as socially dead , [and that at that point] the essential task to be carried
out is to help inmates to make their transition from social death to physical death" (Miller and Gwynne 74).
Against this backdrop, Joyce sought psychoanalysis as a "new world," the place where she would break
free from the deathly institutionalized aspects of her self, and begin her life anew. Her search for a "new
world" included the possibility of a world that was not a pure war world —a prelapsarian Eden. Virilio and
Lotringer state that "war exists in its preparation" (53). And Sun Tsu, who wrote over 2400 years ago and yet is often
considered the originator of modern warfare, said in The Art of War, "Preparation everywhere means lack everywhere" (44). This means
that when the members of a culture must be on guard on all fronts, the resources of that culture are
necessarily scattered and taxed. The more defenses are induced and enacted, the more psychologically
impoverished a culture (or a person) will be. In war-torn nations, resources like food, clothing, and materials for shelter may be
scarce in the general population because they are shunted off to the military. Similarly, the hoarding of psychological
resources and the constant alert status of the defense system are outcomes of existence in a pure war
culture. We can see this scattering and scarcity of resources occurring already in the United States as billions of dollars are shunted from
social services to war efforts and homeland security. In pure war cultures—that is, in cultures that enact a perpetual
preparation for war—the notion of peace is itself a defensive fantasy , although to survive psychically we
distract ourselves from such frightening stimuli as widespread terrorist activities and other events that demonstrate our pure
war status. Pure war obliterates the distinction between soldier and citizen. We have all been drafted.
According to Virilio and Lotringer, "All of us are already civilian soldiers, without knowing it... War happens
everywhere, but we no longer have the means of recognizing it" (42).
Link – Asia
Western conceptions of Asian international relations are filtered from Eurocentric
conceptions of the nation state – reproduces the impact
Kang 3 David-Assc Prof of Government @ Dartmouth College, Adjunct Professor @ the Tuck School of
Business; Getting Asia Wrong: The Need for New Analytical Frameworks; INTERNATIONAL SECURITY,
Vol. 27, No.4, Spring; pp. 57-85.

Most international relations theory is inductively derived from the European experience of the past
four centuries, during which Europe was the locus and generator of war, innovation, and wealth .
According to Kenneth Waltz, “The theory of international politics is written in terms of the great powers of an era. It would be … ridiculous
to construct a theory of international politics based on Malaysia and Costa Rica… A general theory of international politics is necessarily
based on the great powers.” If international relations theorists paid attention to other regions of the globe, it
was to study subjects considered peripheral such as third world security or the behavior of small
states. Accordingly, international relations scholarship has focused on explaining the European
experience, including, for example, the causes of World Wars I and II, as well as the Cold War and U.S.-Soviet relations. Although
this is still true, other parts of the world have become increasingly significant. Accordingly,
knowledge of European relations is no longer sufficient for a well-trained international relations
generalist. During this time Asia itself-sometimes defined as including China, India, Japan, and Russia and comprising perhaps
half the world’s population-had an occasional impact on the great powers, but it was never a primary focus. In the past two
decades, however, Asia has emerged as a region whose economic, military, and diplomatic power has
begun to rival and perhaps even exceed that of Europe. Its growing influence gives scholars a
wonderful opportunity in the fields of international relations generally and Asian security specifically
to produce increasingly rigorous and theoretically sophisticated work. Because Europe was so important for so
long a period, in seeking to understand international relations, scholars have often simply deployed concepts,
theories, and experiences derived from the European experience to project onto and explain Asia.
This approach is problematic at best. Eurocentric ideas have yielded several mistaken conclusions and
predictions about conflict and alignment behavior in Asia. For example, since the early 1990s many
Western analysts have predicted dire scenarios for Asia, whereas many Asian experts have
expressed growing optimism about the region’s future. It is an open question whether Asia, with its very different
political economy, history, culture, and demographics, will ever function like the European state system. This not to criticize
European-derived theories purely because they are based on the Western experience : The origins of a
theory are not necessarily relevant to its applicability. Rather these theories do a poor job as they are applied to
Asia; what I seek to show in this article is that more careful attention to their application can strengthen the theories themselves. In this
article I make two claims about the levels of conflict and types of alignment behavior in Asia. First, I argue that the pessimistic
predictions of Western scholars after the end of the Cold War that Asia would experience a period of
increased arms racing and power politics has largely failed to materialize, a reality that scholars must
confront if they are to develop a better understanding of Asian relations. Second, contrary to the
expectations of standard formulations of realism, and although U.S. power confounds the issue,
Asian states do not appear to be balancing against rising powers such as China. Rather they seem to
be bandwagoning.
Empirics go negative – realist predictions about Asia are simply wrong
Kang 3 David-Assc Prof of Government @ Dartmouth College, Adjunct Professor @ the Tuck School of
Business; Getting Asia Wrong: The Need for New Analytical Frameworks; INTERNATIONAL SECURITY,
Vol. 27, No.4, Spring; pp. 57-85.

Two major problems exist with many of the pessimistic predictions about Asia. First, when
confronted with the
nonbalancing of Asian states against China, the lack of Japanese rearmament, and five decades of
noninvasion by North Korea, scholars typically respond: Just wait. This reply, however, is
intellectually ambiguous. Although it would be unfair to expect instantaneous national responses to changing international
conditions, a dozen years would seem to be long enough to detect at least some change . Indeed Asian
nations have historically shown an ability to respond quickly to changing circumstances. The Meiji restoration in Japan in 1868 was a
remarkable example of governmental response to European and American encroachment, and by 1874 Japan had emerged from centuries
of isolation to occupy Taiwan. More recently, with the introduction of market reforms in late 1978, when Deng Xiaoping famously
declared, “To get rich is glorious,” the Chinese have transformed themselves from diehard socialists to exuberant capitalists beginning less
than three years after Mao’s death in 1976. In
the absence of a specific time frame, the “just wait” response is
unfalsifiable. Providing a causal logic that explains how and when scholars can expect changes is an
important aspect of this response, and reasonable scholars will accept that change may not be immediate but may occur over
time. Without such a time frame, however, the “just wait” response is mere rhetorical wordplay
designed to avoid troubling evidence. A more rigorous response in the Chinese case would be to argue that conditions of
balancing, not timing per se, are the critical factor. In this view, China’s relatively slow military modernization and limited power projection
capabilities suggest that its potential threat to other Asian countries is growing only slowly; thus the conditions necessary to produce costly
all-out balancing efforts do not yet exist. Moreover, even
though many of the conditions that theorists argue can
lead to conflict do already exist in East Asia, the region has so far avoided both major and minor
interstate conflict. Most significant, in less than two decades China has evolved from being a moribund and closed middle power to
the most dynamic country in the region, with an economy that not only will soon surpass Japan’s (it it has not already) but also shows many
signs of continuing growth. This dramatic power transition has evoked hardly any response from China’s neighbors. By
realist
standards, China should be provoking balancing behavior, merely because its overall size and
projected rate of growth are so high. Second, pessimistic predictions about Asia’s future often suffer
from incompletely specified evidentiary standards. Scholars will frequently select evidence that
supports their arguments and dismiss contradictory evidence as epiphenomenal. For example, in his most recent
book, John Mearsheimer argues that although Japan (and Germany) have “the potential in terms of population and wealth to become
great powers…they depend on the United States for their security, and are effectively semi-sovereign states, not great powers.” This begs a
number of questions: For instance, why define Japan, which has the second largest economy in the world, as “semi sovereign”?
Mearsheimer’s book is focused on building a theory of offensive realism, but the
logic of offensive realism would lead to
the conclusion that Japan should have rearmed long ago. The onus is on those predicting an increase
in power politics in Asia to state clearly what evidence would falsify their arguments or challenge
their assumptions, not to explain away objections or ignore contradictory evidence. A clearer
explication of their hypotheses and the refutable propositions would be a genuine contribution to
the field. More than a dozen years after the end of the Cold War, much of Asia bears little
resemblance to the picture painted by the pessimists . Although the years 1950-80 saw numerous armed conflicts, since
then there has been no major interstate war in either Northeast or Southeast Asia. Countries do not
fear for their survival in either area. In Northeast Asia, rivalry and power politics remain muted . Japan
has not rearmed, China shows little sign of having revisionist tendencies, and North Korea has neither imploded nor exploded. Southeast
Asia, as well, remains free f the kinds of arms races and power politics that some have expected. As Muthiah Alagappa writes, “ Viewed
through the ahistorical realist lens, the contemporary security challenges could indeed suggest that
Asia is a dangerous place. But a comprehensive historical view would suggest otherwise . Although Asia
still faces serious internal and international challenges, there are fewer challenges than before and most of the region’s disputes and
conflicts have stabilized.” The field of international relations would be better served if the pessimists not only admitted this reality but also
asked why this might be the case. Because China has such an important influence on Northeast, Southeast, and even South Asia, I offer the
tentative outline of such an explanation in the following section.

Traditional IR tunnel vision can’t predict Asia – misses alternative explanations for
country’s behavior while reproducing the same established identity conceptions which
foment conflict in the first place
Wirth 15 – analyst at Griffin University Asia Institute
(Christian, “‘Power’ and ‘stability’ in the China–Japan–South Korea regional security complex,” The
Pacific Review, Volume 28, Issue 4, pgs. 553-575)

Despite the strong political emphasis on the promotion of (peaceful) economic development and the ensuing
deepening of socio-economic interdependencies, relations between Northeast Asian governments have over the last years
stagnated if not deteriorated. However, in spite of Asia ‘becoming increasingly militarized’, trilateral
cooperation between China, Japan and South Korea has been evolving (IISS 2012, p. 205).¶ How could
trilateralism emerge in such a tense environment? How significant are trilateral initiatives? Conversely, are we going to
see a deepening division of Northeast Asia between rising and declining, democratic and authoritarian
states? In light of these uncertainties commonly attributed to a power shift, what do calls for stability mean and what do
they tell us about our conceptualization of change in Northeast Asia more generally?¶ Based on the comparative
analysis of the most contentious issues in the China–Japan, Japan–South Korea and South Korea–China dyads, this study finds that
while nationalisms continue to foster antagonistic bilateral relations, the trilateral constellation of
amity–enmity patterns locks Beijing, Tokyo and Seoul into a security complex (Buzan and Wæver 2003) that
provides political space for the emergence of regional cooperation evident in emerging trilateral frameworks.
Neither did materially more powerful actors necessarily coerce and prevail over weaker ones, nor was
there clear and consistent balancing or bandwagoning behaviour . This insight not only questions the
validity of power transition, hegemonic stability and democratic peace theories but also raises the
issue of how to analyse identity conflicts manifest in clashing nationalisms beyond the bilateral
dimension. Moreover, the study offers a deeper explanation of how power structures both hinder and
enable cooperation between regional state actors . It demonstrates that material conceptions of power create
tunnel visions centred on great power politics and bilateralism , thereby precluding alternative
explanations of regional politics while reproducing established identity conceptions . Therefore, the
application of a differentiated concept of power provides a better means to understand political change
in Northeast Asia.¶ Since the mid-1990s, relations among Northeast Asian governments tend to become increasingly conflictual. A
recent survey revealed that in the period from 2007 to 2011, the five largest importers of conventional weapons were all in Asia, up to 24%
compared to the 2002–2006 period. South Korea ranked the second and China the fourth (Holtom et al. 2012). Defence budgets of China, South
Korea and the United States increased while Japan, still the world's sixth largest spender, has been transforming its armed forces from
territorial defence towards expeditionary missions (IISS 1995). Since the 2005 stand-off in the East China Sea (ECS) in particular, Tokyo has
reoriented its defence posture from defence against a northern threat towards the Southwest and further bolstered what are Asia's most
potent naval forces (MOD 2013). This shift included repeated exercises, often with US forces in defending and retaking outer islands ostensibly
aimed at increasing the Chinese presence in the Western Pacific (Asahi Shimbun 2013a). Not the least with an eye on Japan, South Korea has
been significantly increasing its defence expenditures and has not missed the turn to the seas (Chosun Ilbo 2012a). Moreover, in light of
the continued emphasis on bilateral military alliances with Japan and South Korea as well as cooperation with
Southeast Asian partners during the Bush administration and the recent ‘pivot’ or ‘rebalancing’ towards the Asia–Pacific
by the Obama administration, proponents are hard-pressed to explain why reinforced US presence has
not been accompanied by greater stability.¶ Contrary to conventional wisdom, economics and politics are difficult
to separate and rational behaviour in the conduct of foreign and security politics is not a given. In the wake
of disputes over the delimitation of exclusive economic zones (EEZ) and territorial sovereignty, communication
between governments broke down, imported goods were shunned and partially violent popular protests
erupted (Asahi Shimbun 2012a). Commentators started to argue that retaliatory measures against Japanese businesses are useful political
tools because the prevailing economic asymmetry would make Japan's economy suffer more than the Chinese (Xinhua 2012). The Japanese
ambassador to China at the time explicitly condemned the ‘extreme view’ that the Chinese economy can get along without Japan stating that
‘such opinions represent a very arrogant attitude toward the economy’ (Okudera 2012).¶ Against
this background, the
phenomenon of intergovernmental cooperation developing alongside increasingly antagonistic relations among China,
Japan and South Korea is surprising and the scant attention it has received is puzzling. Since December 2008,
China–Japan–South Korea (CJK) summits have been convened annually. The process most recently
culminated in the establishment of a Trilateral Cooperation Secretariat in Seoul in September 2011. In this respect,
CJK can be said to have further advanced than the Association of Southeast Asian Nations plus China, Japan and South Korea
(ASEAN+3) framework (Qin and Wei 2008). Moreover, the three economy ministers in May 2012 signed an Agreement for the Promotion,
Facilitation and Protection of Investment. The trilateral framework is currently being pushed forward through negotiations of a trilateral free-
trade agreement (FTA) (MOFA 2013a). These formalized institutions emerged on the background of a wide range of trilateral activities, which
encompass 50 consultative mechanisms including 18 ministerial-level frameworks and over a 100 cooperative projects (TCS 2013). In short,
neither pessimistic approaches with their focus on power shift nor optimistic analyses of regional cooperation are sufficient for the analysis of
recent developments that are characterized by the simultaneous accentuation of conflict and cooperation.

Attempts to establish “stability” in Asia reinforce static identity concepts that


oversimplify the region – reproduces hegemonic power.
Wirth 15 – analyst at Griffin University Asia Institute
(Christian, “‘Power’ and ‘stability’ in the China–Japan–South Korea regional security complex,” The
Pacific Review, Volume 28, Issue 4, pgs. 553-575)

Overshadowed by research on relations between great powers, Washington's alliance relationships with Seoul
and Tokyo and studies of ASEAN-centred regionalism, international relations (IR) literature has been rather silent when it
comes to the explanation of relations between South Korea and its two neighbours and their implications for
Northeast Asian international politics. The frequent foci on triangles, which by default, include the United States and exclude either Korea or
China is another manifestation of this phenomenon. As
a consequence, analysts fail to take into account fundamental
commonalities between China, Japan and South Korea : historical amity and enmity, cultural and civilizational similarities,
models of state-guided economic development, and geographical proximity apparent in ecological interdependence constitute Northeast
Asia as a regional security complex (RSC) overlaid by US power (Buzan and Wæver 2003). This conceptualization
allows for separate perspectives on intra-regional dynamics and their interplay with extra-regional factors. However, when it comes to show
how the various factors and levels come together in leading to change in international relations, the approach falls
back to static
descriptions of altering polarities among territorially, precisely delineated regional and national units classified
according to their power status defined in material terms, and the determining roles of great powers .
Limited to the discussion of effects of power shifts on specific bilateral relations among great powers, other explanations of conflict bear less
insights and need not be rehearsed here.¶ Analyses that are less pessimistic and focus more on cooperation adopt regional perspectives. Kang
(2007) showed that inconsistencies
in what he terms Western-centred IR theorizing led to overly pessimistic
accounts of East-Asian futures. As recent developments suggest, however, his interpretation of events swung towards the other
extreme essentially replacing pessimistic views characterized by balancing against China with an optimistic one in which Asian states
bandwagon with China.¶ Beyond the balancing–bandwagoning dichotomy, while seeing the keys in the hands of Chinese and Japanese leaders,
Rozman (2004, p. 3, 349) recognizes the ‘pivotal position’ of South Korea. He finds that because of ‘modernization without sufficient
globalization’, nationalism has been prevailing over regionalism. Thus, the question is how narratives of national identity and the contingent
antagonistic relations including the pertaining threat perceptions may or may not alter. Integration of political communities is predicated upon
a positive, dynamic and reciprocal relationship between the structure of the region defined in material power and knowledge, and social
processes defined by organizations, transactions and social learning (Adler and Barnett 1998). Adler and Barnett contend that power
understood as the authority that determines shared meaning plays a major role in the development and maintenance of a security community.
Powerful states, however, do not create security per se. Rather, because of the positive image of
security or material progress that is associated with powerful and successful states, security
communities can form around them (Adler and Barnett 1998).¶ Since both Beijing and Washington vie for spheres of influence in
the Western Pacific, sub-regional groupings aimed at restraining a hegemon (Hurrell 1995) have, unlike in Southeast Asia, not emerged.
Therefore, termed ASEAN centrality, Southeast Asian states have commonly perceived themselves as being in ‘the driver's seat’ of regional
cooperation (Ho 2012). While competition between Beijing, Tokyo and Seoul for influence in ASEAN-based regional projects generated positive
dynamics such as exemplified by the ASEAN+1 and ASEAN+3 frameworks (Breslin 2010; Nabers 2010), trilateral cooperation among China,
Japan and South Korea follows different logics. Agenda-setting was often determined by Seoul (Pieczara 2012). The Kim Dae-Jung and Roh Moo-
hyun administrations in particular promoted East Asian regionalism as a means to overcome Korea's uncomfortable position of ‘a shrimp
among whales’ (Shim 2009). A closer look behind the official rhetoric of regional cooperation in non-traditional security issues, however, reveals
that neo-functionalism and transactionalism can hardly account for more than the improvement of atmospherics at diplomatic meetings. The
reason is that, apart from the protection of state-centred notions of security, Northeast Asian governments put utmost premium on economic
growth, and they do this through mercantilist lenses (Wirth 2010).¶ This
conflict-prone and rapidly evolving political
environment led to the mantra of stability. Even studies put emphasis on non-traditional security issues, the
need for institution-building and the promotion of democracy and open markets equate stability with a
continued, ‘robust’ presence of the United States for the purpose of deterrence and the maintenance
of the status quo (Campbell et al. 2008; Nishihara 2012; Tellis 2011; Yan 2010). As with the concept of power, however, stability is
rarely defined beyond an abstract balance of power, let alone operationalized (Choi and Moon 2010).¶
Notions of power shift, balance of power and balance of threat are , despite the conceptual differences, essentially
based on material definitions of power. What drives the discourse is the size of national economies and their
(projected) relative changes extrapolated from gross domestic product (GDP) growth rates (Beckley 2011/12). The
dominant view is that a rising and, therefore, inherently revisionist, assertive China challenges the
stable status quo of US hegemony and threatens Japan (DoD 2012; NIDS 2012).¶ Thus, power shift is constituted by
what Lukes (2005) terms one-dimensional view of power. It is based on the claim that the exercise of power
can be observed by analysing behaviour, that is, by looking at what decisions are taken. A more
sophisticated, two-dimensional view takes into account that power can also be exercised by keeping
issues out of decision-making processes. Even though the second view goes beyond quantifying potentials, both conceptualizations
of power are incomplete because they presume an observable conflict of interests. This is apparent in definitions such as by Keohane and Nye
(2001) for whom power is ultimately about making another actor do or want something that it would otherwise not. This linear, one-
directional conceptualization of power creates a ‘theoretical tunnel vision’ (Barnett and Duvall 2005, p. 40),
which lies at the core of the hierarchy vs. anarchy, balancing vs. bandwagoning and democratic vs.
authoritarian dichotomies. Therefore, rather than to try to operationalize a single, ‘correct’ definition of power, it is important
to be aware that power simultaneously works in different ways in several dimensions. This requires us to understand
power as a relational quality. Consequently, it is not only material capabilities but also systems of knowledge and
discursive practice that need to be taken into account (Barrett 2002; Guzzini 2000; Mann 2012). Barnett and Duvall (2005)
suggest a fourfold taxonomy of power along two dimensions: the kinds of social relations through which power works; and the specificity of
social relations through which effects on actor's capacities are produced. Of the four partially overlapping conceptions of power: compulsory,
institutional,; structural and productive, the first and fourth are particularly relevant for the purpose of this study. Compulsory power
corresponds with Lukes' (2005) one-dimensional view. Due to its reliance on capabilities, it inherently focuses on great power relations. In
contrast, productive power that is ‘the constitution of all social subjects with various social powers through systems of knowledge and
discursive practice’ goes beyond material capabilities and linear, unidirectional relations of power (Lukes 2005, p. 55).
Link – China
The China threat is constructed by biased Western studies that use one-sided
interpretations of geopolitics to twist evidence in service of Washington’s national
security agenda
Hirono & Suzuki 14 (Miwa & Shogo, senior research fellow @ China Policy Institute “Why Do We
Need ‘Myth-Busting’ in the Study of Sino–African Relations?,” Journal of Contemporary China, Volume
23, Issue 87)

For Western scholars, studies of China's international relations are generally still in the service of the national
security agenda of the Western policy community. Historically, the study of the non-West has been closely
linked to Western government policies. Long before the term ‘area studies’ came into existence in the United States (US) in the 1950s, there
was a notion that scholarship should serve the political goals of the elite . ‘Oriental studies’ was recognized as a discipline in the
eighteenth and nineteenth centuries, due to Western colonialists' needs to control non-Western people.7 For example, Sir William Jones, a
British legal scholar, founded the Asiatic Society of Bengal in 1784, and also worked as an official of the British East India Company; he felt
absolutely no conflict of interest in serving imperialism and set the pattern which later Orientalists and area studies experts emulated.8 In
addition to those who engaged in commercial activities, Christian missionaries laid the foundation for the development of Orientalism, with a
belief that the West should help the non-West adopt ‘superior’ Western civilization .9 Edward Said rightly states
that the purpose of Orientalism was ‘to understand , in some cases to control, manipulate, even incorporate ,
what is a manifestly different world’.10 Later, social anthropologists joined the Orientalists by bringing more in-depth understanding
of the customs and lifestyles of the colonized, in order to aid colonial administrators and missionaries.11 However, it was only after World War
II (WWII) that area studies really flourished. One of the earliest and best known examples of this genre of scholarship was The Chrysanthemum
and the Sword: Patterns of Japanese Culture, written by American anthropologist Ruth Benedict in 1946. This was the product of Benedict's
involvement with the US Office of War Information during WWII.12 As the Cold War and the subsequent standoff between the US and the
Soviet Union became entrenched in the 1950s, area studies served to fulfill the West's strategic need to understand the enemy or processes by
which states hostile to Western interests could be brought into the so-called ‘free world’. Research
on Asia, particularly China
and Japan, was a major beneficiary of this development. As Bruce Cummings states: Japan got a favored
placement as a success story of development, and China got obsessive attention as a pathological
example of abortive development. The key processes were things like modernization, or what was for many years called ‘political
development’ toward the explicit or implicit goal of liberal democracy.13 Cummings also documents various area studies departments' close
working relations with US government agencies in the early Cold War period, particularly their role in providing a steady source of new recruits
and specialist consultants: ‘For those scholars studying potential enemy countries, either they consulted with the government or they risked
being investigated by the FBI; working for the CIA thus legitimized academics and fended off J. Edgar Hoover’.14 While the Cold War has
officially ended with the collapse of the Soviet Union, there still remains ample state demand for the study of China ,
partly because of the ‘concern’ of Western elites that China is the only strategically competitive peer that could pose a real threat to the West's
power and dominance. China is the last remaining communist great power, and its antipathy to liberal
democratic governance, coupled with its steady military build-up, has made it a latent ‘threat’ to
Western interests. Therefore, the research agenda of Chinese foreign policy in Africa continues to be influenced
by this national interest. For example, the US Congressional Research Service (CRS) has published five reports between 2008
and 2009 on China's activities in Africa, Latin America and Southeast Asia, showing heightened US interests and
anxieties in this field.15 Furthermore, China is the only non-African country to feature in the CRS's reports on Africa. Studies of the US or
Europe in Africa are conspicuously absent.16 Scholars and analysts have also jumped on this policy bandwagon, and
published a series of works that confirm China's ‘threat’ in Africa. Authors of these works voice their disquiet that the
Chinese government is trying to sabotage Western attempts to introduce ‘good governance’ (such as liberal democratic governance or
improved transparency) by propagating its model of ‘authoritarian capitalism’. Many also warn darkly that an important part of China's
objectives in Africa is to challenge US global hegemony.17 China's non-conditional aid—denounced as ‘rogue aid’ by some critics—and trade-
oriented relations with ‘rogue states’ such as Sudan and Zimbabwe are frequently criticized, because they undermine attempts to introduce
democracy to the region, provide a lifeline for autocratic rulers, and encourage and exacerbate human rights abuses by them.18 In the words of
Gemot Pehnelt, ‘Chinese engagement enables African governments to reject demands made by the IMF, the World Bank and other donors for
enhancing transparency, implementing anti-corruption strategies, and furthering their democratization efforts’.19 China's priority is, these
scholars argue, simply to secure energy resources rather than to improve human rights conditions in those states.20 Chinese firms (often
bundled together under the somewhat misleading label of ‘China’ or ‘China Inc.’) are also accused of neocolonial behavior, such as exploiting
African workers, flooding the African market with cheap Chinese consumer goods and ruining the local economy, or stripping African states of
their resources.21 It is, of course, necessary to acknowledge that not
all works portray China as a threat , as evidenced from the
‘myth-busting’ literature. Brautigam demonstrates that much of China's allegedly pernicious political influence
is greatly exaggerated, and that its aid can, at times, actually deliver real benefits to the recipient states.22 China's aid to
authoritarian leaders has not been as vast as is often claimed , and is not as susceptible to being misused. With regard to
weapons trade, Western corporations also engage in arms trade with rogue states (at times far more than
the Chinese), making Western criticisms of Chinese weapons sales ring somewhat hollow—in fact, a
recent study has concluded that the US ‘tends to transfer conventional arms to authoritarian regimes to
a greater extent than does China, which in turn tends to export more to African democracies and regimes that generally respect
human rights’.23 Studies on China's development activities in Africa have also found that Beijing's role in propping up isolated African autocrats
is greatly exaggerated: there is, for instance, no concrete evidence of a systematic attempt to export China's development model of
authoritarian capitalism, whose existence is highly debatable.24 African scholars such as Adekeye Adebajo have also pointed out that the US
has provided ‘support for a cantankerous warlord's gallery’,25 which again reminds us that many governments support undemocratic regimes,
provided that it is in accordance with their national interests. With regard to economic exploitation of African labor, Chinese
enterprises are found to be neither better nor worse than many of their Western counterparts , and their
buying up of African natural resources often pales into insignificance compared to Western purchases. Yet, it is important to note that even
this type of literature is essentially an extension of the same question that dominates the Western
policy community: does China's rise present a threat to Western interests in Africa? The starting point of
their enquiry is the same as that of their respective governments' national security concerns. This close link
between the academic and national security agendas in the West suggests that there is still a key
governmental interest in understanding the Chinese ‘enemy’ . In the latter half of the twentieth century, the core
objective of US foreign policy was to combat communist regimes and advance liberalism and capitalism. After the end of the Cold War, US
foreign policy has strived for the maintenance of US hegemony or, at least, US strategic superiority vis-à-vis China.
Therefore, the US government has encouraged social scientists to study subjects that assist such policy
purposes. This is demonstrated, for example, by the fact that state funding for Asia-related topics has been linked to
government programs such as the National Security Education Act.26 In addition, since 2008, the US Department of Defense has
provided a number of million-dollar-level Minerva Research Initiative funds to university-based social science
research programs, focusing on ‘areas of strategic importance to US national security policy’ .27 It has seven priority
research topics, including science, technology and military transformation in China and developing states . China is the only
country specifically mentioned in all seven priority research topics . This tendency of government funding policies
fostering close links between academic and policymaking communities is replicated in other states as well. In the United Kingdom (UK), the
Research Excellence Framework (REF) and Research Councils UK pay particular attention to the importance of ‘non-academic impact’,28 such as
that on political decision making. This includes ‘fostering global economic performance, and specifically the economic competitiveness of the
United Kingdom; increasing the effectiveness of public services and policy; enhancing quality of life, health and creative output’.29 According to
this academic strategy, one of the most effective ways to make non-academic impact in area studies (as well as other academic disciplines, for
that matter) would be to demonstrate their utility in achieving these goals set by the policy community. The prospect of a positive evaluation of
research impact, which leads to increased state funding to universities, coupled with greater opportunities for applying for external funding,
could serve to encourage research that addresses the interests of the policy elites. Nevertheless, Stephen Walt, Joseph Nye and
Alexander George argue that the gap between scholars and policy community is ever growing .30 They claim
that policymakers tend to ignore academic research because of its irrelevance to their day-to-day work of
policymaking.31 While this gap may be the case for the disciplines of Political Science and International Relations since WWII
in general, China-specific discussions have remained, as discussed above, closely related to the strategic
interests of the state. When area studies falls into this trap of policy research, academic research tends to end up
focusing on a predictable and stereotyped agendum that fit with national interests . This results in what Amitav
Acharya calls a state of ‘entrapment’, which … occurs when scholars, after having offered consequential intellectual input at an early stage of
policymaking …, remain beholden to the choices made by officials and thereby [become] unwilling or
incapable of challenging officially sanctioned pathways and approaches for the fear of losing their access and influence.32
It is not our intention to claim that the close link between academia and policy community is necessarily problematic. However, what we need
to be vigilant about is the tendency for policy needs to influence the academic research agenda in the study of Sino–African relations, rather
than the other way around (i.e. academic research agenda influencing policy direction). Other options for academia include deliberately
maintaining intellectual distance from the policy community, so that scholars can freely advance their research without being constrained by
structural and political obstacles that the policy community faces. It
is problematic that much of the scholarship on Sino–
African relations remains focused on whether or not China is a threat: such a debate is influenced by and
remains confined to Western governments' strategic interests, and could crowd out the intellectual
space for alternative research topics. This is not to say that all literature that revolves around the China threat and ‘myth-
busting’, necessarily seeks to inform Western policymakers and their interests.33 Particularly, scholars outside Western academic circles often
do not have ties with Western policymakers and may regard the latter as antagonistic to African and/or Chinese interests. However, research
on Sino–African relations undertaken by Chinese analysts has frequently been reactive to Western debates of Sino–African relations, which
results in defensive essays refuting Western criticisms of the PRC's role in Africa.34 Ironically, this only serves to further entrench the ‘myth-
busting’ narrative, irrespective of their intention to inform or not to inform Western policymakers. The
close link between the
national security agenda and the academic literature in the West can be also seen as a by-product of
Western exceptionalism that remains prevalent in the discipline of IR. As a field of study which emerged in the West
as a self-conscious academic discipline, Acharya and Barry Buzan argue, it is almost a truism to say that ‘the main ideas in
this discipline are deeply rooted in the particularities and peculiarities of European history, the rise of
the West to world power, and the imposition of its own political structure onto the rest of the world’ .35
This cultural/geographical bias has often resulted in a somewhat one-sided interpretation of global
order, in that Western dominance is seen as progressive and thus the only form of hegemony that matters
historically and normatively. The rise of a non-Western state or non-Western hegemony is both poorly
theorized and almost axiomatically seen as a threat. The in-built Eurocentric biases of IR theory have
resulted in the European regional order being conceptualized as a product of something rational and
liberal, as it ensured the survival of individual sovereign states and prevented the emergence of a
(universal) empire.36

Sino relations are a function of societal perceptions, not government diplomacy—their


fear-laden conception of them makes relations impossible
Liss 03 – (Journal of Contemporary China (2003), 12(35), 299–318, Images of China in the American
Print Media: a survey from 2000 to 2002, Alexander Liss is an M.A. student in the Asian Studies Program
at the Elliott School of International Affairs, George Washington University,

American society’s images and perceptions of China have had several recurring themes over the years.
In the past, some of these have included the perception of China as a potential market for American goods and as
a potential supply of converts for American missionaries. These images changed during the years of diplomatic isolation of

the Cold War, turning the Chinese into a vast horde of ‘reds’, a faceless, invincible mass that threatened all of
Asia. In the post-Cold War world, Sino–US relations face an uncertain future. The time is not far off when there will again be two superpowers, and there is the
potential for conflict between them. In this new era, it is interesting to examine what images of China have emerged in contemporary American society. The goal of
this paper is to do just that. By examining articles about China in four major American daily newspapers, over a three-year period, a rough sketch emerges of how
China is perceived to the ‘average’ reader of these four publications. These images, while interesting in their own right, also provide a valuable benchmark for the
direction of Sino–US relations. Overall, it seems that, just as in past periods of rivalry, negative images of China overwhelm the positive. But, before we can conclude
that the current relationship is also one of competition, there are also some significant images of a country whose future lies entwined with the US in a partnership,
not a battle. If we can take the articles of this study as a representative slice of American society at large, the general trend seems to be one in which, although

China is sometimes viewed in a harsh and critical light, there is still hope for the two countries to come
together—or even for China to become more like the United States. The relationship between the
United States and China works on many levels and involves many actors. The phrase ‘Sino–US relations’
usually brings to mind an image of interaction between the governments of each country. Yet, if we
merely examine the diplomacy between the two countries, then we are left with an incomplete picture
of the forces that affect how the nations engage each other. A key element to consider is the
relationship between the two societies. Popular opinion and popular perceptions of each culture in the
eyes of the other are far more subtle elements to consider, yet they are no less important than the
official acts of government, and indeed, may even be more so.

Research is influenced by US imperialism by emphasizing Chinese foreign practices


that directly compete with the US and being funded by the government
Hirono & Suzuki 14 – senior research fellow @ China Policy Institute
(Miwa & Shogo, “Why Do We Need ‘Myth-Busting’ in the Study of Sino–African
Relations?,” Journal of Contemporary China, Volume 23, Issue 87)
For Western scholars, studies of China's international relations are generally still in the service of the
national security agenda of the Western policy community . Historically, the study of the non-West has been
closely linked to Western government policies . Long before the term ‘area studies’ came into existence in the United States
(US) in the 1950s, there was a notion that scholarship should serve the political goals of the elite. ‘ Oriental studies’ was recognized
as a discipline in the eighteenth and nineteenth centuries, due to Western colonialists' needs to control
non-Western people.7 For example, Sir William Jones, a British legal scholar, founded the Asiatic Society of Bengal in 1784, and also
worked as an official of the British East India Company; he felt absolutely no conflict of interest in serving imperialism and set the pattern which
later Orientalists and area studies experts emulated.8 In addition to those who engaged in commercial activities, Christian missionaries laid the
foundation for the development of Orientalism, with a belief that the West should help the non-West adopt ‘superior’ Western civilization.9
Edward Saidrightly states that the purpose of Orientalism was ‘to understand, in some cases to control,
manipulate, even incorporate, what is a manifestly different world ’.10 Later, social anthropologists joined
the Orientalists by bringing more in-depth understanding of the customs and lifestyles of the colonized,
in order to aid colonial administrators and missionaries. 11 However, it was only after World War II (WWII) that area
studies really flourished. One of the earliest and best known examples of this genre of scholarship was The Chrysanthemum and the Sword:
Patterns of Japanese Culture, written by American anthropologist Ruth Benedict in 1946. This was the product of Benedict's involvement with
the US Office of War Information during WWII.12 As the Cold War and the subsequent standoff between the US and the Soviet Union became
entrenched in the 1950s, area studies served to fulfill the West's strategic need to understand the enemy or
processes by which states hostile to Western interests could be brought into the so-called ‘free world ’.
Research on Asia, particularly China and Japan, was a major beneficiary of this development . As Bruce
Cummings states: Japan got a favored placement as a success story of development, and China got obsessive attention as a
pathological example of abortive development . The key processes were things like modernization, or what was for many years
called ‘political development’ toward the explicit or implicit goal of liberal democracy.13 Cummings also documents various area
studies departments' close working relations with US government agencies in the early Cold War period,
particularly their role in providing a steady source of new recruits and specialist consultants: ‘ For those scholars studying potential
enemy countries, either they consulted with the government or they risked being investigated by the
FBI; working for the CIA thus legitimized academics and fended off J. Edgar Hoover’ .14 While the Cold War has
officially ended with the collapse of the Soviet Union, there still remains ample state demand for the study of China,
partly because of the ‘concern’ of Western elites that China is the only strategically competitive peer
that could pose a real threat to the West's power and dominance . China is the last remaining communist great power,
and its antipathy to liberal democratic governance, coupled with its steady military build-up, has made it a latent ‘threat’ to Western interests.
Therefore, the
research agenda of Chinese foreign policy in Africa continues to be influenced by this
national interest. For example, the US Congressional Research Service (CRS) has published five reports between 2008 and 2009 on
China's activities in Africa, Latin America and Southeast Asia, showing heightened US interests and anxieties in this field.15 Furthermore,
China is the only non-African country to feature in the CRS's reports on Africa. Studies of the US or
Europe in Africa are conspicuously absent. 16 Scholars and analysts have also jumped on this policy bandwagon, and published a
series of works that confirm China's ‘threat’ in Africa. Authors of these works voice their disquiet that the Chinese
government is trying to sabotage Western attempts to introduce ‘good governance’ (such as liberal democratic
governance or improved transparency) by propagating its model of ‘authoritarian capitalism’ . Many also warn darkly
that an important part of China's objectives in Africa is to challenge US global hegemony .17 China's non-
conditional aid—denounced as ‘rogue aid’ by some critics—and trade-oriented relations with ‘rogue
states’ such as Sudan and Zimbabwe are frequently criticized, because they undermine attempts to
introduce democracy to the region, provide a lifeline for autocratic rulers, and encourage and
exacerbate human rights abuses by them .18 In the words of Gemot Pehnelt, ‘Chinese engagement enables African governments
to reject demands made by the IMF, the World Bank and other donors for enhancing transparency, implementing anti-corruption strategies,
and furthering their democratization efforts’.19 China's priority is, these scholars argue, simply to secure energy resources rather than to
improve human rights conditions in those states.20 Chinese firms (often bundled together under the somewhat misleading label of ‘China’ or
‘China Inc.’) are also accused of neocolonial behavior, such as exploiting African workers, flooding the African market with cheap Chinese
consumer goods and ruining the local economy, or stripping African states of their resources.21 It is, of course, necessary to acknowledge that
not all works portray China as a threat , as evidenced from the ‘myth-busting’ literature . Brautigam demonstrates
that much of China's allegedly pernicious political influence is greatly exaggerated, and that its aid can, at times, actually deliver real benefits to
the recipient states.22 China's aid to authoritarian leaders has not been as vast as is often claimed, and is not as susceptible to being misused.
With regard to weapons trade, Western corporations also engage in arms trade with rogue states (at times far more than the Chinese), making
Western criticisms of Chinese weapons sales ring somewhat hollow—in fact, a recent study has concluded that the US ‘tends to transfer
conventional arms to authoritarian regimes to a greater extent than does China, which in turn tends to export more to African democracies and
regimes that generally respect human rights’.23 Studies on China's development activities in Africa have also found that Beijing's role in
propping up isolated African autocrats is greatly exaggerated: there is, for instance, no concrete evidence of a systematic attempt to export
China's development model of authoritarian capitalism, whose existence is highly debatable.24 African scholars such as Adekeye Adebajo have
also pointed out that the US has provided ‘support for a cantankerous warlord's gallery’,25 which again reminds us that many governments
support undemocratic regimes, provided that it is in accordance with their national interests. With
regard to economic
exploitation of African labor, Chinese enterprises are found to be neither better nor worse than many of
their Western counterparts, and their buying up of African natural resources often pales into
insignificance compared to Western purchases . Yet, it is important to note that even this type of literature is
essentially an extension of the same question that dominates the Western policy community: does
China's rise present a threat to Western interests in Africa? The starting point of their enquiry is the same as that of their
respective governments' national security concerns. This close link between the academic and national security agendas in the West suggests
that there is still a key governmental interest in understanding the Chinese ‘enemy’ . In the latter half of the
twentieth century, the core objective of US foreign policy was to combat communist regimes and advance liberalism and capitalism. After
the end of the Cold War, US foreign policy has strived for the maintenance of US hegemony or, at least,
US strategic superiority vis-à-vis China. Therefore, the US government has encouraged social scientists to
study subjects that assist such policy purposes . This is demonstrated, for example, by the fact that state funding for
Asia-related topics has been linked to government programs such as the National Security Education
Act.26 In addition, since 2008, the US Department of Defense has provided a number of million-dollar-level Minerva Research Initiative funds
to university-based social science research programs, focusing on ‘areas of strategic importance to US national security policy’.27 It has seven
priority research topics, including science, technology and military transformation in China and developing states. China is the only country
specifically mentioned in all seven priority research topics. This tendency of government funding policies fostering close links between
academic and policymaking communities is replicated in other states as well. In the United Kingdom (UK), the Research Excellence Framework
(REF) and Research Councils UK pay particular attention to the importance of ‘non-academic impact’,28 such as that on political decision
making. This includes ‘fostering global economic performance, and specifically the economic competitiveness of the United Kingdom; increasing
the effectiveness of public services and policy; enhancing quality of life, health and creative output’.29 According to this academic strategy, one
of the most effective ways to make non-academic impact in area studies (as well as other academic disciplines, for that matter) would be to
demonstrate their utility in achieving these goals set by the policy community. The prospect of a positive evaluation of research impact, which
leads to increased state funding to universities, coupled with greater opportunities for applying for external funding, could serve to encourage
research that addresses the interests of the policy elites. Nevertheless, Stephen Walt, Joseph Nye and Alexander George argue that the gap
between scholars and policy community is ever growing.30 They claim that policymakers tend to ignore academic research because of its
irrelevance to their day-to-day work of policymaking.31 While this gap may be the case for the disciplines of Political Science and International
Relations since WWII in general, China-specific discussions have remained, as discussed above, closely related to
the strategic interests of the state . When area studies falls into this trap of policy research, academic research tends to end up
focusing on a predictable and stereotyped agendum that fit with national interests. This results in what Amitav Acharya calls a
state of ‘entrapment’, which … occurs when scholars, after having offered consequential intellectual
input at an early stage of policymaking …, remain beholden to the choices made by officials and thereby
[become] unwilling or incapable of challenging officially sanctioned pathways and approaches for the
fear of losing their access and influence.32 It is not our intention to claim that the close link between academia and policy
community is necessarily problematic. However, what we need to be vigilant about is the tendency for policy needs
to influence the academic research agenda in the study of Sino–African relations, rather than the other
way around (i.e. academic research agenda influencing policy direction). Other options for academia include deliberately
maintaining intellectual distance from the policy community, so that scholars can freely advance their
research without being constrained by structural and political obstacles that the policy community faces .
It is problematic that much of the scholarship on Sino–African relations remains focused on whether or not China is a threat: such a debate is
influenced by and remains confined to Western governments' strategic interests, and could crowd out the intellectual space for alternative
research topics.
Link – Climate Cooperation
Belief in global cooperation’s ability to solve obscures the causes of overexploitation
and creates a utopian belief in technology’s ability to solve.
Ahmed 11—Nafeez Mosaddeq Ahmed is an international security analyst. He is Executive Director at the Institute for Policy Research and
Development, and Associate Tutor at the Department of IR, University of Sussex, where he obtained his DPhil. [“The international relations of
crisis and the crisis of international relations: from the securitisation of scarcity to the militarisation of society,” Global Change, Peace &
Security, Volume 23, Issue 3, 2011, Taylor and Francis Online]

2.3 Neoliberalism: mutual over-exploitation as normative On the other hand, we have strategies of international cooperation to establish new
global governance regimes by which states can develop treaties and agreements to encourage mitigating action. It is now clear that the massive
proliferation of international legal treaties designed to regulate activities impacting detrimentally on the environment and thus limit
environmental degradation simply cannot be explained under the realist theoretical framework. While this seemingly vindicates neoliberal
theoretical approaches which underscore the scope for rational state strategies of mutual cooperation,62 the latter are still at a
loss to explain the extent to which ethical norms and values, national cultures and environmental and
scientific advocacy underpin wide-ranging environmental regimes which cannot be reduced purely to
state interests.63 Much of the liberal literature also explores the regressive dynamic of the energy industry and its international
dimensions, though failing to escape realist assumptions about anarchy. Kaldor and her co-authors, for instance, note that conflicts can erupt in
regions containing abundant resources when neopatrimonial states collapse due to competition between different ethnic and tribal factions
motivated by the desire to control revenues.64 Similarly, Collier argues that the most impoverished populations inhabit the most resource-
wealthy countries which, however, lack robust governance, encouraging rampant internal resource predation and therefore civil wars.65 Lack
of robust governance thus facilitates not only internal anarchy over resource control, but also the illicit and corrupt activities of foreign
companies, particularly in the energy sector, in exploiting these countries.66 This
sort of analysis then leads to a staple set of
normative prescriptions concerned largely with ways of inculcating ‘good governance’, such as
transparency measures to avoid excessive secrecy under which oil companies indulge in corruption;
more robust international regulation; corporate social responsibility; and cosmopolitan principles such
as democratisation, political equality and freedom of civil society .67 Yet such well-meaning
recommendations often do not lead to sufficiently strong policy action by governments to rein in energy
sector corruption.68 Furthermore, it is painfully clear from the examples of Kyoto, Copenhagen and Cancun
that international cooperative state strategies continue to be ineffective, with states unable to agree on
the scale of the crises concerned, let alone on the policies required to address them. Indeed, while some
modest successes were apparent in the Cancun Accord, its proposed voluntary emissions regime would
still likely guarantee – according to even mid-range climate models – a global average temperature rise of 4°C or
more, which would in turn culminate in many of the IPCC's more catastrophic scenarios. 69 This calls into
question the efficacy of longstanding recommendations – such as Klare's – that the international community
develop unprecedented international mechanisms to coordinate the peaceful distribution of natural
resources in the era of scarcity and environmental degradation.70 While at face value such regulatory governance
mechanisms would appear essential to avoid violent conflict over depleting resources, they are posited
in a socio-political and theoretical vacuum. Why is it that such potentially effective international
mechanisms continue to be ignored? What are the socio-political obstacles to their implementation? Ultimately, the problem
is that they overlook the structural and systemic causes of resource depletion and environmental
degradation. Although neoliberalism shares neorealism's assumptions about the centrality of the state as a unitary rational actor in the
international system, it differs fundamentally in the notion that gains for one state do not automatically imply losses for another; therefore
states are able to form cooperative, interdependent relationships conducive to mutual power gains, which do not necessarily generate tensions
or conflict.71 While
neoliberalism therefore encourages international negotiations and global governance
mechanisms for the resolution of global crises, it implicitly accepts the contemporary social, political and
economic organisation of the international system as an unquestionable ‘given’, itself not subject to
debate or reform.72 The focus is on developing the most optimal ways of maximising exploitation of the
biophysical environment. The role of global political economic structures (such as centralised private resource-
ownership and deregulated markets) in both generating global systemic crises and inhibiting effective means
for their amelioration is neglected. As such, neoliberalism is axiomatically unable to view the biophysical
environment in anything other than a rationalist, instrumentalist fashion, legitimising the over-
exploitation of natural resources without limits, and inadvertently subordinating the ‘global commons’
to the competitive pressures of private sector profit-maximisation and market-driven solutions, rather
than institutional reform.73 Mutual maximisation of power gains translates into the legitimisation of the
unlimited exploitation of the biophysical environment without recognition of the human costs of doing
so, which are technocratically projected merely as fixable aberrations from an optimal system of
cooperative progress.74 Consequently, neoliberalism is powerless to interrogate how global political
economic structures consistently undermine the establishment of effective environmental regimes .
Link – Environment
Environmental apocalyptic rhetoric demotivates agency. It cultivates denialism and
fatalism, and aesthetisizes catastrophe – makes solvency impossible
Johnson 10 – PhD candidate in philosophy from the department of English @ Indiania University
(Laura Boynton, “UNEARTHING ENVIRONMENTAL AGENCY,” December 2010, PhD thesis submitted to
Indiana University)
My project begins with my concerns about this strategy, which are several. Primary is its tendency to be read (or misread, as I explore in my
definition of apocalypticism below) as apocalyptic. While litanies of disaster aim to inspire a fear rhetors likely see as
prerequisite to audience action, the overwhelming presence they give to environmental degradation
means that any action they inspire is likely to be hampered by doubts about the effectiveness of that
action: instead of engaging as activists, audiences might “flinch, [...] pull back and say, ‘Wait a minute—
things can’t be that bad’” (Slovic, “Epistemology” 105). Moreover, audiences may misread degradation narratives
as predictions—or, considering Carson’s use of the past tense, as foregone conclusions—leading them to conclude that
environmental problems are irreparable and disaster inevitable, or already upon us. The fear degradation narratives
inspire “is not simply a fear that we will deplete a particular natural resource, lose pristine wilderness, or be poisoned. It is the belief
that we may well be facing the ‘end of history,’ that we as a species might be doomed ” (Lutts 37). Fear on
this level is more likely [freezing]* paralyzing than motivating. Scott Slovic notes that the “great advantage of the
jeremiad” (exemplified for him by Silent Spring) “is in the shock effect, which leads to immediate, albeit short-lived,
awakening” (“Epistemology” 105). While Slovic goes on to argue that environmental jeremiads ultimately lead to fundamental shifts in
values for audiences (and are therefore epistemological), I want to consider the not-contradictory possibility that an audience’s “short-
lived[...] awakening” quickly yields to feelings of powerlessness to avert the scenarios described.
Cumulative powerlessness may accompany the pro-environmentalist shift in values for which Slovic argues, but
it may equally lead to apathy or paralyzed inaction. But, the counter-argument runs, Silent Spring’s position at the helm of
environmentalism suggests the effectiveness more than the dangers of apocalypticism (if, that is, we accept “A Fable for Tomorrow” as
apocalypticiii). Indeed the radical success of Silent Spring has even been attributed to the Fable’s apocalypticism (Waddell 9),iv suggesting that
apocalypticism is anything but demotivating. Yet to argue that the Fable’s apocalypticism accounts for the text’s inspiration of a political
movement overlooks Carson’s elsewhere more scientific character, attributing excessive importance and power to the ‘apocalypticism’ of the
Fable. It also oversimplifies the historical situation in which she published by failing to consider that the Fable’s environmental
apocalypticism may have represented a break with rhetorical tradition in 1962, a strategy whose novelty
was more persuasive than its content . Thus we might credit the Fable with beginning the environmental movement less because
it is ‘apocalyptic’ in character than because, in 1962, predictions of environmental apocalypse had not yet hardened
into commonplace. Decades later, apocalyptic environmental commonplace has outlived its utility as a
constructor of exigence. As cliché, it reinscribes notions of environmentalists as negative, fault-finding
advocates of limits; as hysterical, overly dramatic alarmists (see Patrick 143); or as cultish or extreme (echoing
stereotypes of religious apocalyptics). The conventionalization of apocalypticism also means that environmental
texts devote disproportionate attention to the exigence they use it to construct. This in turn sets
environmentalism up as always needing to justify itself to audiences and defend the realities of
ecological decline, contributing to audiences’ skepticism towards environmental rhetors: surely, if they must
devote such energy to defending credibility, then their credibility —and the credibility of their movement—is
questionable indeed. A further objection is that these vivid scenarios risk aestheticizing the very possibilities
they forecast in effort to prevent. Narratives of collapse stand out as creative moments in otherwise
occasionally dry works. Their descriptiveness—as well as their intertextual resonance with works of religious apocalyptic—likely
causes them to swell to mythic proportions within these works.v If they aestheticize or fetishize, even
inadvertently, that fetishization might then undercut the texts’ insistence that collapse scenarios are
to be avoided. It would also contribute to the [inaction] * paralysis such scenarios are likely to inspire,
[inaction] a paralysis born of helplessness (as we are faced with such a monumental view of the ecological
havoc we wreak that we doubt our capacity to intervene) but also of awe (as we stare at walls built of
beautiful prose, as perversely hypnotized as moth to flame). In short, apocalyptic environmental rhetorics are likely to
foreclose the very paths of action they seek to inspire . In dwelling so heavily on ecological problems, they immediately
undercut our faith in our capacity to address or solve these problems; that is to say, they impoverish agency. From Apocalypse to Way
of Life, Frederick Buell’s history of American environmental crisis thought, notes that “[f]aith in effective action has diminished
at the same time that the concern about the gravity of the [environmental] crisis has sharpened ” (xvii).
Once agency (“[f]aith in effective action”) is foreclosed, rhetoric ceases to function as such because it can
motivate no action.vi Thus my project adopts as premise that successful environmental rhetorics must construct environmental agency,
by which I indicate audiences’ faith in their ability to effectively intervene on nature’s behalf.

The affirmative’s apocalyptic framing of climate change results in denial and despair,
undermining the political will to act.
Foust 08, Christina R. Foust, Assistant Professor in the Department of Human Communication Studies
at the University of Denver, ET AL., with William O. Murphy, Doctoral Student and Graduate Teaching
Instructor in the Department of Human Communication Studies at the University of Denver, and Chelsea
Stow, Doctoral Student and Graduate Teaching Instructor in the Department of Human Communication
Studies at the University of Denver, 2008 [“Global Warming and Apocalyptic Rhetoric: A Critical Frame
Analysis of US Popular and Elite Press Coverage from 1997-2007,” Paper Submitted to the Environmental
Communication Division of the National Communication Association Convention in San Diego,
November 20th, p. 22-23, Available Online at http://www.allacademic.com/meta/p260125_index.html,
Accessed 03-18-2009]
Elements of an apocalyptic frame could be said to exist in most of the articles we read, though all elements were not present in each article.
Nonetheless, apocalyptic framing should give us pause, for it threatens to hinder progress in forming a
political will to change the carbon-based energy economy (and thus mitigate the consequences of global warming). To
announce the coming of the apocalypse creates despair as people feel they cannot stop such an event ,
but can only hope that they are among the chosen few to be saved (if they believe in the immanence of the end). Apocalyptic framing
also creates denial, as when people fail to exit the movie theater because they have heard fire yelled
once too often. There may also be a sense of denial in terms of the effectiveness of solutions: Why make
changes to our lifestyle, if the world is going to end [end page 22] quickly and our actions don’t make a difference anyway? If
the end is, indeed, the total destruction of earth, won’t our efforts to make change now be in vain? As Brummett suggests of pre-millennial
apocalyptic rhetoric (which assumes that the world will be destroyed after a judgment day), the cosmically mandated telos of catastrophe
overshadows any efforts to change the trajectory of the narrative. The
only place for human agency within such rhetoric is
the capacity to agree with prophesies, against the polarized opposition of non-believers . By agreeing with the
prophesies, “believers” feel a sense of control over the situation because they are “right,” not necessarily because they are taking collective and
personal steps to resolve the issue.

Crisis discourse causes conservative backlash – makes their politics indefensible and
labeled as hyperbole
Buell 3 (Frederick, cultural critic on the environmental crisis and a Professor of English at Queens
College and the author of five books, From Apocalypse To Way of Life, pages 2-4)
Something happened to strip environmental crisis of what seemed in the 1970s to be its self-evident inevitability. Something happened to allow
environmentalism’s antagonists to stigmatize its erstwhile stewards as unstable alarmists and bad-faith
prophets—and to call their warnings at best hysterical, at worst crafted lies. Indeed, something happened to allow
some even to question (without appearing ridiculous) the apparently commonsensical assumption that
environmentalists were the environment’s best stewards . The most important explanation for these events isn’t hard to
find. In reaction to the decade of crisis, a strong and enormously successful anti-environmental disinformation
industry sprang up. It was so successful that it helped midwife a new phase in the history of U.S. environmental politics, one in which an
abundance of environmental concern was nearly blocked by an equal abundance of antienvironmental contestation. Prophets rushing into the
public space bearing environmental warnings like lanterns held high found themselves suddenly in a very crowded square, one now jammed
with antienvironmental spokespeople also waving lanterns. If formerly too little information had hampered environmental activism, now too
much information achieved the same end. According to Samuel Hays, who carefully chronicled American environmental politics between 1955
and 1985, the public drive for environmental change had been “neutralized ” by the 1980s, blocked by an increasingly
organized and elaborate corporate and conservative opposition.1 Despite scientific evidence and even, in a number of
cases, virtual scientific consensus to the contrary, issue after issue was contested. The ozone hole was denied
and trivialized, food and population crises were debunked , and global warming was hotly denied, doubted, and
dismissed as unproven. Even the most sacred of environmental cows was vigorously attacked: voices were even raised in defense
of DDT, arguing that Carson- inspired hysteria eliminated a chemical essential to preserving public health from diseases like malaria.
Environmentalists, in turn, were stigmatized as extremists. Even to mention environmental crisis meant
being called “Chicken Little ,” or “doomster” or “doomsayer.” Ronald Bailey, in his book Ecoscam: The False Prophets of Eco-logical
Apocalypse, went for this particular jugular with even more ferocity than did his predecessor and model, Julian Simon. Bailey put a different
spin from Thomas Disch’s on the filiation of environmental crisis from fear of nuclear apocalypse. “Modern ecological millenarians, impatient
with waiting for the flash of thermonuclear doom, now claim there is a ‘global environmental crisis’ threatening not just humanity, but all life
on earth.”2 Anything but respectable scientists or responsible citizens, environmentalists were both
pathological fanatics (they were contemporary millenarians) and ill-motivated manipulators of the innocent public .
Bailey thus coined a new term for them. They were “apocalypse abusers”—a disreputable group that presumably used and
misused apocalypse like others did alcohol or dangerous drugs. Environmentalists were also entirely wrong, Bailey asserted. Along with making
ad hominem attacks, he proceeded to marshal supposedly scientific evidence to show how wrong environmentalists were. In doing so, Bailey
joined a large and surprisingly well-organized movement of such writers. For, along with sneering at environmentalists, crisis debunkers
began the “counterscience” movement—a movement devoted to countering the findings of
environmental science with the creation of a body of antienvironmental science . It grew so galling and influential
that one of its targets, Paul Ehrlich, in collaboration with his wife Anne, sought to answer it in a book significantly entitled The Betrayal of
Science and Reason; and environmental scientists and organizations generally recognized that they had to grow adept at quickly refuting
disinformation as well as at researching issues and uncovering new information

Apocalyptic environmental rhetoric causes eco-authoritarianism and political apathy


Buell 3 (Frederick, cultural critic on the environmental crisis and a Professor of English at Queens
College and the author of five books, From Apocalypse To Way of Life, pages 185-186)

Looked at critically, then, crisisdiscourse thus suffers from a number of liabilities. First, it seems to have become a political
liability almost as much as an asset. It calls up a fierce and effective opposition with its predictions; worse, its more specific
predictions are all too vulnerable to refutation by events. It also exposes environmentalists to being called grim
doomsters and antilife Puritan extremists. Further, concern with crisis has all too often tempted people to try to find a “total solution” to
the problems involved— a phrase that, as an astute analyst of the limitations of crisis discourse, John Barry, puts it, is all too reminiscent of the
Third Reich’s infamous “final solution.”55 A total crisis of society—environmental crisis at its gravest—threatens to translate
despair into inhumanist authoritarianism; more often, however, it helps keep merely dysfunctional authority in place. It thus leads,
Barry suggests, to the belief that only elite- and expert-led solutions are possible.56 At the same time it depoliticizes people,
inducing them to accept their impotence as individuals; this is something that has made many people
today feel, ironically and/or passively, that since it makes no difference at all what any individual does on his or
her own, one might as well go along with it. Yet another pitfall for the full and sustained elaboration of environmental crisis is, though least
discussed, perhaps the most deeply ironic. A problem with deep cultural and psychological as well as social effects, it is
embodied in a startlingly simple proposition: the worse one feels environmental crisis is , the more one is tempted to
turn one’s back on the environment. This means, preeminently, turning one’s back on “nature”—on traditions of nature
feeling, traditions of knowledge about nature (ones that range from organic farming techniques to the different departments of ecological
science), and traditions of nature-based activism. If nature is thoroughly wrecked these days, people need to delink from nature and live in
postnature—a conclusion that, as the next chapter shows, many in U.S. society drew at the end of the millenium. Explorations of how
deeply “nature” has been wounded and how intensely vulnerable to and dependent on human actions it is can thus lead,
ironically, to further indifference to nature-based environmental issues, not greater concern with them. But
what quickly becomes evident to any reflective consideration of the difficulties of crisis discourse is that all of these liabilities are in fact
bound tightly up with one specific notion of environmental crisis—with 1960s- and 1970s-style environmental
apocalypticism. Excessive concern about them does not recognize that crisis discourse as a whole has significantly changed since the
1970s. They remain inducements to look away from serious reflection on environmental crisis only if one does not explore how environmental
crisis has turned of late from apocalypse to dwelling place. The apocalyptic mode had a number of prominent features: it was preoccupied
with running out and running into walls; with scarcity and with the imminent rupture of limits; with actions that promised and temporally
predicted imminent total meltdown; and with (often, though not always) the need for immediate “total solution.” Thus doomsterism
was its reigning mode; eco-authoritarianism was a grave temptation; and as crisis was elaborated to
show more and more severe deformations of nature, temptation increased to refute it, or give up, or even cut off ties to
clearly terminal “nature.”
Link – North Korea
The aff's rhetoric of the North Korea nuclear threat serves to reinforces self-referential
cycles of escalatory action - the way that we talk about North Korea structures our
relations with them
Suh 6 (JJ, Assistant Professor—Cornell University, “Producing Security out of Uncertainty: The North
Korean Nuclear Crisis,” Mario Einaudi Center for International Studies)

One characteristic of the U.S. and the North’s perception of each other is a kind of generalized identity
language, where the other side is sharply ‘othered’, and where stark (often ‘orientalizing’ and
‘occidentalizing’) differences between the way the ‘North Koreans’ think and the way the ‘U.S.’ thinks are
presented as truths about each side .25 They routinely resort to overly generalized characterizations that
pit one’s own peacefulness against the other side’s aggressive nature . This convergence in rhetorical type and
polarization in rhetorical content reproduces the antagonistic interdependence of hostile identities . As it
moves to extreme levels on both sides, it also has a more important impact on framing definitions of interest and threat in an antagonistic and
irreconcilable manner. As it moves to extremes, the leaderships in both countries are more likely to interpret
the other’s intentions in an alarmist way , to give up searching for cooperative solutions to conflicts of interest, and to prefer a
“stick” as a reaction to the other side. 3When Americans look at North Korea, they see a dangerous state whose
nuclear arming is inevitable and nuclear disarming out of the question . When they think of North Korea, they think of
gulags, one-man rule and dogmatic devotion to the ideology of Juche. They point out that North Korea started the Korean War in 1950, and
that North Korea has perpetrated a number of terrorist attacks including commando style operations inside the South and the 1983 Rangoon
bombing that killed 17 members of the South Korean President’s entourage. According to some, the
North’s regime can even be
suicidal – i.e., out of desperation, it may lash out against the South even if it knows that means its total
destruction – making it all the more unpredictable and dangerous. They mobilize different evidence and
offer various reasons but most in the United States agree that North Korea is a dangerous revisionist
state that must therefore be guarded against. When American policy makers formulate policies, they act on this kind of
understandings of North Korea’s identity.26 Within such a frame of mind, evident in the “axis of evil” speech and the “outposts of tyranny”
comments, North Korea is naturally seen determined to acquire nuclear weapons. President Bush’s speech writer and a defense policy
advisor hence state
without qualifications: “the North Korean leadership craves a nuclear arsenal even more
desperately than it hungers for international approval or American aid.”27 Others, sharing the same mind-set,
believe that North Korea’s nuclear intention is so entrenched that it is unalterable : Pyongyang pushes its nuclear
weapons project overtly when it can and covertly when it must. With the right enticements, furthermore, Pyongyang can be convinced to
promise to give up its nuke program: it just can't be convinced to give up the program itself.28 According
to this view, the history
of nuclear diplomacy is a story of the South’s and U.S. genuine efforts and the North’s deceptions.
Following is a typical account of nuclear diplomacy with the North, which draws on the presumed perception of the North’s identity. In the
early 1990s, South Korea's President Roh Tae Woo managed to hammer out the North-South Joint 14 Declaration on the Denuclearization of
the Korean Peninsula, only to find out that Pyongyang was cheating on that agreement. Then the United States signed the 1994 Agreed
Framework, only to be cheated on by the North again. When that Framework began to wobble because of North Korean nuclear cheating, the
Clinton Administration tried to stop North Korea's nuclear intentions through the Perry Process. Despite the Perry Process and the Sunshine
Policy, Pyongyang was caught once again cheating on its nuclear deals in 2002. Instead of scrapping its weapons programs, North Korea
admitted the violation, declared the Agreed Framework dead, and forged ahead with its nuclear weapons program. Such a storyline reflects a
truncated understanding of dynamic interactions. At the same time, it is predicated upon, and reproduces, a belief about the North’s identity as
inherently untrustworthy and dangerous. Thereis indeed a positive feedback loop between Washington’s
truncated understanding and its identity perception of the North as the two reproduce and reinforce
each other. Furthermore, the positive feedback loop, in which Washington is caught, is buttressed by another feedback loop: one that
Pyongyang is caught in about the U.S. identity. The two loops are mutually interdependent and reinforcing because one is the mirror image of
the other. This mechanism is well illustrated by the events following the 2002 state of the union address. In the address, President Bush
restored the North’s identity as the “evil” that must be terminated, thereby effectively putting to an end the virtuous cycle that had begun
under his predecessor’s administration. Bush’s name-calling was angrily returned by Pyongyang, which called the U.S. the “empire of devil.”
Here itis important to recognize that no obvious changes in material power condition triggered such
vitriolic verbal exchanges. It is a change in actors’ understanding that started the vicious cycle. After 9/11,
the Bush administration brought to American decision making its Manichean understanding of the world where the North, together with Iraq
and Iran, constituted the axis of evil that proliferates the danger of terrorists, missiles and nuclear weapons. Not
only did the verbal
exchanges reflect the new understanding of actors’ 15 identities but they also provided the ideational
context within which Washington and Pyongyang took actions that restored the vicious cycle. Because the
former could not trust the “intransigent North” to honor its end of the Geneva Framework Agreement, it halted the shipment of heavy oil;
because the latter feared the “aggressive Washington,” it restarted its nuclear programs. And one’s action does not only
necessitate but also justifies the other’s reaction; such an action- reaction chain reinforces one’s identity
perception. Within the restored Hobbesian culture, the other was always a suspect that compelled one to take preventive defensive
measures. Within the enmity, such defensive steps were perceived as a renege, an aggression, or a preparation for an attack. Now the
vicious cycle of insecurity was restored with vengeance. The social structure of enmity serves as a
perceptual prism that privileges an interpretation of facts and uncertainties about the North's nuclear
program as the proliferation dangers. It discounts the possibility that the North, as well as the U.S., is caught in a security
dilemma; it amplifies the possibility that the North lies, cheats, and deceives. It skews the discursive space in such a way as to
dismiss as a deception Pyongyang’s offer to negotiate while accepting its rhetoric of war as true. Its denial
of a weapons program is a lie; its admission a truth. The enmity constitutes the ideational context within which realists have it both ways.
What is striking about the nuclear crisis is the ease with which many of the uncertainties and facts about
the North's nuclear program were assumed away or disregarded. Alternative interpretations were
quickly brushed aside as the threat of the “North’s nuclear weapons” established itself as the reality. In
the beginning years of the 21st century when North Korea's conventional military threat was increasingly coming into question, the reality of
“missile threat” and “nuclear threat” added another layer to the master narrative, the “dangerous North.” Because that master narrative had
existed ever since Korea’s division in 1945, it was all the easier to return to the familiar reality. The relative ease with which the vicious cycle
was brought back and the relative 16 difficulty that the virtuous cycle faced provide evidence that the discursive playing field was skewed in
favor of the North’s dangerous identity. If the U.S. perception of the North as the danger had been heavily influenced by the global Cold War
enmity, since the 1990s the reality of the “dangerous North” was reproduced through the articulation of “nuclear threat” and “missile threat .”
U.S. practices produced a constant articulation of danger to the national security of the allies. The
“danger,” through these practices, has become part of the social reality within which the U.S.
understands the North and the world.

The aff is a hypocritical demonization of North Korean nuclear weapons – this


conception of North Korea prevents effective solutions and is a violent way of viewing
the world
Lal 9, Lal, activist, MA in IR, tenure-track professor in IR 2k9 (Prerna, “North Korea Is Not a Threat – Unveiling Hegemonic Discourses”
http://prernalal.com/2009/04/north-korea-is-not-a-threat-unveiling-hegemonic-discourses/)

That security is socially constructed does not mean that there are not to be found real, material
conditions that help to create particular interpretations of threats, or that such conditions are irrelevant
to either the creation or undermining of the assumptions underlying security policy. Enemies , in part,
“create” each other, via the projections of their worst fears onto the other; in this respect, their
relationship is intersubjective. To the extent that they act on these projections, threats to each other
acquire a material character. -Ronnie Lipschutz, UCSC Kim Jong-Il wants attention. And now he has it. He won’t go in our ‘Morons of
the Week’ column and certainly scores points for knowing how to misuse national resources to get international attention. Our problem
with MSM coverage of the North Korea ‘missile threat’ is with the purported hegemonic discourse.
Hegemonic discourse does not pertain to just speech; it refers to whole narratives, with a hero and a
villain, and us and them that we must defeat and overcome. The point of hegemonic discourse–in this
case the discourse of the United States on demonizing North Korea and drawing attention to its nuclear
activities—is to subjugate and oppress the counter-discourses of a race-war, nuclearism and anti-
capitalism. (1) Race war discourse While this is not a clash of civilizations, it is certainly a race war in that
the entire discourse revolves around preventing certain kinds of people from acquiring and using
nuclear weapons. Would the United States use the same tactics in France? Or even India ? No, in fact it looked
the other way on outrageous French nuclear testing in the Pacific and supports India’s nuclear program despite the fact that it is not a signatory
of the NPT! Ronnie Lipschutz has some fine lines for us in On Security: To be sure, the United States and Russia do not launch missiles against
each other because both know the result would be annihilation. But the same is true for France and Britain, or China and Israel. It was the
existence of the Other that gave deterrence its power; it is the disappearance of the Other that has vanquished that power. Where Russia is
now concerned, we are, paradoxically, not secure, because we see no need to be secured. In other words, as Ole Waever might put it, where
there is no constructed threat, there is no security problem. France is fully capable of doing great
damage to the United States, but that capability has no meaning in terms of U.S. security . On the other hand,
see the Iran nuclear ‘crisis’ as an example. The United States has demonized Ahmadinejad at every opportunity and conjured him up as an
Islamic fundamentalist and nationalist who will defy non-proliferation at all costs. On the other hand, Ahmadinejad cheekily asked the United
States to join the rest of civilization in worshipping God. That is the discourse of race war but it is concealed by juridical
discourse—the hegemonic discourse. To borrow from Michael Foucault, the United States is using the
juridical schema of nuclear non-proliferation to conceal the war-repression schema. North Korea is the
historical Other, the terrorist, the threat against whom the world must be protected in the juridical
schema. Yet, under the war-repression schema, North Korea is a sovereign nation with the right to develop nuclear and communications
technology. And this latest action is really nothing more than a plea for economic help. (2) Nuclearism discourse Tied to the race war
schema, is the discourse of nuclearism, which refers to the ideology that nuclear weapons are instruments of peace.
Nukespeak in the form of MAD or the hype over so-called precision weapons by our leaders has had trickle-down effects to the point of
achieving a mental-wipe or historical amnesia of the U.S. nuclear attacks on Hiroshima and Nagasaki. This
discourse effectively
represents a war on history and subjugation of knowledges about the horrors of nuclear war and fallout.
Closely related to nuclearism is the issue of whiteness around nuclear weapons, the paternalistic
presupposition that Western powers are the responsible and rightful leaders on the issue, the racist
ideology that nuclear weapons in the hands of an Islamic country or “terrorist” spells end to world peace
or catastrophe while it is perfectly alright for France, Britain, the United States, Russia, China and now India, to
have nuclear weapons. The epistemological assumptions of nuclearism are dangerous, besides being
racist and morally repulsive. The formation of a “nuclear club” and an exclusive right to possess nuclear
weapons makes them a forbidden fruit and an issue of prestige, thereby encouraging proliferation . Indeed,
discourse around the North Korea and Iran nuclear buildup denotes that these countries see a successful completion of the fuel cycle or the
launching of a rocket as an issue of great prestige. There is absolutely nothing prestigious about owning weapons of mass destruction, weapons
that can end civilization. However, countries like North Korea and Iran can be forgiven for their nuclearist mentality; after all, it is an implication
of the discourse that has been perpetuated by the West, a discourse that has become common knowledge and culture. Nuclearism must be
addressed and put on the table to move past the current impasse over nuclear negotiations and the non-proliferation regime . Without
denouncing nuclear weapons and facing our moral conscience as the only nation to have ever used
nuclear weapons, we cannot hope to avert nuclear proliferation and prevent ‘rogue states’ from going
that route. (3) Anti-Capitalism Discourse Truth be told, much of the world is suffering from the dire effects of an international economic
system that does not benefit them. All the signs of desperation are present. They come from the rallies andburning of effigies around the world.
The violent protests against NATO and the G-20 summit. The high prices of food. They come as small requests from students on whether
anyone is listening. And even the scapegoating of the Other (be it gays, Muslims, liberals, undocumented immigrants) is really an ignorant
response to our unwanted troubles, thoughts and desires. The
problem is not North Korea or Kim Jung II. The problem is
an international system of haves and have-nots, where people without institutional power vie for
attention. In this scenario, a nuclear missile from an impoverished, wretched country helps garner more
attention than protests, rallies and suicide. How else can North Korea hope to get the help that it
desperately needs? Foolcracy is hits the nail on what might happen next: What else of those “consequences” besides the expected veto
of proposed UN sanctions? It probably means that a deal will be made with North Korea for food and other
essentials. In return, North Korea will “give up” part of its nuclear or rocket program and…then, in a couple of
years, they will go back to the same game of spitting in the face of the world in exchange for food and other essentials. In other words, its a bit
like a dysfunctional family that likes to play with guns. The Obama Administration has scrambled to battle anti-Americanism with new
euphemisms. It is not the ‘global war on terror’ but a ‘global contingency operation.’ Not likely to catch on anytime soon. The people living in
dire states and conditions, ravaged by war, poverty and hardship, know precisely what it is–an attack on their existence predicated by the
United States and its allies.
We have seen and read the master narrative before of demonizing a country,
bringing about regime change and killing, colonizing and repressing more peoples while doing it. By
unearthing these counter-discourses, we can hope to move towards a ‘solution’ to the North Korea
issue. Again, the ‘problem-solution’ is not the missiles, but the manner in which North Korea is seeking
help and attention. Finding common ground requires discovering and deconstructing the cultural and
discursive constructs. However, the window of opportunity is quite small, as seen by positions and
interests of the parties involved. I don’t doubt though, that North Korea will cease to be an entity
sometime in the near future and become into Korea again.

Cold War conceptions of Asia aren't true anymore – disproving the myth of an
apocalyptic North Korea is key
Kang 03. David (Professor of International Relations and Business, Director of Korean Studies Institute), Getting Asia Wrong: The Need for
New Analytical Frameworks International Security, Volume 27, Number 4, Spring 2003, pp. 57-85 MUSE

Following the end of the Cold War in 1991, some scholars in the West began to predict that Asia was
“ripe for rivalry.”12 They based this prediction on the following factors: wide disparities in the levels of
economic and military power among nations in the region; their different political systems, ranging from
democratic to totalitarian; historical animosities; and the lack of international institutions . Many scholars thus
envisaged a return of power politics after de- cades when conºict in Asia was dominated by the Cold War tension between the United States
and the Soviet Union. In addition, scholars envisaged a re- turn of arms racing and the possibility of major conflict
among Asian countries, almost all of which had rapidly changing internal and external environments.
More specific predictions included the growing possibility of Japanese rearmament;13 increased Chinese adventurism spurred by
China’s rising power and ostensibly revisionist intentions;14 conºict or war over the status of Taiwan;15 terrorist or missile attacks
from a rogue North Korea against South Korea, Japan, or even the United States;16 and arms racing or
even conflict in Southeast Asia, prompted in part by unresolved territorial disputes.17 More than a
dozen years have passed since the end of the Cold War, yet none of these pessimistic predictions have
come to pass. Indeed there has not been a major war in Asia since the 1978–79 Vietnam-Cambodia-China conºict; and with only a few
exceptions (North Korea and Taiwan), Asian countries do not fear for their survival. Japan, though powerful, has not rearmed to the ex- tent it
could. China seems no more revisionist or adventurous now than it was before the end of the Cold War. And no Asian country appears to be
balancing against China. In contrast to the period 1950–80, the past two decades have witnessed enduring regional stability and minimal
Social scientists can learn as much from
conºict. Scholars should directly confront these anomalies, rather than dismissing them.
events that do not occur as from those that do. The case of Asian security provides an opportunity to
examine the usefulness of accepted international relations paradigms and to determine how the
assumptions underlying these theories can become misspecified. Some scholars have smuggled
ancillary and ad hoc hypotheses about preferences into realist, institutionalist, and constructivist
theories to make them fit various aspects of the Asian cases, including: assumptions about an irrational
North Korean leadership, predictions of an expansionist and revisionist China, and depictions of Japanese foreign policy as
“abnormal.”18 Social science moves forward from the clear statement of a theory, its causal logic, and its
predictions. Just as important, however, is the rigorous assessment of the theory, especially if
predictions flowing from it fail to materialize. Exploring why scholars have misunderstood Asia is both a
fruitful and a necessary theoretical exercise . Two major problems exist with many of the pessimistic predictions about Asia.
First, when confronted with the nonbalancing of Asian states against China, the lack of Japanese rearmament, and five decades of noninvasion
by North Korea, scholars typically respond: Just wait. This reply, however, is intel- lectually ambiguous. Although
it would be unfair
to expect instantaneous national responses to changing international conditions, a dozen years would
seem to be long enough to detect at least some change . Indeed Asian nations have historically shown an ability to respond
quickly to changing circum- stances. The Meiji restoration in Japan in 1868 was a remarkable example of governmental response to European
and American encroachment, and by 1874 Japan had emerged from centuries of isolation to occupy Taiwan.19 More re- cently, with the
introduction of market reforms in late 1978, when Deng Xiaoping famously declared, “To get rich is glorious,” the Chinese have trans- formed
themselves from diehard socialists to exuberant capitalists beginning less than three years after Mao’s death in 1976.20 In the absence of a
speciªc time frame, the “just wait” response is unfalsiªable. Providing a causal logic that explains how and when scholars can expect changes is
an important as- pect of this response, and reasonable scholars will accept that change may not be immediate but may occur over time.
Without such a time frame, however, the “just wait” response is mere rhetorical wordplay designed to avoid trou- bling evidence.

North Korea isn’t a threat. It’s only problem is that it has nothing to offer the west
Gowans 16 (Stephen Gowans, writer at Global Research, “Why UN Sanctions Against ‘North Korea Are
Wrong. ‘The Americanization of the Korean Peninsula’”, http://www.globalresearch.ca/why-un-
sanctions-against-north-korea-are-wrong-the-americanization-of-the-korean-peninsula/5512760)

The pretext for singling North Korea out for sanction is that it is a threat, but this, like the claim that Saddam
Hussein had concealed WMD in defiance of a UNSC resolution, is pure eye-wash. It has no truth-value, only value as
propaganda for justifying continuing US aggression against a country that refuses to give up public
ownership and economic planning or surrender its political and economic sovereignty to the United
States. In his February 23, 2016 testimony before the Senate Committee on Armed Services, the
Commander of the US Pacific Command Harry B. Harris Jr. said that “North Korea is not an
existential threat to the United States.” [46] US establishment journalist David E. Sanger, a member of the Council on Foreign
Relations, a Wall Street think tank for the US State Department, explained that neither are North Korea’s nuclear weapons a threat to South
Korea or Japan, “because North Korean officials know their government would be decimated in minutes or
hours” if they attacked either of these two US allies. [47] As to the threat posed by North Korea’s conventional forces,
Korea specialist Tim Beal points out that, The available evidence shows that North Korea is in most respects
much weaker militarily than the South, and the balance between the two shifts hugely in the South’s favor in the crucial aspect
of advanced technology equipment. But a limited comparison of North and South is really meaningless because this is essentially a question of
North Korea versus the United States – an attack by North Korea on the South would inevitably be a declaration of war against the United
States. The U.S. has “operational command” of the South Korean military in the event of war, there are 28,500 U.S. military personnel (and
considerably more civilians) stationed there and there is the over-riding geopolitical imperative – the U.S. would not tolerate the establishment
North Korea cannot use nuclear weapons in an
of an independent Korea by force. What is certain, however, is that
offensive manner because the retaliation would be overwhelming. One cannot use a handful of
nuclear weapons, of uncertain efficacy and with unproven delivery systems, against an adversary
with thousands of nuclear weapons and well-tested delivery systems. North Korean cannot
effectively threaten the United States or indeed South Korea (because of the U.S. nuclear umbrella)
with nuclear weapons. [48] The relationship of the United States to North Korea is a complex and multi-dimensional one. Wall Street-
dominated Washington sees the DPRK as offering nothing in the way of profit-making opportunities to please U.S. investors, and hence, has no
motivation to accept the North Korean status quo. This explains why for decades the United States has maintained sanctions on the DPRK for
the reason that it has “a Marxist-Leninist economy.” David Straub, director of the State Department’s Korea desk from 2002 to 2004 explained
that “U.S. administrations have never considered and will never consider establishing a strategic relationship with the DPRK. North Korea’s
[49] Presenting North
closed economic and social system means the country has virtually nothing of value to offer the United States.”
Korea as a threat allows the US military-industrial complex to justify massive defense spending and
to reap huge profits from US taxpayers through a fraud at whose center reposes the myth of the
North Korean threat. Colin Powell, as the United States’ top soldier, once infamously remarked that after the demise of the Soviet
Union he was down to only a few demons, Castro and (North Korea’s) Kim Il-Sung. [50] Portraying North Korea as belligerent,
provocative and threatening justifies the United States’ continued military presence on the Korean
peninsula, where, as Tim Beal observes, “China, Japan, Russia and the United States meet and
contest and as such is the most strategically valuable place on earth.” [51]
Their descriptions of North Korea create militarism and makes cooperation impossible
– turns case
Bleiker 2003 (Roland, Ph.D in international relations from the Australian National University, currently
is a Professor of international relations at the University of Queensland. “A rogue is a rogue is a rogue:
US foreign policy and the Korean nuclear crisis” 2003, InternationalAffairs79, 4 (2003) 719-722, MT)
Dealing with communist North Korea has become one of the most difficult challenges in global politics
today. Totalitarian and reclusive, ideologically isolated and economically ruined, it is the inherent 'other' in a globalized and
neoliberal world order. And yet North Korea goes on surviving, not least because its leaders periodically rely on threats to gain
concessions from the international community. The latest such attempt was signalled in the autumn of 2002, when Pyongyang admitted to
a secret nuclear weapons programme and subsequently withdrew from the Nuclear Nonproliferation Treaty (NPT). Thereafter the
situation rapidly deteriorated. By early 2003 both the US and North Korea were threatening each other
with outright war. Even Japan, adopting its most militaristic posture in decades, publicly contemplated
the possibility of a pre-emptive strike against North Korea .3 The dangers of North Korea's nuclear brinkmanship are
evident. Miscalculation or a sudden escalation of tension could precipitate a human disaster at any
moment. Equally dangerous, although much less evident, are the confrontational and militaristic
attitudes with which some of the key regional and global players, most notably the United States, seek
to contain the situation. The problems associated with these approaches have been largely obscured by
Washington's apparent willingness to de-escalate the crisis through negotiations . 'I believe this is not a military
show-down,' stressed President George W. Bush. 'This is a diplomatic show down.'4 Much has been made of the difference between this
tolerant approach and the far more aggressive stance taken in respect of Iraq, where war was presented as the only possibility of preventing a
dangerous escalationtowardsthe use by the Baghdadregime of weapons of massdestruction. The situation was particularly paradoxical since
Pyongyang publicly admitted to its nuclear ambition and asked inspectors from the International Atomic Energy Agency (IAEA) to leave the
country. Baghdad, by contrast, denied possessing weapons of mass destruction and admitted inspectors for the specific purpose of verifying its
claims.5 The reluctance to use force against North Korea obscures the fact that US foreign policy is guided
by a largely consistent approach towards the phenomenon of so-called 'rogue' states. That war was not
advocated in Korea is a reflection of diplomatic constraints and, above all, strategic limitations. South Korea's then newly
elected president, Roh Moo-hyun, strongly opposed a military solution to the problem. Perhaps even more importantly, the
consequences of an escalation in Korea would be hard to contain. One of the world's biggest cities, Seoul, is only 50
kilometres away from the heavily militarized 'Demilitarized Zone'(DMZ) that separatesNorth Korea from the South. Even if pre-emptive
strikes were to neutralize North Korea's possible nuclear arsenal, they would not be able to destroy all
its conventional weapons. The latter alone could easily trigger a second Korean war, with disastrous consequences on all sides. The
purpose of this article is to examine the role of the United States in the Korean nuclear crisis, for no aspect of the past and present dilemmas on
the peninsula can be addressed or even understood without recourse to the US. This is why China repeatedly stressed that the latest nuclear
crisis was primarily an issue between North Korea and the United States.6 Kim Dae-jung, in his final speech as South Korea's president,
reiterated the same theme: 'more than anything, dialogue between North Korea and the United States is the important key to a solution.'7 A
solution is, however, far from reach. Both
the US and North Korea see the other as a threat . And each has good reasons for
doing so. But each
is also implicated in the production of this threat. The problem is that these interactive
dynamics are hard to see, for the West tends to project a very one-sided image of North Korea -one
that sees it solely as a rogue outlaw, and thus a source of danger and instability. Nicolas Eberstadt, for instance,
stresses that 'North Korean policies and practices have accounted for most of the volatility within the Northeast Asian region since the end of
the Cold War.'8 Very few policy-makers, security analysts and journalists ever make the effort to imagine
how threats are perceived from the North Korean perspective, or consider how these perceptions are
part of an interactive security dilemma in which the West, and US foreign policy in particular, is
implicated as deeply as the vilified regime in Pyongyang. The central argument of this article is that the image of
North Korea as a 'rogue state' severely hinders both an adequate understanding and a possible
resolution of the crisis. The rhetoric of rogue states is indicative of how US foreign policy continues to
be driven by dualistic and militaristic Cold War thinking patterns . The 'Evil Empire' may be gone; not so the
underlying need to define safety and security with reference to an external threat that must be warded off
at any cost. Rogues are among the new threat-images that serve to demarcate the line between good
and evil. As during the Cold War, military means are considered the key tool with which this line is to be defended. In the absence of a
global power that matches the US, this militaristic attitude has, if anything, even intensified. Look at Washington's recent
promulgation of a pre-emptive strike policy against rogue states. The consequences of this posture are
particularly fateful in Korea, for it reinforces half a century of explicit and repeated nuclear threats
against the government in Pyongyang. The impact of these threats has been largely obscured , not least
because the highly technical and specialized discourse of security analysis has enabled the US to
present the strategic situation on the peninsula in a manner that misleadingly attributes responsibility
for the crisis solely to North Korea's actions . A brief disclaimer is in order at this point. I offer neither a comprehensive review
of the Korean security situation nor a detailed analysis of the latest events. As a result, there will be little mention of some admittedly crucial
issues, such as the role of China or the increasingly problematic rift between Washington and Seoul. Instead, I identify broad patterns of conflict
and embark on a con- ceptual engagement with some of the ensuing dilemmas. Focusing on underlying trends inevitably entails glossing over
nuances at times. For instance, there are heated debates between hawks and doves within Washington's policy circles, and as a result periods
dominated by hard-line realist positions have alternated with periods during which softer and more liberal policies prevailed. But the
persistent pattern of seeing North Korea as a rogue state is far more striking, and in many ways far more
significant, than the strategic policy manoeuvring that takes place within these patterns. Focusing on the
big picture also entails departing from some of the conventions that prevail in the field of strategic and
security studies. Contrary to most treatments of the subject, I do not discuss the technical aspects of nuclear and other weapons, except
to show, as indicated above, how these very discussions,jargon-ridden and inaccessible as they are to any but military experts, often serve to
stifle debate about some of the underly- ing political and ethical issues.9

The move towards securitizing North Korea is not born out of self-protection, but
rather is a manifestation of the US’s Colonial Interest in the Korean Peninsula
Gowans 16 (Stephen Gowans, writer at Global Research, “Why UN Sanctions Against ‘North Korea Are
Wrong. ‘The Americanization of the Korean Peninsula’”, http://www.globalresearch.ca/why-un-
sanctions-against-north-korea-are-wrong-the-americanization-of-the-korean-peninsula/5512760)

In 1993, the US Strategic Command announced that it was retargeting some of its strategic nuclear
weapons away from the former Soviet Union to North Korea. A month later, Pyongyang announced that
it would withdraw from the NPT, signaling that if Washington was going to dangle a nuclear sword of
Damocles over its head, North Korea would take steps to counter the threat. [11] This spurred a series of
negotiations which led Pyongyang to reverse its decision and to remain in the treaty. It eventually made another volte-face, announcing its
intention to exit the treaty following US President George W. Bush’s January 29, 2002 designation of North Korea as part of an Axis of Evil.
Bush’s virtual declaration of war against the DPRK was only the tip of an iceberg of threats Washington had directed at the DPRK as part of its
long running Cold War against the Communist country.
In March 2002, the Los Angeles Times revealed classified
Pentagon information listing seven countries as possible targets of a US nuclear strike. Among the
targets was North Korea. The Pentagon’s nuclear strike list also included Russia, China, Syria, Libya, Iran, and Iraq. [12] North
Korean officials explained their withdrawal from the NPT by pointing to the “Bush administration’s
nuclear attack plan” which “showed that the United States…is pursuing world domination with force of
arms and that the United States is not hesitant in launching a nuclear attack on any nation if it is
regarded as an obstacle to this end.” [13] Echoing these concerns, a North Korean diplomat explained
his country’s decision to exit the NPT and embark on the development of nuclear weapons. The NPT clearly
states that nuclear power states cannot use nuclear weapons for the purpose of threatening or endangering non-nuclear states. So the DPRK
thought that if we joined the NPT, we would be able to get rid of the nuclear threat from the US. Therefore we joined. However, the US never
withdrew its right of pre-emptive nuclear strike. They always said that, once US interests are threatened, they always have the right to use their
The world situation changed again after 11 September 2001.
nuclear weapons for pre-emptive purposes. [14] He added:
After this, Bush said that if the US wants to protect its safety, then it must remove the ‘Axis of Evil’
countries from the earth. The three countries he listed as members of this ‘Axis of Evil’ were Iran,
Iraq and North Korea. Having witnessed what happened in Afghanistan and Iraq, we came to realise
that we couldn’t put a stop to the threat from the US with conventional weapons alone. So we realised
that we needed our own nuclear weapons in order to defend the DPRK and its people. [15] The NPT allows
states to exit the accord if they believe their continued participation in it is injurious to their highest interests. “Each Party shall in exercising its
national sovereignty have the right to withdraw from the Treaty if it decides that extraordinary events, related to the subject matter of this
Treaty, have jeopardized the supreme interests of its country.” Clearly, Washington’s overt hostility, the listing of North Korea as a target of a
possible nuclear strike, and the Bush administration’s virtual declaration of war, constituted “extraordinary events” which jeopardized the
DPRK’s “supreme interests.”
Fear of North Korea is a form of orientalism that prioritizes Western concerns about
safety over nonwestern bodies.
Gowans 16 (Stephen Gowans, writer at Global Research, “Why UN Sanctions Against ‘North Korea Are
Wrong. ‘The Americanization of the Korean Peninsula’”, http://www.globalresearch.ca/why-un-
sanctions-against-north-korea-are-wrong-the-americanization-of-the-korean-peninsula/5512760)

North Korea says it developed nuclear weapons “to protect its sovereignty and vital rights from
the U.S. nuclear threat and hostile policy which have lasted for more than half a century” [16] and which culminated in the Bush
administration’s nuclear saber rattling and threat of war. Compare North Korea’s reasons for having nuclear
weapons with those of Britain, one of the NPT’s nuclear weapon states. The UK government’s 2006 White Paper, “The Future of
the United Kingdom’s Nuclear Deterrent,” states that “The primary responsibility of any government is to ensure
the safety and security of its citizens,” and that “For 50 years (Britain’s) independent nuclear
deterrent has provided the ultimate assurance of (the country’s) national security.” “The UK’s nuclear
weapons,” the document concludes, are designed “to deter and prevent nuclear blackmail and acts of aggression against our vital interests that
cannot be countered by other means.” [17] Russia, also a nuclear weapon state, invokes the same rationale
for maintaining a nuclear arsenal. The country’s president, Vladimir Putin, says Russia needs nuclear arms to preserve its
deterrent and strategic stability in the face of threats. [18] Similarly, Washington’s 2015 National Security Strategy
declares that “the United States must invest the resources necessary to maintain….a safe, secure, and
effective nuclear deterrent that preserves strategic ability.” The rationale of nuclear weapon states for maintaining a
stock of nuclear weapons “applies with even greater force to weak states that may come under threat from stronger ones. The smaller
and weaker the state, the greater the need for nuclear weapons to make potential aggressors think
twice before threatening or invading them.” Pointing specifically to Britain, researcher David Morrison
argues, if “one of the strongest states in this world needs to have nuclear weapons in order to deter
potential aggressors, then no state in the world should be without them, if at all possible .” Morrison caps his
point by speculating that: “Had Iraq succeeded in developing nuclear weapons, the US/UK would not have invaded in March 2003 (and
hundreds of thousands of Iraqis who died as a consequence would still be alive).” [19]

Portrayals of North Korea are hypocritical and only serve to mask the crimes of the
U.S. which only allows for colonialist violence to continue
Gowans 16 (Stephen Gowans, writer at Global Research, “Why UN Sanctions Against ‘North Korea Are
Wrong. ‘The Americanization of the Korean Peninsula’”, http://www.globalresearch.ca/why-un-
sanctions-against-north-korea-are-wrong-the-americanization-of-the-korean-peninsula/5512760)
In declaring that the United States will never allow North Korea to develop nuclear-tipped intercontinental ballistic missiles, Samantha Power
called the DPRK “one of the most brutal regimes the world has ever known.“ Is it? North Korea has a
publicly-owned, planned, economy directed toward satisfying the material needs of its citizens while
preserving its sovereignty. With a history of colonization by Japan and alienated from its compatriots in the south by the United
States’ division of the peninsula, North Korea holds independence as an especially important goal. US troops
have been almost continually present in South Korea since 1945, and the Pentagon retains wartime command of the
South Korean military. By contrast, there are no foreign troops or bases in North Korea, and North Korean
troops have never fought beyond Korean borders, unlike South Korea’s military, which took on a
mercenary role in the Vietnam War, joining the United States in an aggression to suppress the independence struggle of another
people which had suffered colonization, the Indo-Chinese. From 1964 to 1973, approximately 312,000 South Korean troops were deployed to
Vietnam, paid 23 times their base pay by the United States. It is not without justification that North Korea reviles South Korea as a puppet state.
And while South Korea nestles under a US nuclear umbrella, North Korea has never been protected by
the nuclear weapons of another state’s military. The DPRK offers attractions typical of communist
countries: free health care, free education, free housing, and virtually free public transportation. [44] A
pastiche of half-truths and outright distortions circulate in the Western media about North Korea, distinguished only by their contempt for the
intelligence of the public. Events regarded as anodyne in the West are presented in dark and menacing hues when they happen in the DPRK.
This has long been true. Observers of North Korea have for decades complained about deceptions in Western media and discourse about North
Korea, aimed at tarring the country’s reputation rather than illuminating its politics, history and economy. Anna Louise Strong wrote “In days to
Yet there is little public knowledge about the country and most of the
come, Korea will continue to supply headlines.
headlines distort rather than reveal the facts .” [45] That was in 1949. Little has changed. But then, propagandistic treatment of
communist, socialist and economically nationalist states is the accustomed practice of Western media, whose owners’ interests have always
been against states which insist on exercising economic sovereignty in preference to subordination to the profit-making interests of Western
There’s more than a little hypocrisy in Power claiming that the United States
financial and business concerns.
spearheaded a Security Council resolution out of opposition to a “brutal regime,” when Washington
counts the brutal regimes of Saudi Arabia, Egypt, Bahrain, Israel, and Colombia among its favored
satellites, not only sheltering these oppressors and bellicists from sanction, but facilitating their
brutalities. The words Hiroshima, Korea, Vietnam, the Bay of Pigs, Grenada, Panama, Afghanistan, Iraq,
Guantanamo Bay, Abu Ghraib, 100 or more prisoners tortured to death in US detention in the ‘war on
terror’, extra-judicial assassinations by drone strike, to say nothing of the genocide of North America’s
aboriginal people and the brutal slavery of Africans on which the country was founded, make the United
States truly one of the most brutal regimes the world has ever known . It is followed closely by its allies, and fellow
Security Council permanent members, Britain and France, on whose empires the sun never set and blood never dried.

Representations of North Korea are rooted in ideological hegemony not objective data
David Shim, Phd Candidate @ GIGA Institute of Asian Studies, ‘8 [Paper prepared for presentation at the 2008 ISA, Production,
Hegemonization and Contestation of Discursive Hegemony: The Case of the Six-Party Talks in Northeast Asia,
www.allacademic.com/meta/p253290_index.html]

Laclau and Mouffe’s (2001: chapter 2) concept of hegemony, which is used here, rely on a notion developed by Antonio Gramsci (1971).
Gramsci broadened the traditional notion of hegemony beyond the view of mapping hegemony in terms of leadership
and dominance, which are based on material capabilities, by introducing inter-subjective and ideological aspects into
this concept. Accordingly, hegemony contains the ability of a class (bourgeois) to project the world view
over another (workers, peasantry) in terms of the former, so that it is accepted as common sense or reality. His merit was to
conceptualize hegemony in terms of power without the use of force to reach consent by the dominated class through education and, what he
calls, the role of intellectuals (“men of letters”) such as philosophers, journalists and artists (Gramsci 1971: 5-43). The process of fixing meaning,
that is, in terms of Laclau and Mouffe (2001: 105), when an element (sign with unfixed meaning) is transformed through articulation into a
moment (sign with fixed meaning), is hegemonic, since it reduces the range of possibilities and excludes alternative meanings by determining
the ways in which the signs are related to each other. That is to say, when
meaning is fixed, i.e. hegemonized, it
determines, what can be thought, said or done in a meaningful way. 13 Applied to this case, the exclusive
character of a hegemonic discourse makes it unintelligible to make sense of North Korea’s nuclear
program in terms of, for instance, energy needs, because – as it is argued – practices of problematization
hegemonized the ways of thinking, acting and speaking about North Korea . Discursive hegemony can be regarded
as the result of certain practices, in which a particular understanding or interpretation appears to be the natural order of things (Laclau/Mouffe
2001). This
naturalization consolidates a specific idea, which is taken for granted by involved actors and
makes sense of the(ir) world. As Hall (1998: 1055-7) argues, common sense resembles a hegemonic discourse, which is a dominant
interpretation and representation of reality and therefore accepted to be the valid truth and knowledge. Referring to the productive character
of discursive hegemony, the Six-Party Talks can be regarded as an outcome of the dominating interpretation of reality (cf. also Jackson 2005:
20; Cox 1983; Hajer 2005). The
hegemonic discourse regarding North Korea provides the framework for a
specific interpretation in which the words, actions or policies of it are attached with meaning , that is, are
problematized. As Jacob Torfing argues “a
discursive truth regime […] specifies the criteria for judging something
to be true of false”, and further states, that within such a discursive framework the criteria for acknowledging
something as true, right or good are negotiated and defined (Torfing 2005a: 14; 19; cf. also Mills 2004: 14-20). However,
important to note is, if one is able to define this yardstick, not only one is able to define what is right, good or true, but also what kinds of action
are possible. In other words, if
you can mark someone or something with a specific label, then certain kinds of acts
become feasible.14 Basically, it can be stated that discursive hegemony depends on the interpretation and representation by actors of
real events since the interpretation of non-existent facts would not make sense. But the existence of real events does not necessarily have to
be a prerequisite for hegemonizing interpretational and representational practices because actions
do not need to be carried
out, thus, to become a material fact, in order to be interpreted and represented in a certain way (Campbell
1998: 3). Suh Jae-Jung (2004: 155) gives an example of this practice. In 1999 US intelligence agencies indicated to
preparing measures taken by North Korea to test fire a missile. Although the action was not yet
executed, it was treated as a fact, which involved and enabled certain implications and material
consequences such as the public criticism of North Korea, the issuance of statements, diplomatic activity and efforts to
hegemonize and secure this certain kind of reality , i.e. to build a broad majority to confirm this view on North Korea. In
other words, the practices of problematizing North Korea took place even before an action was done.

Their representations are a product of imperialism and media bias


Alexandra Homolar-Riechmann, @ Peace Research Institute Frankfurt & Kings College, ‘9 [Prepared for delivery at the 2009
Annual Meeting of the International Studies Association, “Rebels without a cause: US foreign policy and the concept of rogue states,” p.
allacademic]

The 1993-94 North Korean nuclear crisis had a profound impact on the development of an overarching
rogue states narrative in three key respects. First, with a particular focus on weapons of mass destruction the
terminology began to be officially applied to describe the chief threat to US national interests amidst the
height of the tensions with the DPRK. A study conducted by Paul D. Hoyt (2000) on the rise of the rogue states image in American
foreign policy between 1993 and 1998 demonstrates this. Using the built-in search engines of the White House, the Department of State,
the Department of Defense and CIA Web pages, Hoyt generated a database on the basis of all documents mentioning the term ‘rogue’ from the
Clinton administration. The
results show that the first reference to rogue states was made early in the Clinton administration, in
January 1994. Over time, usage
of the term rogue became considerably more frequent in general, and also
became more widespread across different agencies and departments. In particular, the increased emphasis on rogue
states in Pentagon documents from mid-1994 onwards contributed to a rise of the total numbers of annual
agency mentions of the term (Hoyt 2000: 298-301). However, although North Korea’s nuclear ambitions appear to have catalyzed
the usage of the rogue states terminology, and although WMD capabilities remained the chief concern in the period covered by Hoyt’s study,
the DPRK does not lead the list of rogues mentioned between 1993 and 1998. Instead, Iran and Iraq were mentioned most frequently, followed
by Libya and then North Korea (Hoyt 2000: 302). Second, the North Korean nuclear crisis had a major impact on the development of US policy
options that aimed at countering the threats posed by rogue states armed with weapons of mass destruction and the means to deliver them. In
December 1993, for example, Secretary of Defense Les Aspin announced that the Pentagon had launched a program, the
‘Defense Counterproliferation Initiative’, to counter the perceived increased threat posed by Third World
countries armed with nuclear, chemical, or biological weapons, which included developing a range of new military
capabilities (Abrams 1993). In a Foreign Affairs article in spring 1994, Clinton’s National Security Advisor Anthony Lake (1994: 45-6)
emphasized that the outlaws 16Cuba, North Korea, Iran, Iraq, and Libya are ‘often aggressive and defiant’, which share a ‘siege mentality’ and
have the potential to threaten the international community with advanced weaponry, in particular weapons of mass destruction and missile
delivery systems. Invoking George Kennan’s 1947 strategy, Lake specifically laid out a strategy of ‘dual containment’ toward the ‘backlash’
states of Iran and Iraq as part of the US objective to develop ‘a strategy to neutralize, contain and, through selective pressure, perhaps
eventually transform’ these states (Lake 1994: 46). Applying the language of ‘containment’ to the problem of dealing with rogue states is
significant because, similar to its original application to the Soviet threat, the term containment implies the introduction of a carrot-and-stick
approach to changing rogue state behavior, as well as an ambitious attempt at the ‘restructuring of geopolitical space’. This,
in turn,
further enabled the establishment of an overarching security narrative and conceptual framework
that has influenced how the intentions and capabilities of rogue states are interpreted by US defense
policymakers (Chilton 1996: 134). Both the concept of ‘counterproliferation’ and the ‘dual containment strategy’ subsequently became integral
parts of Clinton’s 1995 National Security Strategy of Engagement and Enlargement (cf. Clinton 1995: 13; 30). By the mid-1990s, the rogue state
terminology had thus become a central element in President Clinton’s approach to US foreign and defense policy (cf. Litwak 2000: 56-70; Klare
1998: 14). Finally, the North Korean nuclear crisis contributed significantly to the proliferation of the rogue states terminology in political
discourse outside the branches of US government. Among other things, in order for actors to take advantage of such catalytic events to
promote particular policy agendas ‘requires planning, organization, publicity and political positioning’ (Parmar 2005: 8). In light of North Korea’s
potential nuclear ambitions and the proliferation of weapons of mass destruction and related technologies, conservative
US think
thanks, such as the Heritage Foundation and the American Enterprise Institute (AEI), began to actively engage in the
proliferation and consolidation of the rogue states terminology. A review of the Heritage Foundation’s online archives
shows that while the term ‘rogue state’ did not feature in its publications during the early post-Cold War era prior to the North Korean nuclear
crisis, the number of articles mentioning rogue states in general and those with a specific focus on North Korea skyrocketed between 1993 and
1994 (www.heritage.org). A similar effect can be observed in US mainstream news media . A search of the Lexis-Nexis
database for ‘major mentions’ of the ‘rogue states’ label in all US news media shows the impact of the
North Korea-US nuclear
crisis on the development of an overarching rogue states narrative. Despite the War in the Persian Gulf against Saddam Hussein’s Iraq, which
later became the ‘model’ rogue state, only two newspaper articles mentioned the term ‘rogue state’ in 1992 – one in relation to human rights
and one with respect to fissile material. In 1993, US news organizations began to report on ‘rogue states’ more frequently: twelve newspaper
articles use the term and seven made an explicit connection between nuclear weapons, missile technology, proliferation and North Korea.
While Iran was counted as a rogue state in five of the 1993 newspaper articles, Iraq was mentioned only twice. One year later, over thirty
newspaper articles mentioned the term ‘rogue state’, many of them now referring to North Korea, along with Iraq and Iran, as ‘the archetypal
rogue state’ (Sigal 1994: 22). By 1996 the ‘rogue state’ concept had become firmly rooted in the US news media vocabulary: the term ‘rogue
state’ was mentioned in 129 newspaper articles and has not fallen under the benchmark of one hundred in any year since. What
is
particularly important is that the conservative US think tank community portrayed North Korea as
immoral, unpredictable, and belligerent so vigorously that opposing views were marginalized and
even deemed questionable in the wider defense policy community and political debates (Smith 2000: 596-8). In the
absence of major alternative sources of information that were independent from US government agencies, both policymakers and
US news media tended to reflect these views, thereby distorting the complexities of North Korea’s
actions, politics, and policies (cf. Bleiker 2003; Smith 2000).
Link – Nuclear War
The modern state utilizes the threat of nuclear weapons to justify invasion in the
interim to prevent catastrophe. Their representations are more likely to lead to
preemption than passivity.
Massumi 07 (Brian, Communication Department of the Université de Montréal , “Potential Politics
and the Primacy of Preemption”)

Fear is always a good reason to go politically conditional. Fear is the palpable action in the present of a threatening
future cause. It acts just as palpably whether the threat is determinate or not. It weakens your resolve, creates stress, lowers
consumer confidence, and may ultimately lead to individual and/or economic paralysis . To avoid the paralysis, which would
make yourself even more of a target and carry the fear to even higher level, you must simply act. In
Bush administration parlance, you "go kinetic."6 You leap into action on a level with the potential that frightens you. You do that, once
again, by inciting the potential to take an actual shape you can respond to. You
trigger a production of what you fear. You
turn the objectively indeterminate cause into an actual effect so you can actually deal with it in
some way. Any time you feel the need to act, then all you have to do is actuate a fear. The production of the effect follows as smoothly
as a reflex. This affective dynamic is still very much in place, independent of Rumsfeld's individual fate. It will remain in place as long as fear
and remains politically actuatable. The
logic of preemption operates on this affective plane, in this proliferative or
ontogenetic way: in a way that contributes to the reflex production of the specific being of the threat. You're
afraid Iraq is a
breeding ground for terrorists? It could have been. If it could have been, it would have been. So go
ahead, make it one. "Bring 'em on," the President said, following Hollywood-trained reflex. He knew it in his "guts." He couldn't
have gone wrong. His reflex was right. Because "now we can all agree" that Iraq is in actual fact a
breeding ground for "terrrorists". That just goes to prove that the potential was always there.
Before, there was doubt in some quarters that Saddam had to be removed from power. Some agreed he had to go, some didn't. Now we
can all agree. It was right to remove him because doing so made Iraq become what it always could have been. And that's the truth. Truth,
in this new world order, is by nature retroactive. Fact grows conditionally in the affective soil of an indeterminately present futurity. It
becomes objective as that present reflexively plays out, as a effect of the preemptive action taken. The reality-based community wastes
time studying empirical reality, the Bushites said: "we create it." And because of that, "we" the preemptors will always be right. We
always will have been right to preempt, because we have objectively produced a recursive truth-
effect for your judicious study. And while you are looking back studying the truth of it, we will have
acted with reflex speed again, effecting a new reality. 7 We will always have had no choice but to
prosecute the "war on terror," ever more vigilantly and ever more intensely on every potential front.
We, preemptors, are the producers of your world . Get used to it. The War in Iraq is a success to the extent that it made
the productivity of the preemptive "war on terror" a self-perpetuating movement. Even if the US were to withdraw from Iraq tomorrow,
the war would have to continue on other fronts no matter who controls Congress or who is in the White House. It would have to continue
in Afghanistan, for example, where the assymetrical tactics perfected in Iraq are now being applied to renew the conflict there. Or in Iran,
which also always could have/would have been a terrorist breeding ground. Or it could morph and move to the Mexican-US border, itself
morphed into a distributed frontline proliferating throughout the territory in the moving form of "illegal immigration". On
the
indefinite Homeland Security front of a protieform war, who knows what threats may be
spinelessly incubating where, abetted by those who lack the "backbone" to go kinetic. Preemption is
like deterrence in that it combines a proprietary epistemology with a unique ontology in such a way
as to make present a future cause that sets a self-perpetuating movement into operation . Its
differences from deterrence hinge on its taking objectively indeterminate or potential threat as its self-constitutive cause rather than fully
formed and specified threat. It situates itself on the ground of ontogenetic potential. There, rather than deterring the feared effect, it
actualizes the potential in a shape to which it hopes it can respond. It assumes a proliferation of potential threats, and mirrors that capacity
in its own operation. It becomes proliferative. It
assumes the objective imbalance of a far-from-equilibrium state
as a permanent condition. Rather than trying to right the imbalance, it seizes it as an opportunity for
itself. Preemption also sets a race in motion . But this is a race run on the edge of chaos. It is a race of movement-flushing,
detection, perception, and affective actuation, run in irreparably chaotic or quasi-chaotic conditions. The race of preemption has any
number of laps, each ending in the actual effecting of a threat. Each actualization of a threat triggers the next lap, as a continuation of the
first in the same direction, or in another way in a different field. Deterrence revolved around an objective cause. Preemption revolves
around a proliferative effect. Both are operative logics. The operative logic of deterrence, however, remained causal even as it displaced its
cause's effect. Preemption is an effective operative logic rather than a causal operative logic. Since its
ground is potential, there is no actual cause for it to organize itself around. It compensates for the
absence of an actual cause by producing an actual effect in its place . This it makes the motor of its
movement: it converts an absent or virtual cause really, directly into a taking-actual-effect . It does this
affectively. It uses affect to effectively trigger a virtual causality.8 Preemption is when the futurity of unspecified threat
is affectively held in the present in a perpetual state of potential emergence(y) so that a movement
of actualization may be triggered that is not only self-propelling but also effectively, indefinitely,
ontologically productive, because it works from a virtual cause whose potential no single actualization exhausts. Preemption's
operational parameters mean that is never univocal. It operates in the element of vagueness and objective uncertainty. Due to its
proliferative nature, it cannot be monolithic. Its logic cannot close in around its self-causing as the logic deterrence does. It includes an
essential openness in its productive logic.9 It incites its adversary to take emergent form. It then strives to become as proteiform as its
ever-emergent adversary can be. It is as shape-shifting as it is self-driving. It infiltrates across boundaries, sweeping up existing formations
in its own transversal movement. Faced with gravity-bound formations too inertial for it to sweep up and carry off with its own operative
logic, it contents itself with opening windows of opportunity to pass through. This is the case with the domestic legal and juridical structure
in the US. It can't sweep that away. But it can build into that structure escape holes for itself. These take the form of formal provisions
vastly expanding the power of the executive, in the person of the president in his role as commander-in-chief, to declare states of
exception which suspend the normal legal course in order to enable a continued flow of preemptive action.10 Preemption
stands
for conflict unlimited: the potential for peace amended to become a perpetual state of undeclared
war. This is the "permanent state of emergency" so presciently described by Walter Benjamin. In current Bush
administration parlance, it has come to be called "Long War" replacing the Cold War: a preemptive war
with an in-built tendency to be never-ending. Deterrence produced asymmetrical conflict as a by-product. The MADly
balanced East-West bipolarity spun off a North-South sub-polarity. This was less a polarity than an axis of imbalance. The "South" was
neither a second Western First nor another Eastern Second. It was an anomalous Third. In this chaotic " Third World ," local conflicts
prefiguring the present "imbalance of terror" proliferated. The phrase "the war on terror" was in fact first popularized by Richard Nixon in
1972 in response to the attack at the Munich Olympics when the Israeli-Palestinian conflict spectacularly overspilled northward.
Asymmetrical conflicts, however, were perceivable by the reigning logic of deterrence only as a reflection of itself. The dynamic of
deterrence were overlaid upon them. Their heterogeneity was overcoded by the familiar US-Soviet duality. Globally such conflicts figured
only as opportunities to reproduce the worldwide balance of terror on a reduced scale. The strategy of "containment" adopted toward
them was for the two sides in the dominant dyad to operate in each local theater through proxies in such a way that their influence, on the
whole, balanced out. "I decided," Nixon said after Munich , "that we must maintain a balance."11 He did not, as Bush did after 9-11, decide
to skew things by going unilaterally "kinetic." The rhetoric of the "war on terror" fell into abeyance during the remainder of the 1970s, as
Southern asymmetries tended to be overcoded as global rebalancings, and going kinetic was "contained" to the status of local anomaly.

Representations of future nuclear war rest on racist fears of irrational non-whites—


the bomb is the epitome of the destructive capacity of Whiteness, naturalizing
structural violence through the projection of a spectacular extinction.
Williams 11 [Paul, lecturer in English at the University of Exeter, “Race, Ethnicity, and Nuclear War”, Liverpool Science Fiction Texts and
Studies, 2011, p.1-3]

In this study, nuclear representations are defined as depictions of the following subjects: (1) the invention and use of the first atomic bombs; (2)
nuclear weapons testing stockpiling of the Cold War superpowers; and (3) nuclear war (often referred to as World War Three) and

life after such a cataclysm. Nuclear technology has been the subject of narratives of racial and national
belonging and exclusion undoubtedly because its emergence (and deployment against Japan) was read
by some commentators as an act of genocidal racist violence, and by some as the apex of Western
civilization’s scientific achievement. These opposing perspectives are interpretative poles that have been central to nuclear representations. By
posing white moral and technological superiority against the destructive technology it supposedly invented, cultural producers have cited nuclear weapons as
evidence against white Anglo-Saxon supremacism. From this point of view, the scientific achievement of splitting the atom does not reveal white superiority;
instead, the enormity of nuclear weapons reminds one that the technology first created by the white world
imperils the whole Earth. Through a range of media, from novels to poetry, short stories to film, comics to oratory, the terms that modern
European imperialism depended upon – ‘civilization, ‘race’, and ‘nation’, in particular – often recur in
nuclear representations. Some of these representations, emerging when Europe’s empires were relinquishing direct control of their colonies, share
the uncertainty that beset the colonial powers following the uneven and often violent decolonizing preocess. The historical congruence of

nuclear representations and decolonization intimates the importance of this context to future visions of
World war Three: tropes of genocide, technological and and scientific modernity, and the (re)population
of the planet are relevant to this apocalyptic subgenre of SF as well as being recurrent elements in colonial history.
Several of the nuclear representations discussed reproduce the justifications of the modern imperial project . But an
alternative tradition makes these justifications visible and demonstrates their corrosive, lingering presence in contemporary culture through the depiction of nuclear
technology and its possible consequences. Significantly, the
idea that nuclear weapons are used to buttress a racial order
that privileges whiteness – an idea that prohibits non-white peoples from accessing such technology –
remains a potent current running from 1945 until the present day. Having raised this point to emphasize the importance of the
themes in this study, I am mindful to repeat that my focus is literary, cultural and filmic texts. I am not seeking to explain how race and ethnicity have structured
Cold War history. If I may be excused a brief aside, I do think such moments have occurred. Civil rights and Cold War historians have long understood that US

foreign policy had to negotiate the American government’s response to domestic systems of racial
discrimination, and vice versa. Recently decolonized nations whose populations had been excluded along similar lines by European imperialism
followed the narrrative of American desegregation closely, and the allegiances of these nations played and important role in the Cold War. When the black student
James Meredith was not permitted to join the University of Mississippi in 1962, President Kennedy ordered federal marshals to force his registration through. This
took place on 1 October 1962, after a night of fighting between demonstrators and troops. While not universally praised, Kennedy’s actions were widely perceived
in the international press as evidence to resolve to oppose racial discrimination. When the Cuban Missile Crisis took place three weeks later, the presidents of
Guinea and Ghaa denied refuelling facilities to Soviet planes flying to the Caribbean. Kennedy aside Arthur Schlesinger directly attributed the African presidents’
actions to the intervention in Mississippi. The subject of this book is not the mechanisms of history. The subject of this book is the way that representations

of nuclear weapons and the world after nuclear war postulate meanings that are not only fully activated
when considered through a lens of race, ethnicity, nationhood and civilization . In many of the texts discussed, a
primary consideration is whether the vestigial master narrative of white supremacy, the narrative of
racial superiority that underpinned modern European colonization, is being resuscitated . I have in mind Fredric
Jameson’s expression, ‘if interpretation in terms of […] allegorical master narratives remains a constant temptation, this is because such master narratives have
inscribed themselves in the texts as well as in our thinking about them. For Jameson the interpretative act runs the risk of being an act of hermeneutic bad faith –
the risk that the critic finds what they are looking for all along because they gathered up a series of texts whose selection is far from arbitrary, and consequently the
reading of said texts confirms the ubiquity of the historical essence with which they were initially ascribed. Yet, as Jameson writes, one should not be too cynical
about the act of interpretation. If the critical analysis of a text finds evidence of the historical trends it set out to discover the success of the interpretation is not in
itself a reason to reject the idea that texts allow one to think closely and critically about historical attitudes. The act of interpretation can sometimes be the
imposition of a preconvieved set of ideas onto a series of texts chosen precisely because they corroborate the hypothesis being tested, but it can also be credible
because texts are inscribed by history and by master naratives. As a way of referring to an explanation of the movement of history and its future direction,
Jameson’s sense of master narratives is worth retaining. My usage here designates the explanation itself, specifically the
master narrative of white
supremacism that proved so useful to European colonialism and the settlement of North America. How
do texts come to be inscribed by master narratives ? What justification do I have in reading the master narrative o white supremacism
and related narratives of settlement through the literary, cultural and filmic texts analysed here?
Link – Positive Peace
The aff’s understanding of war as an event masks everyday violence during
“peacetime” and reinforces militarism.
Cuomo 96 – PhD, University of Wisconsin Department of Philosophy (1996, Chris, Hypatia, Vol. 111,
Iss. 4, page 30)

In "Gender and `Postmodern' War," Robin Schott introduces some of the ways in which waris currently best seen not as an event but as a
presence (Schott 1995). Schott argues that postmodern understandings of persons, states, and politics, as well as the high-tech nature of much contemporary
warfare and the preponderance of civil and nationalist wars, render an eventbased conception of war inadequate, especially insofar as gender is taken into account.
In this essay, I will expand upon her argument by showing that accounts of war that only focus on events are impoverished in a number of ways, and therefore
feminist consideration of the political, ethical, and ontological dimensions of war and the possibilities for resistance demand a much more complicated approach. I
take Schott's characterization of war as presence as a point of departure, though I am not committed to the idea that the constancy of militarism, the fact of its
omnipresence in human experience, and the paucity of an event-based account of war are exclusive to contemporary postmodern or postcolonial circumstances.(1)
Theory that does not investigate or even notice the omnipresence of militarism cannot represent or
address the depth and specificity of the everyday effects of militarism on women, on people living in
occupied territories, on members of military institutions, and on the environment. These effects are relevant to
feminists in a number of ways because military practices and institutions help construct gendered and national identity, and

because they justify the destruction of natural nonhuman entities and communities during peacetime.
Lack of attention to these aspects of the business of making or preventing military violence in an
extremely technologized world results in theory that cannot accommodate the connections among the
constant presence of militarism, declared wars, and other closely related social phenomena, such as
nationalistic glorifications of motherhood, media violence, and current ideological gravitations to
military solutions for social problems. Ethical approaches that do not attend to the ways in which
warfare and military practices are woven into the very fabric of life in twenty-first century technological
states lead to crisis-based politics and analyses . For any feminism that aims to resist oppression and create alternative social and political
options, crisis-based ethics and politics are problematic because they distract attention from the need for

sustained resistance to the enmeshed, omnipresent systems of domination and oppression that so often
function as givens in most people's lives. Neglecting the omnipresence of militarism allows the false
belief that the absence of declared armed conflicts is peace, the polar opposite of war. It is particularly easy for those whose lives
are shaped by the safety of privilege, and who do not regularly encounter the realities of militarism, to maintain this false belief. The belief that militarism is an
ethical, political concern only regarding armed conflict, creates forms of resistance to militarism that are merely exercises in crisis control. Antiwar resistance is then
mobilized when the "real" violence finally occurs, or when the stability of privilege is directly threatened, and at that point it is difficult not to respond in ways that
make resisters drop all other political priorities. Crisis-driven
attention to declarations of war might actually keep resisters
complacent about and complicitous in the general presence of global militarism. Seeing war as necessarily embedded in constant military presence draws
attention to the fact that horrific, state-sponsored violence is happening nearly all over, all of the time, and that it is perpetrated by military institutions and other
Moving away from crisis-driven politics and ontologies concerning war and military
militaristic agents of the state.

violence also enables consideration of relationships among seemingly disparate phenomena, and
therefore can shape more nuanced theoretical and practical forms of resistance .

The affirmative’s geopolitical imagination of policy centers on macropolitical state


rivalries and threats—this conflict prevention strategy merely stabilizes the world for
the imposition of an institutionalized slow violence that squashes any resistance.
Dalby 11 (Simon, professor @ Carleton University, ‘PEACE AND GEOPOLITICS: IMAGINING PEACEFUL
GEOGRAPHIES,’ Paper for presentation to the University of Newcastle symposium on Peace in
Geography and Politics)
Contemporary social theory might point to Michel Foucault, and the argument drawn from his writings that politics is the extension of war
rather than the other way round. Given the interest in biopolitics and geogovernance within the discipline these matters are obviously relevant
but the connection to peace
needs to be thought carefully beyond formulations that simply assume it as the
opposite of wars (Morrissey 2011). This is especially the case given the changing modes of contemporary
warfare and the advocacy of violence as an appropriate policy in present circumstances. The modes of warfare at the heart of
liberalism suggest that the security of what Reid and Dillon (2009) call the biohuman, the liberal consuming subject, involves a
violent series of practices designed to pacify the world by the elimination of political alternatives . The
tension here suggests an imperial peace, a forceful imposition of a state of non-war . In George W. Bush’s
terms justifying the war on terror, a long struggle to eliminate tyranny (Dalby 2009a). Peace is, in this geopolitical understanding, what comes
after the elimination of opposition. In late 2011 such formulations dominated discussions of the death of Colonel Gadaffi in Libya.¶ The
dramatic transformation of human affairs in the last couple of generations do require that would-be peaceful
geographers look both to the importance of non-violence and simultaneously to how global transformations
are changing the landscape of violence and social change, all of it still under the threat of nuclear
devastation should major inter-state war occur once again. The re-emergence of non-violence as an explicit political strategy, and in
particular the use of Gene Sharp’s (1973) ideas of non-violent direct action in recent events pose these questions very pointedly.
Geographers have much to offer in such re-thinking that may yet play their part in a more global understanding
of how interconnected our fates are becoming and how inappropriate national state boundaries are as the premise for
political action in a rapidly changing biosphere.¶ But to do so some hard thinking is needed on geopolitics, and on how it
works as well as how peace-full scholarship might foster that which it desires . Linking the practical actions of non-
violence from Tahrir Square to those of the Occupy Wall Street actions, underway as the first draft of this paper was keyboarded, requires that
we think very carefully about the practices that now are designated in terms of globalization. Not all this is novel, but the
geopolitical
scene is shifting in ways that need to be incorporated into the new thinking within geography about war,
peace, violence and what the discipline might have to say about, and contribute to, non-violence as well as to
contestations of contemporary lawfare (Gregory 2006).¶ Whether the delegitimization of violence as a mode of rule will be
extended further in coming decades is one of the big questions facing peace researchers. The American reaction to 9/11 set things back
dramatically, an opportunity to respond in terms of response to a crime and diplomacy was squandered, but the wider social refusal to
accept repression and violence as appropriate modes of rule has interesting potential to constrain the use of
military force. The professionalization of many high technology militaries also reduces their inclination to involve themselves in repressing
social movements, although here Mikhail Gorbachev’s refusal to use the Red Army against dissidents in Eastern Europe in the late 1980s
remains emblematic of the changes norms of acceptable rule that have been extended in the last few generations. ¶ A
geography
discipline seriously interested in peace needs to link the social processes on the relatively small scale
such as the non-violent protests Megoran (2011) highlights, and the peaceful accompaniment actions that Koopman
(2011) documents, to the larger geopolitical transformations of our times , to make the eminently geographical
point that peace activities vary widely from place to place, but now are an important part of larger contemporary
geopolitical transformations. ¶ Geopolitics has mostly been about rivalries between great powers and
their contestations of power on the large scale. These specifications of the political world focus on states
and the perpetuation of threats mapped as external dangers to supposedly pacific polities. Much geopolitical
discourse specifies the world as a dangerous place , hence precisely because of these mappings, one
supposedly necessitating violence in what passes for a realist interpretation of great powers as the prime movers
of history (Mearsheimer 2001). Geopolitical thinking is about order and order is in part a cartographic notion. Juliet
Fall (2010) once again emphasizes the importance of taken for granted boundaries as the ontological given of
contemporary politics. Politics is about the cartographic control of territories, as Megoran (2011) too ponders regarding the first half of
the twentieth century, but it also about much more than this, despite the fascination that so many commentators have with the ideal form of
the supposedly national territorial state. Part of what geographers bring to the discussion of peace is a more nuanced
geographical imagination than that found in so much of international studies (Dalby 2011a).¶ On the other hand much of the
discussion of peace sees war as the problem, peace as the solution. Implied in that is geopolitics as the problem, mapping
dangers turns out to be a dangerous enterprise insofar as it facilitates the perpetuation of violence by
representing other places as threats to which our place is susceptible. But this only matters if this is related to the
realist assumptions of the inevitability of rivalry, the eternal search for power as key to humanity’s self-
organisation and the assumption that organized violence is the ultimate arbiter (Dalby 2010). Critical geopolitics is
about challenging such contextualizations, and as such its relationships to peace would seem to be obvious, albeit as Megoran (2011) notes
mostly by way of a focus on what Galtung (1969, 1971) calls negative peace. Given the repeated reinvention of colonial
tropes in contemporary Western political discourse such critique remains an essential part of a political geography
that grants peoples “the courtesy of political geography” (Mitchell and Smith 1991). Undercutting the moral logics of violence,
so frequently relying on simplistic invocations of geographical inevitability, to structure their apologetics, remains a
crucial contribution.¶ Both the practical matters of recent history and the scholarly contributions by geographers do not
allow simple binary distinctions of peace and war to be used as the premise of either scholarship or
political practice. History and scholarship suggest rather that peace is what comes after war; the
relationship is temporal, stages in matters of violence, geography and reorganizing facts on the ground. Historically in the era of
European warfare, coincident with the rise of modernity, that many people hope is now near its end, peace was that which was
imposed by the victors, who in turn were the most powerful in whatever contest was followed by a “peace”. Much recent
geographical scholarship suggests that post conflict re-construction is a mode of peace building literally (Kirsch and Flint
2011). But those of us who would challenge war as a human institution , or think about non-violence as a
strategy for a better world, will not be satisfied with a geography that is concerned only to pick up the pieces and
reconfigure them after they have been shattered by the latest round of organized violence.¶ The key points
are that reconstruction is a violent transformation of society, a world where frequently neo-liberal globalization is
seen as the imposition of social forms that will not resist its logics . Hence peace is what victors impose, an
imperial peace that may eventually be quite welcome to those who benefit from the new arrangements. Is peace then post-war?
Perhaps it can be understood in these terms. But the corollary is the equally important point that peace is also frequently
what comes before the next war. The normal human situation these days is a matter of non-war, but it is far from clear where
security is enforced that this is more than a limited form of negative peace. Without large-scale de-militarization then peace
is just what happens between wars. But given this then one additional key point that geographers interested in war need to pay
attention to is the matter of how peace fails, how conflict escalates and how geography matters in these processes (Flint et al 2009).
Peacekeeping is frequently about geographical separation as the Orwellian names for contemporary walls in terms of lines of peace have it. But
there is much more geographical thinking to be done about these matters and the scales of interactions across supposedly peaceful borders,
not least where what matters most is state security and its ordering principles rather than local interactions across frontiers. This is so not least
because of the marked current trend to build fences around states as the supposed solution to numerous security challenges (Jones 2011). ¶
Putting matters into historical context also suggests that war
is not what it used to be, at least not after the events of the 1940s.
Negative peace is about preventing conflict; non-violence is about political strategies to delegitimise
violence, to challenge the human norms of behavior that allow cultures of violence . It is important to
link this to the issues of what are now called lawfare (Morrissey 2011), the use of law as power and coercion
to set the rules of social and political life too. This has been a key part of the US strategy for a long time;
shaping institutions to the benefit of the US economy as been what much of international relations has been about, but
the larger benefits of constraining conflict are part of the larger process that international law struggles to legitimize. Rules of conduct matter in
the international system and the wide-scale repudiation of the American invasion of Iraq in 2003 demonstrated this point clearly.
Link – Proliferation
Prolif is an epistemological excuse for violence – their discourse wrecks alternative
approaches and trades off with structural violence
Woods 7 Matthew, PhD in IR @ Brown - Researcher @ Thomas Watson Institute of International
Relations [Journal of Language and Politics 6.1“Unnatural Acts: Nuclear Language, proliferation, and
order,” p. 116-7]

It is important to identify, expose and understand the successful creation of 'proliferation' as the inevitable,
uncontrollable and dangerous spread of nuclear arms because it changed the world in innumerable ways . On
one hand, it is the chief motivation for a wide array of cooperative endeavors among states and the central rationale for the most
successful arms control agreement in modern history, the NPT. It inspired sacrifices that led to faith in our regard for others and stimulated
confidence in international law. On the other hand, it is the reason for an unparalleled collection of international
denial and regulatory institutions and it is the omnipresent and ineliminable threat at the heart of our
chronic, unremitting suspicion of others. It is a cause of global inequality and double-standards among
states and the progenitor of the name and identity 'rogue state' (states that reject the whaling ban are not 'rogue
states'). It is a central element in world-wide toleration for human misery, such as starvation in North
Korea, and in public toleration for the clear deception and dissembling of government elites , such as in the
US. It is a vehicle in some media for racial stereotypes. The existence of 'proliferation' is a primary rationale
among nuclear states for preserving and improving their nuclear arsenals. And faith in the existence of
'Proliferation’: most recently, brought about i nvasion, war and continuing death in the Middle East .
Every individual that fears it, organization that studies it and state that strives to prevent it embraces
'proliferation' as a real and known thing and , in part, orients their identity and behavior according to it.
The successful creation of 'proliferation' represents the creation of our common sense, our everyday life and our
natural attitude toward the nuclear world 'out there.' It is uncontestable and to suggest otherwise that nuclear states might
be to blame for any spread of nuclear arms, or that it has actually been rare and so far benign or that it may even be beneficial (see a critical
review of this literature in Woods 2002) - is to invite derision and ostracism. The reality of 'proliferation' is so massive and solidified that the
essential role of (cell) proliferation in maintaining life and health is virtually forgotten, overwhelmed, its positive meaning restricted to the
doctor's office and biology lab. In short, the
creation of 'proliferation' is a textbook example of what some term
hegemony, the creation by a dominant group of a world that realizes its ideological preferences while
marginalizing other possibilities and co-opting subordinates.

Proliferation as a security frame structures Western imagination so as to produce an


irrational and dangerous “rogue” other. This discourse is racist and genocidal.
Gusterson ‘6 (Hugh, Associate Professor of Anthropology and Science and Technology Studies at MIT, April 2006, “A Double Standard
on Nuclear Weapons?,” MIT Center for International Studies Audit of the Conventional Wisdom, 06-08)

Nuclear Orientalism

According to the anthropological literature on risk, shared


fears often reveal as much about the identities and
solidarities of the fearful as about the actual dangers that are feared. The immoderate reactions in the
West to the nuclear tests conducted in 1998 by India and Pakistan, and to Iraq’s nuclear weapons program earlier, are
examples of an entrenched discourse on nuclear proliferation that has played an important role in
structuring the Third World, and our relation to it, in the Western imagination . This discourse, dividing
the world into nations that can be trusted with nuclear weapons and those that cannot, dates back , at
least, to the Non-Proliferation Treaty of 1970. The Non-Proliferation Treaty embodied a bargain between the five
countries that had nuclear weapons in 1970 and those countries that did not. According to the bargain, the
five official nuclear states (the United States, the Soviet Union, the United Kingdom, France, and China) promised to assist
other signatories to the treaty in acquiring nuclear energy technology as long as they did not use that
technology to produce nuclear weapons, submitting to international inspections when necessary to prove
their compliance. Further, in Article 6 of the treaty, the five nuclear powers agreed to pursue “negotiations in
good faith on effective measures relating to cessation of the nuclear arms race at an early date and to
nuclear disarmament.” One hundred eighty-seven countries have signed the treaty. Saying it enshrines a system of global ‘nuclear
apartheid,’ Israel, India, and Pakistan have refused. North Korea has withdrawn from the treaty. The Non-Proliferation Treaty has
become the legal anchor for a global nuclear regime that is increasingly legitimated in Western public
discourse in racialized terms. In view of recent developments in global politics—the collapse of the Soviet Union, two wars against
Iraq, and international crises over the nuclear weapons programs of North Korea and Iran— the importance of this discourse in
organizing Western geopolitical understandings is only growing. It has become an increasingly
important way of legitimating U.S. military programs in the post-cold war world, where “rogue states”
have supplanted the old evil empire in the imaginations of American war planners. The dominant
discourse that stabilizes this system of nuclear apartheid in Western ideology was labeled “Orientalism” (albeit in a different
context) by Edward Said. According to Said, orientalist discourse constructs the world in terms of a series of binary
oppositions that produce the Orient as the mirror image of the West: where “we” are rational and
disciplined, “they” are impulsive and emotional; where “we” are modern and flexible, “they” are slaves
to ancient passions and routines; where “we” are honest and compassionate, “they” are treacherous
and uncultivated. 6 While the blatantly racist orientalism of the high colonial period has softened, more subtle orientalist ideologies
endure in contemporary politics and are applicable here.
Link – Terrorism
Securitizing terrorism causes violent lashout and reproduces their harms.
Mitchell 5 (Andrew , Stanford U. Humanities Post-Doctoral Fellow, "Heidegger and Terrorism," Research in Phenomenology, Volume
35, Number 1, 2005 , pp. 181-218)

Government and politics are simply further means of directing ways of life according to plan; and no
one, neither terrorist nor politician, should be able to alter these carefully constructed ways of life. Ways
of life are themselves effects of the plan, and the predominant way of life today is that of an all-consuming
Americanism. National differences fall to the wayside . The homeland, when not completely outmoded, can only appear as
commodified quaintness. All governments participate in the eradication of national differences. Insofar as Americanism represents the attempt
to annihilate the "homeland," then under
the aegis of the abandonment of being, all governments and forms of
leadership become Americanism. The loss of national differences is accordant with the advent of
terrorism, since terrorism knows no national bounds but, rather, threatens difference and boundaries as such.
Terrorism is everywhere, where "everywhere" no longer refers to a collection of distinct places and locations but instead to a "here" that is the
same as there, as every "there." The threat of terrorism is not international, but antinational or, to strain a Heideggerian formulation,
Homeland security, insofar as it destroys the very thing that it claims to protect, is nothing
unnational.
opposed to terrorism, but rather the consummation of its threat. Our leaders, in their attempt to
secure the world against terrorism, only serve to further drive the world towards its homogenized
state. The elimination of difference in the standing-reserve along with the elimination of national differences serve to identify the threat of
terrorism with the quest for security. The absence of this threat would be the absence of being, and its consummation would be the absence of
being as well. Security
is only needed where there is a threat. If a threat is not perceived, if one believes
oneself invulnerable, then there is no need for security. Security is for those who know they can be
injured, for those who can be damaged. Does America know that it can be damaged? If security requires a recognition of one's
own vulnerability, then security can only be found in the acknowledgment of one's threatened condition, and this means that it can only be
found in a recognition of being as threat. To
be secure, there must be the threat. For this reason, all of the planned
securities that attempt to abolish the threat can never achieve the security they seek. Security requires
that we preserve the threat, and this means that we must act in the office of preservers . As preservers, what
we are charged to preserve is not so much the present being as the concealment that inhabits it. Preserving a thing means to not challenge it
forth into technological availability, to let it maintain an essential concealment. That we participate in this essencing of being does not make of
it a subjective matter, for there is no isolated subject in preservation, but an opening of being. Heidegger will name this the clearing of the truth
(Wahrhet) of being, and it is this clearing that Dasein preserves (bewahrt). When a thing truthfully is, when it is what it is in truth, then it is
preserved. In preserving beings, Dasein participates in the truth (preservation) of being. The truth of being is being as threat, and this threat
only threatens when Dasein preserves it in terror. Dasein is not innocent in the terrorization of being. On the contrary, Dasein is complicit in it.
Dasein refuses to abolish terrorism. For this reason, a Heideggerian thinking of terrorism must remain skeptical of all the various measures
taken to oppose terrorism, to root it out or to circumvent it. These are so many attempts to do away with what threatens, measures that are
themselves in the highest degree willful. This will can only impose itself upon being, can only draw out more and more of its wrath, and this
inward wrath of being maintains itself in a never-ending supply. The will can only devastate the earth. Rather than approaching the world in
terms of resources to be secured, true security can only be found in the preservation of the threat of being. It
is precisely when we
are busy with security measures and the frantic organization of resources that we directly assault the
things we would preserve. The threat of being goes unheeded when things are restlessly shuttled back
and forth, harried, monitored, and surveilled. The threat of being is only preserved when things are
allowed to rest. In the notes to the "Evening Conversation," security is thought in just such terms: Security (what one understands by this)
arises not from securing and the measures taken for this; security resides in rest [in der Ruhe] and is itself made superfluous by this. (MA 77:
244)23 The rest in question is a rest from the economic cycling and circulating of the standing reserve. The technological unworld, the situation
of total war, is precisely the era of restlessness ("The term 'totality' says nothing more; it names only the spread of the hitherto known into the
'restless"' [GA 69: 181]). Security is superfluous here, which is only to say that it is unnecessary or useless. It is not found in utility, but in the
preserved state of the useless. Utility and function are precisely the dangers of a civil that has turned antagonistic towards nature. In rest, they
no longer determine the being of the thing. In resting, things are free of security measures, but not for all that rendered insecure. Instead, they
are preserved. There is no security; this is what we have to preserve. Heideggerian thinking is a thinking that thinks away from simple presence
and absence. It thinks what Heidegger calls "the between" (das Zwischen). This between is a world of nonpresence and nonabsence.
Annihilation is impossible for this world and so is security. The terror experienced today is a clue to the withdrawal of being. The world is
denatured, drained of reality. Everything is threatened and the danger only ever increases. Dasein flees to a metaphysics of presence to escape
the threatened world, hoping there to find security. But security cannot do away with the threat, rather it must guard it. Dasein guards the
truth of being in the experience of terror. What is perhaps repugnant to consider in all this is that being calls for terrorism and for
terrorists. With the enframing of being and the circulation of standing-reserve, what is has already been destroyed. Terrorism is merely the
ugly confirmation of this point. As we have seen, being does not linger behind the scenes but is found in the staging itself. If being is to
terrorize-if, in other words, this is an age of terrorism-then being must call for terrorists. They are simply
more "slaves of the history of beyng" (GA 69: 209) and, in Heidegger's eyes, no different from the politicians
of the day in service to the cause of Americanism . But someone might object, the terrorists are murderers and the politicians
are not. Granting this objection despite its obvious naivety, we can nonetheless see that both politicians and terrorists are called for
by the standing-reserve, the one to ensure its nonabsence, that the plan will reach everyone
everywhere, and the other to ensure its nonpresence, that all beings will now be put into circulation by
the threat of destruction. In this regard, "human resources" are no different from "livestock," and with this,
an evil worse than death has already taken place. Human resources do not die, they perish.
Impacts
Impact – Global War
Security discourse sanitizes global destruction by proliferating symptom-focused
solutions to power imbalances – causes cycles of violence that make global warfare
and extinction inevitable
Ahmed 11 [2011, Dr. Nafeez Mosaddeq Ahmed is Executive Director of the Institute for Policy
Research and Development [IPRD], an independent think tank focused on the study of violent conflict,
he has taught at the Department of International Relations, University of Sussex "The international
relations of crisis and the crisis of international relations: from the securitisation of scarcity to the
militarisation of society" Global Change, Peace %26 Security Volume 23, Issue 3, Taylor Francis]

This analysis thus calls for a broader approach to environmental security based on retrieving the manner
in which political actors construct discourses of ‘scarcity’ in response to ecological, energy and economic
crises [critical security studies] in the context of the historically-specific socio-political and geopolitical
relations of domination by which their power is constituted, and which are often implicated in the
acceleration of these very crises [historical sociology and historical materialism]. Instead, both realist and liberal
orthodox IR approaches focus on different aspects of interstate behaviour, conflictual and cooperative respectively, but each lacks the capacity
to grasp that the unsustainable trajectory of state and inter-state behaviour is only explicable in the context of a wider global system
concurrently over-exploiting the biophysical environment in which it is embedded. Theyare, in other words, unable to
addressthe relationship of the inter-state system itself to the biophysical environment as a key analytical
category for understanding the acceleration of global crises. They simultaneously therefore cannot
recognise the embeddedness of the economy in society and the concomitant politically-constituted
nature of economics.84 Hence, they neglect the profound irrationality of collective state behaviour, which
systematically erodes this relationship, globalising insecurity on a massive scale – in the very process of
seeking security.85 In Cox’s words, because positivist IR theory ‘does not question the present order [it instead] has the effect of
legitimising and reifying it’. 86 Orthodox IR sanitises globally-destructive collective inter-state behaviour as a
normal function of instrumental reason – thus rationalising what are clearly deeply irrational
collective human actions that threaten to permanently erode state power and security by destroying
the very conditions of human existence. Indeed, the prevalence of orthodox IR as a body of disciplinary
beliefs, norms and prescriptions organically conjoined with actual policy-making in the international
system highlights the extent to which both realism and liberalism are ideologically implicated in the
acceleration of global systemic crises.87 By the same token, the incapacity to recognise and critically
interrogate how prevailing social, political and economic structures are driving global crisis acceleration
has led to the proliferation of symptom-led solutions focused on the expansion of state/regime military–
political power rather than any attempt to transform root structural cause s.88 It is in this context that, as the
prospects for meaningful reform through inter-state cooperation appear increasingly nullified under the pressure of actors with a vested
interest in sustaining prevailing geopolitical and economic structures, states have resorted progressively more to militarised responses designed
to protect the concurrent structure of the international system from dangerous new threats. In effect, the
failure of orthodox
approaches to accurately diagnose global crises, directly accentuates a tendency to ‘securitise’them–
and this, ironically, fuels the proliferation of violent conflict and militarisation responsible for magnified
global insecurity. ‘Securitisation’ refers to a ‘speech act’ – an act of labelling – whereby political
authorities identify particular issues or incidents as an existential threat which, because of their extreme
nature, justify going beyond the normal security measures that are within the rule of law . It thus
legitimises resort to special extra-legal powers. By labelling issues a matter of ‘security’, therefore, states
are able to move them outside the remit of democratic decision-making and into the realm of
emergency powers, all in the name of survival itself. Far from representing a mere aberration from
democratic state practice, this discloses a deeper ‘dual’ structure of the state in its institutionalisation of
the capacity to mobilise extraordinary extra-legal military– police measures in purported response to an
existential danger.89 The problem in the context of global ecological, economic and energy crises is that such levels of emergency
mobilisation and militarisation have no positive impact on the very global crises generating ‘new security challenges’, and are thus entirely
disproportionate.90 All that remains to examine is on the ‘surface’ of the international system [geopolitical competition, the balance of power,
international regimes, globalisation and so on], phenomena which are dislocated from their structural causes by way of being unable to
recognise the biophysically-embedded and politically-constituted social relations of which they are comprised. The
consequence is
that orthodox IR has no means of responding to global systemic crises other than to reduce them to
their symptoms. Indeed, orthodox IR theory has largely responded to global systemic crises not with new
theory, but with the expanded application of existing theory to ‘new security challenges’ such as ‘low-
intensity’ intra-state conflicts; inequality and poverty; environmental degradation; international criminal
activities including drugs and arms trafficking; proliferation of weapons of mass destruction; and
international terrorism.91 Although the majority of such ‘new security challenges’ are non-military in origin – whether their referents
are states or individuals – the inadequacy of systemic theoretical frameworks to diagnose them means they are
primarily examined through the lenses of military-political power .92 In other words, the escalation of global
ecological, energy and economic crises is recognised not as evidence that the current organisation of the
global political economy is fundamentally unsustainable, requiring urgent transformation, but as
vindicating the necessity for states to radicalise the exertion of their military–political capacities to
maintain existing power structures, to keep the lid on .93 Global crises are thus viewed as amplifying
factors that could mobilise the popular will in ways that challenge existing political and economic
structures, which it is presumed [given that state power itself is constituted by these structures] deserve
protection. This justifies the state’s adoption of extra-legal measures outside the normal sphere of
democratic politics. In the context of global crisis impacts, this counter-democratic trend-line can result
in a growing propensity to problematise potentially recalcitrant populations – rationalising violence
toward them as a control mechanism. 3.2 From theory to policy Consequently, for the most part, the policy implications of
orthodox IR approaches involve a redundant conceptualisation of global systemic crises purely as potential ‘threat-multipliers’ of traditional
security issues such as ‘political instability around the world, the collapse of governments and the creation of terrorist safe havens’. Climate
change will serve to amplify the threat of international terrorism, particularly in regions with large populations and scarce resources.94 The US
Army, for instance, depicts climate change as a ‘stress-multiplier’ that will ‘exacerbate tensions’ and ‘complicate American foreign policy’; while
the EU perceives it as a ‘threat-multiplier which exacerbates existing trends, tensions and instability’. 95 In practice, this generates an excessive
preoccupation not with the causes of global crisis acceleration and how to ameliorate them through structural transformation, but with their
purportedly inevitable impacts, and how to prepare for them by controlling problematic populations. Paradoxically, this
‘securitisation’
of global crises does not render us safer. Instead, by necessitating more violence, while inhibiting
preventive action, it guarantees greater insecurity. Thus, a recent US Department of Defense report
explores the future of international conflict up to 2050. It warns of ‘resource competition induced by
growing populations and expanding economies’, particularly due to a projected ‘youth bulge’ in the
South, which ‘will consume ever increasing amounts of food, water and energy’. This will prompt a
‘return to traditional security threats posed by emerging near-peers as we compete globally for
depleting natural resources and overseas markets’ . Finally, climate change will ‘compound’ these stressors by generating
humanitarian crises, population migrations and other complex emergencies.96 A similar study by the US Joint Forces Command draws attention
to the danger of global energy depletion through to 2030. Warning of ‘the dangerous vulnerabilities the growing energy crisis presents’, the
report concludes that ‘The
implications for future conflict are ominous.’ 97 Once again, the subject turns to
demographics: ‘In total, the world will add approximately 60 million people each year and reach a total
of 8 billion by the 2030s’, 95 per cent accruing to developing countries, while populations in developed
countries slow or decline. ‘Regions such as the Middle East and Sub-Saharan Africa, where the youth
bulge will reach over 50% of the population, will possess fewer inhibitions about engaging in conflict.’ 98
The assumption is that regions which happen to be both energy-rich and Muslim-majority will also be sites of violent conflict due to their
rapidly growing populations. A British Ministry of Defence report concurs with this assessment, highlighting an inevitable ‘youth bulge’ by 2035,
with some 87 per cent of all people under the age of 25 inhabiting developing countries. In particular, the Middle East population will increase
by 132 per cent and sub-Saharan Africa by 81 per cent. Growing resentment due to ‘endemic unemployment’ will be channelled through
‘political militancy, including radical political Islam whose concept of Umma, the global Islamic community, and resistance to capitalism may lie
uneasily in an international system based on nation-states and global market forces’. More strangely, predicting an intensifying global divide
between a super-rich elite, the middle classes and an urban under-class, the report warns: ‘ The
world’s middle classes might
unite, using access to knowledge, resources and skills to shape transnational processes in their own class
interest.’ 99 3.3 Exclusionary logics of global crisis securitisation? Thus, the securitisation of global crisis
leads not only to the problematisation of particular religious and ethnic groups in foreign regions of
geopolitical interest, but potentially extends this problematisation to any social group which might
challenge prevailing global political economic structures across racial, national and class lines. The previous
examples illustrate how securitisation paradoxically generates insecurity by reifying a process of militarisation against social groups that are
constructed as external to the prevailing geopolitical and economic order. In other words, the internal reductionism, fragmentation and
compartmentalisation that plagues orthodox theory and policy reproduces precisely these characteristics by externalising global crises from
one another, externalising states from one another, externalising the inter-state system from its biophysical environment, and externalising
new social groups as dangerous ‘outsiders’. Hence, a simple discursive analysis of state militarisation and the construction of new ‘outsider’
identities is insufficient to understand the causal dynamics driving the process of ‘Otherisation’. As Doug Stokes points out, the Western state
preoccupation with the ongoing military struggle against international terrorism reveals an underlying ‘discursive complex’, where
representations about terrorism and non-Western populations are premised on ‘the construction of stark boundaries’ that ‘operate to exclude
and include’. Yet these exclusionary discourses are ‘intimately bound up with political and economic processes’, such as strategic interests in
proliferating military bases in the Middle East, economic interests in control of oil, and the wider political goal of ‘maintaining American
hegemony’ by dominating a resource-rich region critical for global capitalism.100 But even this does not go far enough, for arguably the
construction of certain hegemonic discourses is mutually constituted by these geopolitical, strategic and economic interests – exclusionary
discourses are politically constituted. New conceptual developments in genocide studies throw further light on this in terms of the concrete
socio-political dynamics of securitisation processes. It is now widely recognised, for instance, that the distinguishing criterion of genocide is not
the pre-existence of primordial groups, one of which destroys the other on the basis of a preeminence in bureaucratic military–political power.
Rather, genocide
is the intentional attempt to destroy a particular social group that has been socially
constructed as different. 101 As Hinton observes, genocides precisely constitute a process of‘othering’in which an imagined
community becomes reshaped so that previously ‘included’ groups become ‘ideologically recast’ and dehumanised as threatening and
dangerous outsiders, be it along ethnic, religious, political or economic lines – eventually legitimising their annihilation.102 In other words,
genocidal violence is inherently rooted in a prior and ongoing ideological process, whereby exclusionary
group categories are innovated, constructed and ‘Otherised’ in accordance with a specific socio-political
programme. The very process of identifying and classifying particular groups as outside the boundaries
of an imagined community of ‘inclusion’, justifying exculpatory violence toward them, is itself a political
act without which genocide would be impossible.103 This recalls Lemkin’s recognition that the intention
to destroy a group is integrally connected with a wider socio-political project – or colonial project –
designed to perpetuate the political, economic, cultural and ideological relations of the perpetrators in
the place of that of the victims, by interrupting or eradicating their means of social reproduction . Only by
interrogating the dynamic and origins of this programme to uncover the social relations from which that programme derives can the emergence
of genocidal intent become explicable.104 Building on this insight, Semelin demonstrates that the process of exclusionary social group
construction invariably derives from political processes emerging from deep-seated sociopolitical crises that undermine the prevailing
framework of civil order and social norms; and which can, for one social group, be seemingly resolved by projecting anxieties onto a new
‘outsider’ group deemed to be somehow responsible for crisis conditions. It
is in this context that various forms of mass
violence, which may or may not eventually culminate in actual genocide, can become legitimised as
contributing to the resolution of crises.105 This does not imply that the securitisation of global crises by
Western defence agencies is genocidal. Rather, the same essential dynamics of social polarisation and
exclusionary group identity formation evident in genocides are highly relevant in understanding the
radicalisation processes behind mass violence. This highlights the fundamental connection between social crisis, the
breakdown of prevailing norms, the formation of new exclusionary group identities, and the projection of blame for crisis onto a newly
constructed ‘outsider’ group vindicating various forms of violence. Conclusions While recommendations to shift our frame of orientation away
from conventional state-centrism toward a ‘human security’ approach are valid, this cannot be achieved without confronting the deeper
theoretical assumptions underlying conventional approaches to ‘non-traditional’ security issues.106 By occluding the structural origin and
systemic dynamic of global ecological, energy and economic crises, orthodox approaches are incapable of transforming them. Coupled with
their excessive state-centrism, this means they operate largely at the level of ‘surface’ impacts of global crises in terms of how they will affect
quite traditional security issues relative to sustaining state integrity, such as international terrorism, violent conflict and population movements.
Global crises end up fuelling the projection of risk onto social networks, groups and countries that cross the geopolitical fault-lines of these
‘surface’ impacts – which happen to intersect largely with Muslim communities. Hence, regions particularly vulnerable to climate change
impacts, containing large repositories of hydrocarbon energy resources, or subject to demographic transformations in the context of rising
population pressures, have become the focus of state security planning in the context of counter-terrorism operations abroad. The intensifying
problematisation and externalisation of Muslim-majority regions and populations by Western security agencies – as a discourse – is therefore
not only interwoven with growing state perceptions of global crisis acceleration, but driven ultimately by an epistemological failure to
interrogate the systemic causes of this acceleration in collective state policies [which themselves occur in the context of particular social,
political and economic structures]. This
expansion of militarisation is thus coeval with the subliminal normative
presumption that the social relations of the perpetrators, in this case Western states, must be protected
and perpetuated at any cost – precisely because the efficacy of the prevailing geopolitical and economic
order is ideologically beyond question. As much as this analysis highlights a direct link between global
systemic crises, social polarisation and state militarisation, it fundamentally undermines the idea of a
symbiotic link between natural resources and conflict per se. Neither ‘resource shortages’ nor ‘resource abundance’ [in
ecological, energy, food and monetary terms] necessitate conflict by themselves. There are two key operative factors that determine whether
either condition could lead to con- flict. The first is the extent to which either condition can generate socio-political crises that challenge or
undermine the prevailing order. The second is the way in which stakeholder actors choose to actually respond to the latter crises. To
understand these factors accurately requires close attention to the political, economic and ideological strictures of resource exploitation,
consumption and distribution between different social groups and classes. Overlooking the systematic causes of social crisis
leads to a heightened tendency to problematise its symptoms, in the forms of challenges from particular
social groups. This can lead to externalisation of those groups, and the legitimisation of violence towards
them. Ultimately, this systems approach to global crises strongly suggests that conventional policy ‘reform’ is woefully inadequate. Global
warming and energy depletion are manifestations of a civilisation which is in overshoot. The current scale and organisation of human activities
is breaching the limits of the wider environmental and natural resource systems in which industrial civilisation is embedded. This breach is now
increasingly visible in the form of two interlinked crises in global food production and the global financial system. In short, industrial civilisation
in its current form is unsustainable. This calls for a process of wholesale civilisational transition to adapt to the inevitable arrival of the post-
carbon era through social, political and economic transformation. Yet conventional
theoretical and policy approaches fail to
[1] fully engage with the gravity of research in the natural sciences and [2] translate the social science
implications of this research in terms of the embeddedness of human social systems in natural systems.
Hence, lacking capacity for epistemological self-reflection and inhibiting the transformative responses
urgently required, they reify and normalise mass violence against diverse ‘Others’, newly constructed as
traditional security threats enormously amplified by global crises – a process that guarantees the
intensification and globalisation of insecurity on the road to ecological, energy and economic
catastrophe. Such an outcome, of course, is not inevitable, but extensive new transdisciplinary research
in IR and the wider social sciences – drawing on and integrating human and critical security studies,
political ecology, historical sociology and historical materialism, while engaging directly with
developments in the natural sciences – is urgently required to develop coherent conceptual frameworks
which could inform more sober, effective, and joined-up policy-making on these issues.
Impact – Value to Life
Security politics destroys the value to life
Der Derian 93 (James, “The value of security: Hobbes, Marx, Nietzsche, and Baudrillard,” in The
Political Subject of Violence, pp. 102-105)

The desire for securityis manifested as a collective resentment of difference that which is not us, not certain, not
predictable. Complicit with a negative will to power is the fear-driven desire for protection from the unknown. Unlike the positive will to
power which produces an aesthetic affirmation of difference, the search for truth produces a truncated life which
conforms to the rationally knowable, to the causally sustainable. In The Gay Science Nietzsche asks of the reader: Look, isn't our
need for knowledge precisely this need for the familiar, the will to uncover everything strange, unusual, and questionable, something that no
longer disturbs us? Is it not the instinct of fear that bids us to know? And is the jubilation of those who obtain knowledge not the jubilation over
the restoration of a sense of security?" The
fear of the unknown and the desire for certainty combine to produce a
domesticated life, in which causality and rationality become the highest sign of a sovereign self, the surest
protection against contingent forces. The fear of fate assures a belief that everything reasonable is true, and everything true reasonable. In
short, the security imperative produces and is sustained by the strategies of knowledge which seek to explain it. Nietzsche elucidates the nature
of this generative relationship in The Twilight of the Idols: A safe life requires safe truths. The strange and the alien remain unexamined, the
unknown becomes identified as evil, and evil provokes hostility - recycling the desire for security. The 'influence of
timidity,' as Nietzsche puts it, creates a people who are willing to subordinate affirmative values to the
'necessities' of security: 'they fear change, transitoriness: this expresses a straitened soul, full of mistrust
and evil experiences'." The point of Nietzsche's critical genealogy is to show the perilous conditions which created the
security imperative - and the western metaphysics which perpetuate it - have diminished if not disappeared; yet the
fear of life persists: 'Our century denies this perilousness, and does so with a good conscience: and yet it continues to drag along with it
the old habits of Christian security, Christian enjoyment, recreation and evaluation." Nietzsche's worry is that the collective reaction against
older, more primal fears has created an even worse danger: the tyranny of the herd, the lowering of man, the apathy of the last man which
controls through conformity and rules through passivity. The
security of the sovereign, rational self and state comes at
the cost of ambiguity, uncertainty, paradox - all that makes life worthwhile. Nietzsche's lament for this lost life is
captured at the end of Daybreak in a series of rhetorical questions:
Structural Violence First
Structural violence outweighs – ignoring slow violence creates priming that
psychologically structures escalation and makes war inevitable
Scheper-Hughes and Bourgois 4 (Nancy, Prof of Anthropology @ Cal-Berkeley and Philippe, Prof
of Anthropology @ UPenn, Introduction: Making Sense of Violence, in Violence in War and Peace, pg.
19-22)
This large and at first sight “messy” Part VII is central to this anthology’s thesis. It encompasses everything from the routinized, bureaucratized,
and utterly banal violence of children dying of hunger and maternal despair in Northeast Brazil (Scheper-Hughes, Chapter 33) to elderly African
Americans dying of heat stroke in Mayor Daly’s version of US apartheid in Chicago’s South Side (Klinenberg, Chapter 38) to the racialized class
hatred expressed by British Victorians in their olfactory disgust of the “smelly” working classes (Orwell, Chapter 36). In these readings violence
is located in the symbolic and social structures that overdetermine and allow the criminalized drug addictions, interpersonal bloodshed, and
racially patterned incarcerations that characterize the US “inner city” to be normalized (Bourgois, Chapter 37 and Wacquant, Chapter 39).
Violence also takes the form of class, racial, political self-hatred and adolescent self-destruction (Quesada, Chapter 35), as well as of useless (i.e.
Absolutely central to our approach is a
preventable), rawly embodied physical suffering, and death (Farmer, Chapter 34).
blurring of categories and distinctions between wartime and peacetime violence. Close attention to the
“little” violences produced in the structures, habituses, and mentalites of everyday life shifts our
attention to pathologies of class, race, and gender inequalities . More important, it interrupts the voyeuristic tendencies
of “violence studies” that risk publicly humiliating the powerless who are often forced into complicity with social and individual pathologies of
power because suffering is often a solvent of human integrity and dignity. Thus, in this anthology we are positing a violence continuum
comprised of a multitude of “small wars and invisible genocides” (see also Scheper- Hughes 1996; 1997; 2000b) conducted in the normative
social spaces of public schools, clinics, emergency rooms, hospital wards, nursing homes, courtrooms, public registry offices, prisons, detention
violence continuum also refers to the ease with which humans are capable of
centers, and public morgues. The
reducing the socially vulnerable into expendable nonpersons and assuming the license - even the
duty - to kill, maim, or soul-murder. We realize that in referring to a violence and a genocide continuum we are flying in the face
of a tradition of genocide studies that argues for the absolute uniqueness of the Jewish Holocaust and for vigilance with respect to restricted
purist use of the term genocide itself (see Kuper 1985; Chaulk 1999; Fein 1990; Chorbajian 1999). But we hold an opposing and alternative view
that, to the contrary, it
is absolutely necessary to make just such existential leaps in purposefully linking
violent acts in normal times to those of abnormal times. Hence the title of our volume: Violence in War and in Peace. If (as
we concede) there is a moral risk in overextending the concept of “genocide” into spaces and corners of everyday life where we might not
ordinarily think to find it (and there
is), an even greater risk lies in failing to sensitize ourselves, in misrecognizing
protogenocidal practices and sentiments daily enacted as normative behavior by “ordinary” good-
enough citizens. Peacetime crimes, such as prison construction sold as economic development to impoverished communities in the
mountains and deserts of California, or the evolution of the criminal industrial complex into the latest peculiar institution for managing race
relations in the United States (Waquant, Chapter 39), constitute the “small wars and invisible genocides” to which we refer.
This applies to African American and Latino youth mortality statistics in Oakland, California, Baltimore, Washington DC, and New York City.
These are “invisible” genocides not because they are secreted away or hidden from view, but quite
the opposite. , the things that are hardest to perceive are those which are right
As Wittgenstein observed

before our eyes and therefore taken for granted. In this regard, Bourdieu’s partial and unfinished theory of violence (see
Chapters 32 and 42) as well as his concept of misrecognition is crucial to our task. By including the normative everyday forms of violence hidden
in the minutiae of “normal” social practices - in the architecture of homes, in gender relations, in communal work, in the exchange of gifts, and
so forth - Bourdieu forces us to reconsider the broader meanings and status of violence, especially the links between the violence of everyday
life and explicit political terror and state repression, Similarly, Basaglia’s notion of “peacetime crimes” - crimini di pace - imagines a direct
relationship between wartime and peacetime violence. Peacetimecrimes suggests the possibility that war crimes are
merely ordinary, everyday crimes of public consent applied systematic- ally and dramatically in the
extreme context of war. Consider the parallel uses of rape during peacetime and wartime, or the family resemblances between the
legalized violence of US immigration and naturalization border raids on “illegal aliens” versus the US government- engineered genocide in 1938,
known as the Cherokee “Trail of Tears.” Peacetime crimes suggests that everyday forms of state violence make a certain kind of domestic peace
possible. Internal “stability” is purchased with the currency of peacetime crimes, many of which take the form of professionally applied
“strangle-holds.” Everyday forms of state violence during peacetime make a certain kind of domestic “peace” possible. It is an easy-to-identify
peacetime crime that is usually maintained as a public secret by the government and by a scared or apathetic populace. Most subtly, but no less
politically or structurally, the phenomenal growth in the United States of a new military, postindustrial prison industrial complex has taken
place in the absence of broad-based opposition, let alone collective acts of civil disobedience. The
public consensus is based
primarily on a new mobilization of an old fear of the mob, the mugger, the rapist, the Black man, the
undeserving poor. How many public executions of mentally deficient prisoners in the United States are
needed to make life feel more secure for the affluent? What can it possibly mean when incarceration becomes the
“normative” socializing experience for ethnic minority youth in a society, i.e., over 33 percent of young African American men (Prison Watch
is essential that we recognize the existence of a genocidal capacity among otherwise
2002). In the end it
good-enough humans and that we need to exercise a defensive hypervigilance to the less dramatic,
permitted, and even rewarded everyday acts of violence that render participation in genocidal
acts and policies possible (under adverse political or economic conditions), perhaps more easily than we would like to recognize.
Under the violence continuum we include, therefore, all expressions of radical social exclusion,
dehumanization, depersonal- ization, pseudospeciation, and reification which normalize atrocious
behavior and violence toward others. A constant self-mobilization for alarm, a state of constant
hyperarousal is, perhaps, a reasonable response to Benjamin’s view of late modern history as a chronic
“state of emergency” (Taussig, Chapter 31). We are trying to recover here the classic anagogic thinking that enabled Erving Goffman,
Jules Henry, C. Wright Mills, and Franco Basaglia among other mid-twentieth-century radically critical thinkers, to perceive the symbolic and
structural relations, i.e., between inmates and patients, between concentration camps, prisons, mental hospitals, nursing homes, and other
“total institutions.” Making
that decisive move to recognize the continuum of violence allows us to see the
capacity and the willingness - if not enthusiasm - of ordinary people, the practical technicians of the
social consensus, to enforce genocidal-like crimes against categories of rubbish people. There is no
primary impulse out of which mass violence and genocide are born, it is ingrained in the common
sense of everyday social life. The mad, the differently abled, the mentally vulnerable have often fallen
into this category of the unworthy living, as have the very old and infirm, the sick-poor, and, of course,
the despised racial, religious, sexual, and ethnic groups of the moment. Erik Erikson referred to “pseudo- speciation”
as the human tendency to classify some individuals or social groups as less than fully human - a prerequisite to genocide and one that is

carefully honed during the unremark- able peacetimes that precede the sudden, “seemingly unintelligible” outbreaks of mass violence .
Collective denial and misrecognition are prerequisites for mass violence and genocide . But so are formal
bureaucratic structures and professional roles. The practical technicians of everyday violence in the backlands of Northeast Brazil (Scheper-
Hughes, Chapter 33), for example, include the clinic doctors who prescribe powerful tranquilizers to fretful and frightfully hungry babies, the
Catholic priests who celebrate the death of “angel-babies,” and the municipal bureaucrats who dispense free baby coffins but no food to
Everyday violence encompasses the implicit, legitimate, and routinized forms of violence
hungry families.
inherent in particular social, economic, and political formations. It is close to what Bourdieu (1977, 1996) means by
“symbolic violence,” the violence that is often “nus-recognized” for something else, usually something good. Everyday violence is similar to
what Taussig (1989) calls “terror as usual.” All these terms are meant to reveal a public secret - the hidden links between violence in war and
violence in peace, and between war crimes and “peace-time crimes.” Bourdieu (1977) finds domination and violence in the least likely places -
in courtship and marriage, in the exchange of gifts, in systems of classification, in style, art, and culinary taste- the various uses of culture.
Violence, Bourdieu insists, is everywhere in social practice. It is misrecognized because its very everydayness and its familiarity render it
invisible. Lacan identifies “rneconnaissance” as the prerequisite of the social. The exploitation of bachelor sons, robbing them of autonomy,
independence, and progeny, within the structures of family farming in the European countryside that Bourdieu escaped is a case in point
(Bourdieu, Chapter 42; see also Scheper-Hughes, 2000b; Favret-Saada, 1989). Following Gramsci, Foucault, Sartre, Arendt, and other modern
theorists of power-vio- lence, Bourdieu treats direct aggression and physical violence as a crude, uneconomical mode of domination; it is less
efficient and, according to Arendt (1969), it is certainly less legitimate. While power and symbolic domination are not to be equated with
violence - and Arendt argues persuasively that violence is to be understood as a failure of power - violence, as we are presenting it here, is more
than simply the expression of illegitimate physical force against a person or group of persons. Rather, we need to understand violence as
encompassing all forms of “controlling processes” (Nader 1997b) that assault basic human freedoms and individual or collective survival. Our
task is to recognize these gray zones of violence which are, by definition, not obvious. Once again, the point of bringing into the discourses on
genocide everyday, normative experiences of reification, depersonalization, institutional confinement, and acceptable death is to help answer
the question: What makes mass violence and genocide possible? In this volume we are suggesting that mass violence is part of a
continuum, and that it is socially incremental and often experienced by perpetrators, collaborators,
bystanders - and even by victims themselves - as expected, routine, even justified. The preparations for mass
killing can be found in social sentiments and institutions from the family, to schools, churches, hospitals, and the military. They harbor the

the “genocidal continuum” (as we call it)


early “warning signs” (Charney 1991), the “priming” (as Hinton, ed., 2002 calls it), or

that push social consensus toward devaluing certain forms of human life and lifeways from the refusal
of social support and humane care to vulnerable “social parasites” (the nursing home elderly, “welfare
queens,” undocumented immigrants, drug addicts) to the militarization of everyday life (super-maximum-security
prisons, capital punishment; the technologies of heightened personal security, including the house gun and gated communities; and reversed
feelings of victimization).

Structural violence locks in social and environmental tension---culminates in


extinction and makes war inevitable
Tamás Szentes 8, Professor Emeritus at the Corvinus University of Budapest. “Globalisation and
prospects of the world society” 4/22/08
http://www.eadi.org/fileadmin/Documents/Events/exco/Glob.___prospects_-_jav..pdf

It’ s a common place that human society can survive and develop only in a lasting real peace. Without peace countries
cannot develop. Although since 1945 there has been no world war, but --numerous local wars took place, --terrorism has spread all over the
world, undermining security even in the most developed and powerful countries, -- arms race and militarisation have not ended
with the collapse of the Soviet bloc, but escalated and continued, extending also to weapons of mass destruction and misusing enormous
resources badly needed for development, --many “invisible wars” are suffered by the poor and oppressed people,
manifested in mass misery, poverty, unemployment, homelessness, starvation and malnutrition, epidemics
and poor health conditions, exploitation and oppression, racial and other discrimination, physical terror,
organised injustice, disguised forms of violence, the denial or regular infringement of the democratic rights of citizens, women, youth,
ethnic or religious minorities, etc., and last but not least, in the degradation of human environment , which means that --the
“war against Nature”, i.e. the disturbance of ecological balance, wasteful management of natural resources, and large-scale
pollution of our environment, is still going on, causing also losses and fatal dangers for human life. Behind global terrorism and
“invisible wars” we find striking international and intrasociety inequities and distorted development
patterns , which tend to generate social as well as international tensions , thus paving the way for unrest
and “visible” wars . It is a commonplace now that peace is not merely the absence of war. The prerequisites of a lasting
peace between and within societies involve not only - though, of course, necessarily - demilitarisation, but also a systematic
and gradual elimination of the roots of violence, of the causes of “invisible wars”, of the structural and
institutional bases of large-scale international and intra-society inequalities, exploitation and oppression. Peace
requires a process of social and national emancipation, a progressive, democratic transformation of societies and the world
bringing about equal rights and opportunities for all people, sovereign participation and mutually advantageous co-operation among nations. It
further requires a pluralistic democracy on global level with an appropriate system of proportional representation of the world society,
articulation of diverse interests and their peaceful reconciliation, by non-violent conflict management, and thus also a global governance with a
really global institutional system. Under the contemporary conditions of accelerating globalisation and deepening global interdependencies in
our world, peace is indivisible in both time and space. It cannot exist if reduced to a period only after or before war, and cannot
be
safeguarded in one part of the world when some others suffer visible or invisible wars . Thus, peace requires,
indeed, a new, demilitarised and democratic world order, which can provide equal opportunities for sustainable development. “Sustainability of
development” (both on national and world level) is often interpreted as an issue of environmental protection only and reduced to the need for
preserving the ecological balance and delivering the next generations not a destroyed Nature with overexhausted resources and polluted
environment. However, no ecological balance can be ensured, unless the deep international development
gap and intra-society inequalities are substantially reduced . Owing to global interdependencies there may exist hardly
any “zero-sum-games”, in which one can gain at the expense of others, but, instead, the “negative-sum-games” tend to predominate, in which
everybody must suffer, later or sooner, directly or indirectly, losses. Therefore, the actual question
is not about “sustainability
of development” but rather about the “sustainability of human life”, i.e. survival of mankind – because of
ecological imbalance and globalised terrorism. When Professor Louk de la Rive Box was the president of EADI, one day we had an exchange of
views on the state and future of development studies. We agreed that development studies are not any more restricted to the case of
underdeveloped countries, as the developed ones (as well as the former “socialist” countries) are also facing development problems, such as
those of structural and institutional (and even system-) transformation, requirements of changes in development patterns, and concerns about
natural environment. While all these are true, today I would dare say that besides (or even instead of) “development studies” we must speak
about and make “survival studies”. While the monetary, financial, and debt crises are cyclical, we live in an almost permanent
crisis of the world society, which is multidimensional in nature, involving not only economic but also socio-psychological, behavioural, cultural
and political aspects. The narrow-minded, election-oriented, selfish behaviour motivated by thirst for power
and wealth, which still characterise the political leadership almost all over the world, paves the way for the final, last
catastrophe. One cannot doubt, of course, that great many positive historical changes have also taken place in the world in the last
century. Such as decolonisation, transformation of socio-economic systems, democratisation of political life in some former fascist or
authoritarian states, institutionalisation of welfare policies in several countries, rise of international organisations and new forums for
negotiations, conflict management and cooperation, institutionalisation of international assistance programmes by multilateral agencies,
codification of human rights, and rights of sovereignty and democracy also on international level, collapse of the militarised Soviet bloc and
system-change3 in the countries concerned, the end of cold war, etc., to mention only a few. Nevertheless, the crisis of the world society has
extended and deepened, approaching to a point of bifurcation that necessarily puts an end to the present tendencies, either by the final
catastrophe or a common solution. Under
the circumstances provided by rapidly progressing science and
technological revolutions, human society cannot survive unless such profound intra-society and
international inequalities prevailing today are soon eliminated. Like a single spacecraft, the Earth can no
longer afford to have a 'crew' divided into two parts: the rich, privileged, wellfed, well-educated, on the one hand,
and the poor, deprived, starving, sick and uneducated, on the other. Dangerous 'zero-sum-games' (which mostly prove to be
“negative-sum-games”) can hardly be played any more by visible or invisible wars in the world society. Because of global interdependencies,
the apparent winner becomes also a loser. The real choice for the world society is between negative- and positive-sum-games: i.e. between, on
the one hand, continuation of visible and “invisible wars”, as long as this is possible at all, and, on the other, transformation of the world order
by demilitarisation and democratization. No ideological or terminological camouflage can conceal this real dilemma any more, which is to be
faced not in the distant future, by the next generations, but in the coming years, because of global terrorism soon having nuclear and other
mass destructive weapons, and also due to irreversible changes in natural environment.
Turns Environment
The everyday practices of militarism destroy the environment
Cuomo 96 (Chris, prof of women’s studies @ UGA, War is Not Just an Event: Reflections on the Significance of
Everyday Violence, Hypatia 11:4, Women and Violence, Autumn, pp. 30-45)
Drucker's view depends on sharp distinctions: between combatants and noncombatants, between war and peace. But both human and
nonhuman noncombatants are always harmed or otherwise affected by militarism, even when they are not directly harmed in battles. This
simple truth was captured in a popular Vietnam War era antiwar poster that read, "War is not healthy for children and other living things."
Because natural noncombatants are every- where; their destruction is necessary for war and for the
existence of military institutions, even when wars are not explicitly being fought. The ecological
realities of war, and of what it takes to be prepared for war in the contemporary world, are mind-boggling. To take nature at
all seriously entails acknowledging the effects of combat as well as the severe harm caused by everyday military
practices. In Scorched Earth: The Military's Assault on the Environment, William Thomas, a U.S. Navy veteran, illustrates the extent to which
the peacetime practices of military institutions damage natural environments and communities. Thomas argues
that even "peace" entails a dramatic and widespread war on nature , or as Joni Seager puts it, "The
environmental costs of militarized peace bear suspicious resemblance to the costs of war" (Thomas 1995, xi). All
told, including peacetime activities as well as the immense destruction caused by combat, military institutions probably
present the most dramatic threat to ecological well-being on the planet. The military is the largest
generator of hazardous waste in the United States, creating nearly a ton of toxic pollution every minute ,
and military analyst Jillian Skeel claims that, "Global military activity may be the largest worldwide polluter and
consumer of precious resources" (quoted in Thomas 1995, 5). A conventionally powered aircraft carrier consumes
150,000 gallons of fuel a day. In less than an hour's flight, a single jet launched from its flight deck consumes as
much fuel as a North American motorist bums in two years. One F-16 jet engine requires nearly four and a half tons of
scarce titanium, nickel, chromium, cobalt, and energy-intensive aluminum (Thomas 1995, 5), and nine percent of all the iron and steel used by
humans is consumed by the global military (Thomas 1995, 16). The United States Department of Defense generates 500,000
tons of toxins annually, more than the world's top five chemical companies combined. The military is
the biggest single source of environmental pollution in the United States. Of 338 citations issued by the United
States Environmental Protection Agency in 1989, three-quarters went to military installations (Thomas 1995, 17). The feminization,
commodification, and devaluation of nature helps create a reality in which its destruction in warfare is easily
justified. In imagining an ethic that addresses these realities, feminists cannot neglect the extent to which military ecocide is
connected, conceptually and practically, to transnational capitalism and other forms of human oppression and
exploitation. Virtually all of the world's thirty-five nuclear bomb test sites, as well as most radioactive dumps and uranium mines,
occupy Native lands (Thomas 1995, 6). Six multinationals control one-quarter of all United States defense con- tracts (Thomas 1995,
10), and two million dollars per minute is spent on the global military (Thomas 1995, 7). One could go on for volumes about the
effects of chemical and nuclear testing, military-industrial development and waste, and the disruption of
wildlife, habitats, communities, and lifestyles that are inescapably linked to military practices. There are
many conceptual and practical connections between military practices in which humans aim to kill and harm each other for some declared
"greater good," and nonmilitary practices in which we displace, destroy, or seriously modify nonhuman communities, species, and ecosystems
in the name of human interests. An early illustration of these connections was made by Rachel Carson in the first few pages of The Silent
Spring (1962), in which she described insecticides as the inadvertent offspring of World War II chemical weapons research. We can now also
trace ways in which insecticides were part of the Western-defined global corporatization of agriculture that helped kill off the small family
Military practices
farm and made the worldwide system of food production dependent on the likes of Dow Chemical and Monsanto.
are no different from other human practices that damage and irreparably modify nature. They are often a result
of cost-benefit analyses that pretend to weigh all likely outcomes yet do not consider nonhuman entities
except in terms of their use value for humans and they nearly always create unforeseeable effects for humans and nonhumans. In addition,
everyday military peacetime practices are actually more destructive than most other human activities ,
they are directly enacted by state power, and, becausethey function as unquestioned "givens," they enjoy a unique
near-immunity to enactments of moral reproach. It is worth noting the extent to which everyday military
activities remain largely unscrutinized by environmentalists , especially American environmentalists, largely because
fear allows us to be fooled into thinking that "national security" is an adequate excuse for "ecological
military mayhem" (Thomas 1995, 16). If environmental destruction is a necessary aspect of war and the
peacetime practices of military institutions, an analysis of war which includes its embeddedness in
peacetime militarism is necessary to address the environmental effects of war. Such a perspective must pay
adequate attention to what is required to prepare for war in a technological age, and how women and other Others are affected by the
realities of contemporary military institutions and practices .
Alternative/Framework
Alt Solvency – Warming
Rejection of the affirmative’s catastrophic rhetoric is crucial – alternative discursive
frames are key to solve warming
Foust 08, Christina R. FOUST, Assistant Professor in the Department of Human Communication Studies
at the University of Denver, ET AL., with William O. Murphy, Doctoral Student and Graduate Teaching
Instructor in the Department of Human Communication Studies at the University of Denver, and Chelsea
Stow, Doctoral Student and Graduate Teaching Instructor in the Department of Human Communication
Studies at the University of Denver, 2008 [“Global Warming and Apocalyptic Rhetoric: A Critical Frame
Analysis of US Popular and Elite Press Coverage from 1997-2007,” Paper Submitted to the Environmental
Communication Division of the National Communication Association Convention in San Diego,
November 20th, p. 24-25, Available Online at http://www.allacademic.com/meta/p260125_index.html,
Accessed 03-18-2009]
In conclusion, we hope to inspire more scholarship in the spirit of Moser and Dilling’s (2007) call for a greater inter-disciplinary conversation on
climate change. The methodological tool of frame analysis can help foster common ground between humanities scholars, social scientists, and
climate scientists, concerned about global warming. Frame analysis can also be a valuable tool in identifying the troubling aspects of how a
discourse evolves and is communicated—and in so doing, it can lead to more effective communication. Deconstructing the harmful
effects of an apocalyptic frame, we feel some responsibility to try to offer alternative frames which
might balance the need to communicate the urgency of climate change, without [end page 24] moving
people to denial and despair. We would like to see the press inspire more of a public dialogue on how
we can mitigate climate change, rather than encouraging readers to continue to be resigned to the
catastrophic telos. This does not mean that we should ignore the potentially devastating consequences of global warming (now and in
the future); but it does mean that we must begin a conversation about how to change our daily routines to
make things better. We believe that the press could promote greater human agency in the issue of
climate change, so that people do not become resigned to the telos of global warming. This includes
encouraging more personal and civic responsibility, rather than suggesting that experts will take care of it (or that we can do nothing to
mitigate the impacts of climate change). Journalists could acknowledge the expertise of scientists, balanced with an acknowledgement of the
power of common sense and morality— such a move may help avoid casting scientists as prophets. Through
a less tragic, more
productive framing of the issues of climate change, we may expand the common ground needed to build
a political will for dealing with climate change .
Alt Solvency – IR
The alternative’s rejection of security critically interrogates the affirmative’s theory of
geopolitics – paradigmatic criticism in the academy empirically has shaped policies
and can effectively foment dissent and spur social movements for change
Jones 99 – Professor International Politics @ Aberystwyth University (1999, Richard Wyn, Security,
Strategy, and Critical Theory, p. 155-163)

The central political task of the intellectuals is to aid in the construction of a counterhegemony and thus
undermine the prevailing patterns of discourse and interaction that make up the currently dominant
hegemony. This task is accomplished through educational activity, because, as Gramsci argues, “every relationship of
‘hegemony’ is necessarily a pedagogic relationship” (Gramsci 1971: 350). Discussing the relationship of the “philosophy of praxis” to political practice, Gramsci claims:
It [the theory] does not tend to leave the “simple” in their primitive philosophy of common sense, but rather to lead them to a higher conception of life. If it affirms the need for contact between intellectuals and “simple” it is not in
order to restrict scientific activity and preserve unity at the low level of the masses, but precisely in order to construct an intellectual-moral bloc which can make politically possible the intellectual progress of the mass and not only
of small intellectual groups. (Gramsci 1971: 332-333). According to Gramsci, this attempt to construct an alternative “intellectual-moral bloc” should take place under the auspices of the Communist Party – a body he described as
the “modern prince.” Just as Niccolo Machiavelli hoped to see a prince unite Italy, rid the country of foreign barbarians, and create a virtu-ous state, Gramsci believed that the modern price could lead the working class on its
journey toward its revolutionary destiny of an emancipated society (Gramsci 1971: 125-205). Gramsci’s relative optimism about the possibility of progressive theorists playing a constructive role in emancipatory political practice
was predicated on his belief in the existence of a universal class (a class whose emancipation would inevitably presage the emancipation of humanity itself) with revolutionary potential. It was a gradual loss of faith in this axiom
that led Horkheimer and Adorno to their extremely pessimistic prognosis about the possibilities of progressive social change. But does a loss of faith in the revolutionary vocation of the proletariat necessarily lead to the kind of
quietism ultimately embraced by the first generation of the Frankfurt School? The conflict that erupted in the 1960s between them and their more radical students suggests not. Indeed, contemporary critical theorists claim that
the deprivileging of the role of the proletariat in the struggle for emancipation is actually a positive move. Class remains a very important axis of domination in society, but it is not the only such axis (Fraser 1995). Nor is it valid to
reduce all other forms of domination – for example, in the case of gender – to class relations, as orthodox Marxists tend to do. To recognize these points is not only a first step toward the development of an analysis of forms of
exploitation and exclusion within society that is more attuned to social reality; it is also a realization that there are other forms of emancipatory politics than those associated with class conflict.1 This in turn suggests new
possibilities and problems for emancipatory theory. Furthermore, the abandonment of faith in revolutionary parties is also a positive development. The history of the European left during the twentieth century provides myriad
examples of the ways in which the fetishization of party organizations has led to bureaucratic immobility and the confusion of means with ends (see, for example, Salvadori 1990). The failure of the Bolshevik experiment illustrates
how disciplined, vanguard parties are an ideal vehicle for totalitarian domination (Serge 1984). Faith in the “infallible party” has obviously been the source of strength and comfort to many in this period and, as the experience of
the southern Wales coalfield demonstrates, has inspired brave and progressive behavior (see, for example, the account of support for the Spanish Republic in Francis 1984). But such parties have so often been the enemies of

History furnishes examples of progressive


emancipation that they should be treated with the utmost caution. Parties are necessary, but their fetishization is potentially disastrous.

developments that have been positively influenced by organic intellectuals operating outside the
bounds of a particular party structure (G. Williams 1984). Some of these developments have occurred in the
particularly intractable realm of security . These examples may be considered as “resources of hope” for
critical security studies (R. Williams 1989). They illustrate that ideas are important or, more correctly, that change is the product of
the dialectical interaction of ideas and material reality . One clear security-related example of the role of critical thinking and critical thinkers in
aiding and abetting progressive social change is the experience of the peace movement of the 1980s . At that time the ideas of dissident defense

intellectuals (the “alternative defense” school) encouraged and drew strength from peace activism . Together they had an
effect not only on short-term policy but on the dominant discourses of strategy and security, a far more
important result in the long run. The synergy between critical security intellectuals and critical social movements and the potential influence of both working in tandem
can be witnessed particularly clearly in the fate of common security . As Thomas Risse-Kappen points out, the term “common security” originated in the contribution of
peace researchers to the German security debate of the 1970s (Risse-Kappen 1994: 186ff.); it was subsequently popularized by the Palme Commission report (Independent Commission on Disarmament and Security Issues 1982).

Initially, mainstream defense intellectuals dismissed the concept as hopelessly idealistic ; it certainly had no place in their
allegedly hardheaded and realist view of the world. However, notions of common security were taken up by a number of different

intellectuals communities, including the liberal arms control community in the United States, Western European peace researchers, security specialists in the center-left political parties of Western
Europe, and Soviet “institutchiks” – members of the influential policy institutes in the Soviet Union such as the United States of America and Canada Institute (Landau 1996: 52-54; Risse-Kappen 1994: 196-200; Kaldor 1995; Spencer

1995). These communities were subsequently able to take advantage of public pressure exerted through social movements in
order to gain broader acceptance for common security. In Germany, for example, “in response to social movement pressure, German social organizations such as
churches and trade unions quickly supported the ideas promoted by peace researchers and the SPD” (Risse-Kappen 1994: 207). Similar pressures even had an effect on the

Reagan administration. As Risse-Kappen notes: When the Reagan administration brought hard-liners into power, the US arms control community was removed from policy influence. It was
the American peace movement and what became known as the “freeze campaign” that revived the
arms control process together with pressure from the European allies. (Risse-Kappen 1994: 205; also Cortright 1993: 90-110). Although it would be difficult to sustain a claim that the combination of critical
movements and intellectuals persuaded the Reagan government to adopt the rhetoric and substance of common security in its entirety, it is clear that it did at least have a substantial impact

on ameliorating U.S. behavior. The most dramatic and certainly the most unexpected impact of alternative defense ideas was felt in the
Soviet Union. Through various East-West links, which included arms control institutions, Pugwash conferences, interparty contacts, and even direct personal links, a coterie of Soviet
policy analysts and advisers were drawn toward common security and such attendant notions as “nonoffensive defense” (these links are detailed in
Evangelista 1995; Kaldor 1995; Checkel 1993; Risse-Kappen 1994; Landau 1996 and Spencer 1995 concentrate on the role of the Pugwash conferences). This group, including Palme Commission member Georgii
then
Arbatov, Pugwash attendee Andrei Kokoshin , and Sergei Karaganov, a senior adviser who was in regular contact with the Western peace researchers Anders Boserup and Lutz Unterseher (Risse-Kappen 1994: 203),

influenced Soviet leader Mikhail Gorbachev. Gorbachev’s subsequent championing of common security may be attributed to several factors. It is clear, for example, that new Soviet
leadership had a strong interest in alleviating tensions in East-West relations in order to facilitate much-
needed domestic reforms (“the interaction of ideas and material reality”). But what is significant is that the Soviets’ commitment to common
security led to significant changes in force sizes and postures. These in turn aided in the winding down of
the Cold War, the end of Soviet domination over Eastern Europe, and even the collapse of Russian
control over much of the territory of the former Soviet Union . At the present time, in marked contrast to the situation in the early 1980s, common security is
part of the common sense of security discourse. As MccGwire points out, the North Atlantic Treaty Organization (NATO) (a common defense pact) is using the rhetoric of common security in order to justify its expansion into

This points to an interesting and potentially important aspect of the impact of ideas on
Eastern Europe (MccGwire 1997).

politics. As concepts such as common security, and collective security before it (Claude 1984: 223-260), are adopted by governments and military
services, they inevitably become somewhat debased. The hope is that enough of the residual meaning
can survive to shift the parameters of the debate in a potentially progressive direction. Moreover, the adoption of the
concept of common security by official circles provides critics with a useful tool for (immanently) critiquing aspects of security policy (as MccGwire 1997 demonsrates in relation to NATO expansion). The example of common

critical intellectuals can be politically engaged and play a role – a significant one at that – in
security is highly instructive. First, it indicates that

making the world a better and safer place . Second, it points to potential future addressees for critical
international theory in general, and critical security studies in particular . Third, it also underlines the role of
ideas in the evolution in society. CRITICAL SECURITY STUDIES AND THE THEORY-PRACTICE NEXUS Although most proponents of critical security studies reject aspects of Gramsci’s theory of
organic intellectuals, in particular his exclusive concentration on class and his emphasis on the guiding role of the party, the desire for engagement and relevance must remain at the heart of their project. The example of the peace
movement suggests that critical theorists can still play the role of organic intellectuals and that this organic relationship need not confine itself to a single class; it can involve alignment with different coalitions of social movements

Said captures this broader orientation when


that campaign on an issue or a series of issues pertinent to the struggle for emancipation (Shaw 1994b; R. Walker 1994). Edward

he suggests that critical intellectuals “are always tied to and ought to remain an organic part of an
ongoing experience in society: of the poor, the disadvantaged, the voiceless, the unrepresented, the powerless” (Said 1994: 84). In the specific case of critical security studies, this
means placing the experience of those men and women and communities for whom the present world
order is a cause of insecurity rather than security at the center of the agenda and making suffering
humanity rather than raison d’etat the prism through which problems are viewed . Here the project stands full-square within the
critical theory tradition. If “all theory is for someone and for some purpose,” then critical security studies is for “the

voiceless, the unrepresented, the powerless,” and its purpose is their emancipation . The theoretical
implications of this orientation have already been discussed in the previous chapters. They involve a fundamental reconceptualization of security
with a shift in referent object and a broadening of the range of issues considered as a legitimate part of the discourse. They also
involve a reconceptualization of strategy within this expanded notion of security. But the question remains at the conceptual level of how these

alternative types of theorizing – even if they are self-consciously aligned to the practices of critical or new social movements, such as peace activism, the struggle for human rights, and the
survival of minority cultures – can become “a force for the direction of action .” Again, Gramsci’s work is insightful. In the Prison Notebooks, Gramsci advances a
sophisticated analysis of how dominant discourses play a vital role in upholding particular political and economic orders, or, in Gramsci’s terminology, “historic blocs” (Gramsci 1971: 323-377). Gramsci adopted Machiavelli’s view of
power as a centaur, ahlf man, half beast: a mixture of consent and coercion. Consent is produced and reproduced by a ruling hegemony that holds sway through civil society and takes on the status of common sense; it becomes
subconsciously accepted and even regarded as beyond question. Obviously, for Gramsci, there is nothing immutable about the values that permeate society; they can and do change. In the social realm, ideas and institutions that
were once seen as natural and beyond question (i.e., commonsensical) in the West, such as feudalism and slavery, are now seen as anachronistic, unjust, and unacceptable. In Marx’s well-worn phrase, “All that is solid melts into
the air.” Gramsci’s intention is to harness this potential for change and ensure that it moves in the direction of emancipation. To do this he suggests a strategy of a “war of position” (Gramsci 1971: 229-239). Gramsci argues that in

states with developed civil societies, such as those in Western liberal democracies, any successful attempt at progressive social change requires a slow, incremental, even
molecular, struggle to break down the prevailing hegemony and construct an alternative
counterhegemony to take its place. Organic intellectuals have a crucial role to play in this process by helping to
undermine the “natural,” “commonsense,” internalized nature of the status quo. This in turn helps create political space within
which alternative conceptions of politics can be developed and new historic blocs created. I contend that Gramsci’s strategy of a war of position suggests an
appropriate model for proponents of critical security studies to adopt in relating their theorizing to political practice. THE TASKS OF CRITICAL SECURITY STUDIES If the project of critical security

studies is conceived in terms of war of position, then the main task of those intellectuals who align
themselves with the enterprise is to attempt to undermine the prevailing hegemonic security discourse.
This may be accomplished by utilizing specialist information and expertise to engage in an immanent critique of the
prevailing security regimes, that is, comparing the justifications of those regimes with actual outcomes.
When this is attempted in the security field, the prevailing structures and regimes are found to fail
grievously on their own terms. Such an approach also involves challenging the pronouncements of those
intellectuals, traditional or organic, whose views serve to legitimate, and hence reproduce, the prevailing world
order. This challenge entails teasing out the often subconscious and certainly unexamined assumptions that
underlie their arguments while drawing attention to the normative viewpoints that are smuggled into
mainstream thinking about security behind its positivist façade. In this sense, proponents of critical security
studies approximate to Foucault’s notion of “specific intellectuals” who use their expert knowledge to
challenge the prevailing “regime of truth” (Foucault 1980: 132). However, critical theorists might wish to reformulate this sentiment along more familiar Quaker lines of “speaking
truth to power” (this sentiment is also central to Said 1994) or even along the eisteddfod lines of speaking “truth against the world.” Of course, traditional strategists can, and indeed do, sometimes claim a similar role. Colin S. Gray,
for example, states that “strategists must be prepared to ‘speak truth to power’” (Gray 1982a: 193). But the difference between Gray and proponents of critical security studies is that, whereas the former seeks to influence

critical
policymakers in particular directions without questioning the basis of their power, the latter aim at a thoroughgoing critique of all that traditional security studies has taken for granted. Furthermore,

theorists base their critique on the presupposition, elegantly stated by Adorno, that “the need to lend
suffering a voice is the precondition of all truth” (cited in Jameson 1990: 66). The aim of critical security studies in attempting to undermine the prevailing orthodoxy is
ultimately educational. As Gramsci notes, “every relationship of ‘hegemony’ is necessarily a pedagogic relationship” (Gramsci 1971: 350; see also the discussion of critical pedagogy in Neufeld 1995: 116-121). Thus, by

criticizing the hegemonic discourse and advancing alternative conceptions of security based on different understandings of
human potentialities, the approach is simultaneously playing apart in eroding the legitimacy of the ruling historic

bloc and contributing to the development of a counterhegemonic position . There are a number of avenues of avenues open to critical
security specialists in pursuing this educational strategy. As teachers, they can try to foster and encourage skepticism toward accepted

wisdom and open minds to other possibilities. They can also take advantage of the seemingly
unquenchable thirst of the media for instant pundistry to forward alternative views onto a broader
stage. Nancy Fraser argues: “As teachers, we try to foster an emergent pedagogical counterculture …. As
critical public intellectuals we try to inject our perspectives into whatever cultural or political public
spheres we have access to” (Fraser 1989: 11). Perhaps significantly, support for this type of emancipatory strategy can even be found in the work of the ultrapessimistic Adorno, who
argues: In the history of civilization there have been not a few instances when delusions were healed
not by focused propaganda, but, in the final analysis, because scholars, with their unobtrusive yet insistent work
habits, studied what lay at the root of the delusion. (cited in Kellner 1992: vii) Such “unobtrusive yet insistent work” does not in itself create the social change to which
Adorno alludes. The conceptual and the practical dangers of collapsing practice into theory must be guarded

against. Rather, through their educational activities, proponent of critical security studies should aim to
provide support for those social movements that promote emancipatory social change . By providing a
critique of the prevailing order and legitimating alternative views, critical theorists can perform a
valuable role in supporting the struggles of social movements. That said, the role of theorists is not to direct and instruct those movements with which
they are aligned; instead, the relationship is reciprocal. The experience of the European, North American, and Antipodean peace movements of the 1980s shows how

influential social movements can become when their efforts are harnessed to the intellectual and
educational activity of critical thinkers. For example, in his account of New Zealand’s antinuclear stance in the 1980s, Michael C. Pugh cites the
importance of the visits of critical intellectuals such as Helen Caldicott and Richard Falk in changing the country’s
political climate and encouraging the growth of the antinuclear movement (Pugh 1989: 108; see also COrtright 1993: 5-13). In the 1980s
peace movements and critical intellectuals interested in issues of security and strategy drew strength and succor from each other’s efforts. If such critical social movements do not exist, then this creates obvious difficulties for the
critical theorist. But even under these circumstances, the theorist need not abandon all hope of an eventual orientation toward practice. Once again, the peace movement of the 1980s provides evidence of the possibilities. At that
time, the movement benefited from the intellectual work undertaken in the lean years of the peace movement in the late 1970s. Some of the theories and concepts developed then, such as common security and nonoffensive
defense, were eventually taken up even in the Kremlin and played a significant role in defusing the second Cold War. Those ideas developed in the 1970s can be seen in Adornian terms of the a “message in a bottle,” but in this
case, contra Adorno’s expectations, they were picked up and used to support a program of emancipatory political practice. Obviously, one would be naïve to understate the difficulties facing those attempting to develop alternative
critical approaches within academia. Some of these problems have been alluded to already and involve the structural constraints of academic life itself. Said argues that many problems are caused by what he describes as the

Academics are now so constrained by the requirements of job security and


growing “professionalisation” of academic life (Said 1994: 49-62).

marketability that they are extremely risk-averse. It pays – in all senses – to stick with the crowd and avoid the
exposed limb by following the prevalent disciplinary preoccupations , publish in certain prescribed journals, and so on. The result is
the navel gazing so prevalent in the study of international relations and the seeming inability of security
specialists to deal with the changes brought about by the end of the Cold War (Kristensen 1997 highlights
the search of U.S. nuclear planners for “new targets for old weapons ”). And, of course, the pressures for conformism
are heightened in the field of security studies when governments have a very real interest in
marginalizing dissent. Nevertheless, opportunities for critical thinking do exist, and this thinking can connect
with the practices of social movements and become a “force for the direction of action. ” The experience of the 1980s, when,
in the depths of the second Cold War, critical thinkers risked demonization and in some countries far worse in order to challenge received wisdom, thus arguably playing a crucial role in the very survival of the human race, should
act as both an inspiration and a challenge to critical security studies.
Alt Solvency – Asia
The alternative critically interrogates the theoretical foundations of the Asia threat
claims of the 1AC which are the very knowledges that drive instability – emancipatory
IR theory with an improved quality of analysis can rewrite the norms of modern
statehood towards a more sustainable peace in East Asia
Wirth 15 – analyst at Griffin University Asia Institute
(Christian, “‘Power’ and ‘stability’ in the China–Japan–South Korea regional security complex,” The
Pacific Review, Volume 28, Issue 4, pgs. 553-575)

The salience of very similar disputes over wartime history and territorial delimitation in all dyads between China, Japan
and South Korea leads to several conclusions about Northeast Asian international relations and their
theorizing.¶ First, parsimonious explanations based on either regime type or material power status are
inadequate. This is not to claim that differentials in material power are irrelevant. In contrast to unidirectional, material
conceptions of power, however, this has not necessarily made weaker states give in to demands by stronger states. To the contrary,
due to their anxieties weaker actors have often displayed assertive behaviour and successfully prevailed while stronger, more
confident states tended to be more inclined to de-escalate conflicts. The April 2006 confrontation over
Dokdo/Takeshima between Japan and South Korea is a good example. ¶ Second, amity–enmity patterns divide the
RSC along different lines depending on the area concerned. Rising China is seen as a great threat in Japan but as
relatively minor concern in South Korea. Despite its decline, Japan is perceived as a threat in South Korea and even in China.
When it comes to territorial disputes it is not only Japan being pitted against China. For South Korea, Japanese claims are extremely worrisome
while Chinese claims remain a concern. Geopolitics, the area, which is due to the inherent uncertainty that comes with the
aspiration to forecast long-term developments most dividing, reveals that strategic calculations diverge from conventional
views.¶ Overall, the constellation of interlocking interests and threat perceptions in Northeast Asia points to the
possibility that the three competitive and antagonistic dyadic relations combined may produce cooperative or cancel out
negative effects. Such an understanding brings agency – particularly Asian leaders' agency – back in . The
politics surrounding the project of a China–Japan–Korea FTA is an illustrative example for how competitive initiatives in the bilateral dimension
spur trilateral cooperation (Japan Times 2012b). The avoidance of traditional security issues in trilateral fora, rather
than vindicating the preponderance of material power, points to the different dimensions of power at play. It shows the
strength of productive power manifest in the salience of national identities and political legitimation. The more issues touch upon
and question national myths – history textbooks and military affairs are prime examples – the more they become securitized
and channelled into the tunnel views of bilateralism. This is not to argue that multilateral institutions have the power to
mitigate interstate rivalry. Therefore, the fact that trilateral meetings have in the face of bilateral tensions been downgraded to the sub-
ministerial level in 2013 does not question the present argument. Thatthe US ‘rebalancing’ towards the Asia–Pacific
reinforces bilateral military alliances and indirectly affects trilateralism further reinforces my point.¶
Given the rather bleak picture, one may also argue that classical realist approach provides sufficient means for the explanation of the
described phenomena. The mere fact that there exist in the present case at present time tendencies towards an anarchic world of self-help as it
is envisioned by realists, however, does
not justify this paradigm's claim to the universal applicability of unchanging laws (Wendt
1999). More importantly, from a rationalist standpoint it is hard to understand , for instance, why Japanese leaders
are willing to significantly curtail sovereignty in relation to the United States while displaying an
uncompromising stance towards seemingly minor issues concerning features at sea with China and South
Korea. Not the least, the conflation and even convergence of realist (rational interest) with liberal
(normative value) arguments such as in the case of the power shift and arc of freedom and prosperity propositions are
irreconcilable with the respective theoretical assumptions.¶ Although this study applied a conventional, state-centric
perspective on bilateral relations, the striking similarities of contentious issues in all three dyads and the unequal attention that is given to them
in the international relations realm demonstrates how productive power works. The strong tendency of looking for
differences that confirm the existing enmity patterns while turning blind eyes on commonalities –
cooperative or conflictual – reproduces bilateralism across the Asia–Pacific. The securitization of the maritime
sphere spurred by anxieties over the freedom of navigation in the sea lanes of communication is a case on point.
China, Japan and South Korea, all heavily dependent on Middle Eastern oil, sit in the same boat.
Nevertheless, governments and academics, rather than recognize how their countries' interests
converge, super-size potential rivalries.¶ This reading of political dynamics in Northeast Asia is in line with
explanations that see international politics mainly driven by domestic struggles for political legitimacy
(Rozman and Lee 2006). The making and active use of national myths by conservative elites in China, Japan and
South Korea often align well with views on Asia–Pacific security in the United States . The US restraining of
allies in military crisis management and nationalist rhetoric is undisputable. Such actions that influence interaction change (Gilpin 1983),
however, should not distract from more fundamental influence that often produces basic tensions in the first place .
It is far more convenient for Northeast Asian elites to interact with Washington than with their
neighbours. Rather than to question official narratives that legitimize their political systems, foci on
bilateral interaction with the US reaffirm long-held beliefs. This holds true regardless of whether relations are
cooperative, competitive or even conflicting as in the case of North Korea. Northeast Asian preferences for bilateralism are
welcome in Washington as this allows for the continuation of informal and formal alliance relationships and
assuages anxieties of being excluded from East Asia. ¶ Despite the repeated commitment to alliances and
the general view on the desirability , with exceptions, of US presence, conservative elites both in Northeast Asia and
in Washington tend to become nervous when substantial matters of alliance relationships are discussed among
the broader public. President Kim's sunshine policy of engaging the North met with great suspicion among conservatives both in Seoul and
Washington (Chung 2008). When
the South Korean President Roh in 2005 and the Japanese Prime Minister
Hatoyama in 2009 suggested recalibrating their respective alliance relationships with Washington,
anxiety engulfed conservative policy and academic circles on both sides of the Pacific (Choe 2005; Kang 2006;
Roos 2009). These fears have proven unfounded.¶ The argument put forward here is no claim to comprehensive explanation nor
do I suggest that China, Japan and South Korea are the same. Differences in capacities for adaptation are crucial and vary according to the
natures of political systems. It is in this regard that the
distribution of power within states, for instance, in the form of the
centralization of political and economic institutions should be further explored . What the present study shows is that for
states, politicians, government officials and academics to be powerful means to be able to maintain long-held
beliefs in spite of rapidly changing environments . Deutsch (1966, p. 111) was, therefore, right in stating that power is
‘the ability to afford not to learn’. His warning that power when carried to extremes ‘becomes blind, and the
person or organization becomes insensitive to the present, and is driven, like a bullet or torpedo, wholly by its
past’, should remind us that arguing for stability, due to powerful actors' ability to talk instead of listening
(Deutsch 1966, p. 111), may well be the very cause of instability . This holds true for the national as well as the international
level. Thus, the meaning of power changes depending on the time horizon applied. In the short-term, compulsory and institutional, that is,
material power may secure an actors' identity by forestalling views that contradict established self-conceptions. The longer the time horizon,
however, the more there is a need to adopt reflexive concepts of power because a powerful actor's inherent blindness towards profound
changes in its environment will eventually lead to decline. Ironically,
it is fear rooted in the uncertain quest to project
developments into the distant future such as apparent in the securitizing effect of geopolitical thought that
leads to short-sighted fixation on material power and forecloses potentially more sustainable alternate
courses of action. To look at Northeast Asia as an RSC and to define the concept of power appropriate for
the specific issue in question, therefore, improves the quality of analyses by pointing to the issue of
change and adaptation.
Epistemology First
Interrogating epistemology is key to successful IR theory
Reus-Smit 12 – Professor of International Relations at the European University Institute, Florence
(Christian, “International Relations, Irrelevant? Don’t Blame Theory”, Millennium - Journal of
International Studies June 2012 vol. 40 no. 3 525-540)

However widespread it might be, the notion that IR’s lack of practical relevance stems from excessive theorising
rests more on vigorous assertion than weighty evidence . As noted above, we lack good data on the field’s
practical relevance, and the difficulties establishing appropriate measures are all too apparent in the fraught attempts by several
governments to quantify the impact of the humanities and social sciences more generally. Beyond this, though, we lack any credible
evidence that any fluctuations in the field’s relevance are due to more or less high theory. We hear that policymakers
complain of not being able to understand or apply much that appears in our leading journals, but it is unclear why we should be any more
concerned about this than physicists or economists, who take theory, even high theory, to be the bedrock of advancement in knowledge.
Moreover, there is now a wealth of research, inside and outside IR, that shows that policy communities are not
open epistemic or cognitive realms, simply awaiting well-communicated, non-jargonistic knowledge – they are
bureaucracies, deeply susceptible to groupthink , that filter information through their own intersubjective frames. 10 Beyond
this, however, there are good reasons to believe that precisely the reverse of the theory versus relevance thesis might
be true; that theoretical inquiry may be a necessary prerequisite for the generation of practically relevant
knowledge. I will focus here on the value of metatheory, as this attracts most contemporary criticism and would appear the most difficult of
theoretical forms to defend. Metatheories take other theories as their subject. Indeed, their precepts establish the conditions of possibility for
second-order theories. In general, metatheories divide into three broad categories: epistemology, ontology and meta-ethics. The first concerns
the nature, validity and acquisition of knowledge; the second, the nature of being (what can be said to exist, how things might be categorised
and how they stand in relation to one another); and the third, the nature of right and wrong, what constitutes moral argument, and how moral
arguments might be sustained. Second-order theories are constructed within, and on the basis of, assumptions formulated at the
metatheoretical level. Epistemological assumptions about what constitutes legitimate knowledge and how it is legitimately acquired
delimit the questions we ask and the kinds of information we can enlist in answering them . 11 Can social
scientists ask normative questions? Is literature a valid source of social-scientific knowledge? Ontological assumptions
about the nature and distinctiveness of the social universe affect not only what we ‘see’ but also how we order what we
see; how we relate the material to the ideational, agents to structures, interests to beliefs, and so on. If we assume, for example, that
individuals are rational actors, engaged in the efficient pursuit of primarily material interests, then
phenomena such as faith-motivated politics will remain at the far periphery of our vision . 12 Lastly, meta-ethical
assumptions about the nature of the good, and about what constitutes a valid moral argument, frame how we reason about concrete ethical
problems. Both deontology and consequentialism are meta-ethical positions, operationalised, for example, in the differing arguments of
Most scholars would acknowledge the background, structuring role that
Charles Beitz and Peter Singer on global distributive justice. 13
metatheory plays, but argue that we can take our metatheoretical assumptions off the shelf, get on with the
serious business of research and leave explicit metatheoretical reflection and debate to the philosophers. If practical
relevance is one of our concerns, however, there are several reasons why this is misguided. Firstly, whether IR is
practically relevant depends, in large measure, on the kinds of questions that animate our research . I am not
referring here to the commonly held notion that we should be addressing questions that practitioners want answered. Indeed, our work
will at times be most relevant when we pursue questions that policymakers and others would prefer left
buried. My point is a different one, which I return to in greater detail below. It is sufficient to note here that being practically
relevant involves asking questions of practice; not just retrospective questions about past practices – their
nature, sources and consequences – but prospective questions about what human agents should do. As I have argued elsewhere, being
practically relevant means asking questions of how we , ourselves, or some other actors (states, policymakers, citizens,
NGOs, IOs, etc.) should act. 14 Yet our ability, nay willingness, to ask such questions is determined by the
metatheoretical assumptions that structure our research and arguments . This is partly an issue of ontology – what
we see affects how we understand the conditions of action , rendering some practices possible or impossible, mandatory
or beyond the pale. If, for example, we think that political change is driven by material forces, then we are
unlikely to see communicative practices of argument and persuasion as potentially successful sources of change .
More than this, though, it is also an issue of epistemology. If we assume that the proper domain of IR as a social science
is the acquisition of empirically verifiable knowledge, then we will struggle to comprehend , let alone answer,
normative questions of how we should act. We will either reduce ‘ought’ questions to ‘is’ questions, or place them off the
agenda altogether. 15 Our metatheoretical assumptions thus determine the macro-orientation of IR towards
questions of practice, directly affecting the field’s practical relevance . Secondly, metatheoretical revolutions
license new second-order theoretical and analytical possibilities while foreclosing others, directly affecting those
forms of scholarship widely considered most practically relevant . The rise of analytical eclecticism illustrates this. As
noted above, Katzenstein and Sil’s call for a pragmatic approach to the study of world politics, one that addresses real-world problematics by
combining insights from diverse research traditions, resonates with the mood of much of the field, especially within the American mainstream.
Epistemological and ontological debates are widely considered irresolvable dead ends , grand theorising is
unfashionable, and gladiatorial contests between rival paradigms appear, increasingly, as unimaginative rituals. Boredom and fatigue are partly
responsible for this new mood, but something deeper is at work. Twenty-five years ago, Sil and Katzenstein’s call would have fallen on deaf
ears; the
neo-neo debate that preoccupied the American mainstream occurred within a metatheoretical
consensus, one that combined a neo-positivist epistemology with a rationalist ontology. This singular metatheoretical
framework defined the rules of the game ; analytical eclecticism was unimaginable. The Third Debate of the 1980s and early
1990s destabilised all of this; not because American IR scholars converted in their droves to critical theory or poststructuralism (far from it), but
because metatheoretical absolutism became less and less tenable. The anti-foundationalist critique of the idea that there is any single measure
of truth did not produce a wave of relativism, but it did generate a widespread sense that battles on the terrain of epistemology were
unwinnable. Similarly, the Third Debate emphasis on identity politics and cultural particularity, which later found expression in constructivism,
did not vanquish rationalism. It did, however, establish a more pluralistic, if nevertheless heated, debate about ontology, a terrain on which
many scholars felt more comfortable than that of epistemology. One can plausibly argue, therefore, that the
metatheoretical
struggles of the Third Debate created a space for – even made possible – the rise of analytical eclecticism and its
aversion to metatheoretical absolutes, a principal benefit of which is said to be greater practical relevance. Lastly, most of us
would agree that for our research to be practically relevant, it has to be good – it has to be the product of sound
inquiry, and our conclusions have to be plausible. The pluralists among us would also agree that different research questions require different
methods of inquiry and strategies of argument. Yet across this diversity there are several practices widely recognised
as essential to good research. Among these are clarity of purpose, logical coherence, engagement with alternative arguments and
the provision of good reasons (empirical evidence, corroborating arguments textual interpretations, etc.). Less often noted, however, is
the importance of metatheoretical reflexivity. If our epistemological assumptions affect the questions
we ask, then being conscious of these assumptions is necessary to ensure that we are not fencing off
questions of importance, and that if we are, we can justify our choices. Likewise, if our ontological assumptions affect
how we see the social universe, determining what is in or outside our field of vision, then reflecting on these
assumptions can prevent us being blind to [ignorant of] things that matter. A similar argument applies to our meta-
ethical assumptions. Indeed, if deontology and consequentialism are both meta-ethical positions, as I suggested earlier, then reflecting on
our choice of one or other position is part and parcel of weighing rival ethical arguments (on issues as diverse
as global poverty and human rights). Finally, our epistemological, ontological and meta-ethical assumptions are not
metatheoretical silos; assumptions we make in one have a tendency to shape those we make in another .
The oft-heard refrain that ‘if we can’t measure it, it doesn’t matter’ is an unfortunate example of epistemology supervening on ontology,
something that metatheoretical reflexivity can help guard against. In sum, like clarity, coherence, consideration
of alternative
arguments and the provision of good reasons, metatheoretical reflexivity is part of keeping us honest,
making it practically relevant despite its abstraction.

Critique must come before policymaking


Korous 97, George Korous, Philosophy at Emory, 1997, Become What You Are, pp 22-25
The thought that inhabits critique is not bent on achieving quick and efficient solutions . Nor is critique
simply a means for some eventual action; for Foucault, the distinction between theory and practice is
shallow, as thought is a practice, and practice is always informed by thought . Yet despite the close relationship
between thought and action, the practice of critique operates according to a mode of thought quite different
from the calculative thinking that drives technological practices. This other mode of thinking is what Heidegger would call
"meditative thought." Meditative thought is characterized by its disengagement from the technological imperative
to react. 14 This is not to say that meditative thought does not result in action, but rather, thought is not
reducible to action, as if its only function were to usher in a solution : "thinking does not become action
only because some effect issues from it or because it is applied" (LH 217). Thought has value in and of
itself. It allows us to take stock of our ontological situation. As Foucault explains, Thought is not what inhabits
a certain conduct and gives it its meaning; rather it is what allows one to step back away from this way
of acting or reacting, to present it to oneself as an object of thought and question it as to its meaning,
its conditions, its goals. Thought is freedom in relation to what one does, the motion by which one
detaches oneself from it, and establishes it as an object, and reflects on it as a problem . (PPP 388)15 Heidegger
echoes these sentiments when he writes, "Reflection is the courage to make the truth of our own presuppositions
and the realm of our own goals into the things that most deserve to be called into question. ,16 The
calculative mode of engaging the world is forever asking , "What should I do?"; it is bent on producing
immediate and practical solutions. Problems take on an urgency that demand quick action, and
calculative thought eschews the task of thinking as a luxury that cannot be afforded. But as Heidegger points
out, "All attempts to reckon existing reality morphologically, psychologically, in terms of decline and loss,
in terms of fate, catastrophe and destruction, are merely technological behavior. That behavior operates
through the device of the enumerating of symptoms whose standing-reserve can be increased to
infinity and always varied anew" (T 48).The call for action already operates with the understanding that
the world is an ordered whole that can be manipulated as necessary to avoid immanent danger. As
long as reality is problematized as one crisis after the other, action will always beat out thought as the
preferred mode of engagement. For Heidegger and Foucault both, this knee jerk sense of action is systemically
destined to produce nothing but more of the same . By failing to engage problems at the level of
thought, that is, the level at which the problem is understood as a problem for thought, the imperative
to act merely operates on superficial features of reality, applying band-aids to wounds when the real
injury is festering way beneath the surface. The first step in overcoming the calculative understanding of reality is to
recognize that it is only one understanding among many. This is much more difficult than it might sound. First of all, the calculative mode of
revealing the world, Enframing, is something that conceals itself in the process of revealing the world (QT 27). The mode of revealing is so
pervasive that it is invisible to us, unless we reflect on it. When we are mired in the concerns of the everyday, Enframing is not encountered, it
is only lived. That is, as someone thinking technologically, reality reveals itself to me as a series of objects. I am attuned to that objectness when
I am engaging with the world. Precisely because Enframing is not an object, but a mode of revealing, it itself will not show up within my
observational field. In order for me to confront technological thought for what it is, a way of revealing, I have to be prepared to
momentarily suspend my calculative mode of thinking and pursue ontological questions . Second, the
continued successes of technological thought blinds us to the fact that it is only an interpretation of
reality and not reality in itself. As Heidegger warns, "The approaching tide of technological revolution in the
atomic age could so captivate, bewitch, dazzle, and beguile man that calculative thinking may someday
come to be accepted and practiced as the only way of thinking “(DT 56). For every time that a scientific theory pans
out, or technological planning achieves desired ends, we are less capable of viewing technology as only one of many different ways to reveal
the world. Heidegger is not arguing that science is false or useless. In fact, he recognizes that technological representations of reality often do
allow us to make correct determinations about the world:"In a similar way the unconcealment in accordance with which nature presents itself
as a ceculable complex of the effects of forces can indeed permit correct determinations; but precisely through these successes the danger can
remain that in the midst of all that is correct the true will withdraw" (QT 26).While it might be the case that a river that can yield a calculable
amount of hydropower, this does not mean that the river is, in its essence, a source of energy. But for every power plant built on a river it
becomes increasingly more difficult to appreciate that rivers are not primarily stockpiles of potential energy waiting to be unleashed.
Discourse First
Only incorporation of representations can make sense of political reality
Jourde 6 – Ph.D., Political Science, University of Wisconsin-Madison, M.A., Political Science, University
of Wisconsin-Madison, B.Sc., Political Science, Université de Montréal (Cedric, 2006, “1995 Hegemony or
Empire?: The redefinition of US Power under George W Bush,” Ed. David and Grondin p. 182-3)

Relations between states are, at least in part, constructed upon representations. Representations are
interpretative prisms through which decision-makers make sense of a political reality, through which
they define and assign a subjective value to the other states and non-state actors of the international
system, and through which they determine what are significant international political issues.2 For instance,
officials of a given state will represent other states as 'allies', 'rivals', or simply 'insignificant', thus assigning a subjective value to these states.
Such subjective categorizations often derive from representations of these states' domestic politics, which can for instance be perceived as
'unstable*, 'prosperous', or 'ethnically divided'. It
must be clear that representations are not objective or truthful
depictions of reality; rather they are subjective and political ways of seeing the world, making certain
things 'seen' by and significant for an actor while making other things 'unseen' and 'insignificant' .3 In other
words, they are founded on each actor's and group of actors' cognitive, cultural-social, and emotional
standpoints. Being fundamentally political, representations are the object of tense struggles and
tensions, as some actors or groups of actors can impose on others their own representations of the
world, of what they consider to be appropriate political orders, or appropriate economic relations, while others may in turn accept, subvert
or contest these representations. Representations of a foreign political reality influence how decision-making
actors will act upon that reality. In other words, as subjective and politically infused interpretations of reality,
representations constrain and enable the policies that decision-makers will adopt vis-a-vis other states ;
they limit the courses of action that are politically thinkable and imaginable, making certain policies
conceivable while relegating other policies to the realm of the unthinkable .4 Accordingly, identifying how a
state represents another state or non-state actor helps to understand how and why certain foreign policies
have been adopted while other policies have been excluded . To take a now famous example, if a transnational
organization is represented as a group of 'freedom fighters', such as the multi-national mujahideen in Afghanistan in the 1980s, then military
cooperation is conceivable with that organization; if on the other hand the same organization is represented as a 'terrorist network', such as Al-
Qaida, then military cooperation as a policy is simply not an option. In sum.
the way in which one sees, interprets and
imagines the 'other* delineates the course of action one will adopt in order to deal with this 'other'.

Security is a site of contestation—dominant discourses determine response


McDonald ’12 – Senior Lecturer in International Relations at the School of Political Science and
International Studies @ University of Queensland, Australia (Matt, “Security, The Environment, and
Emancipation: Contestation over Environmental Change”, Routledge, p.24)

By way of a conclusion to this discussion of security as a social construction, it is worth reiterating that security
is understood as constructed in the sense that
it is understood
by different political communities in different ways in different contexts, and is a site of
inter-subjective negotiation and contestation . And while it is appropriate to focus on how security is given meaning through the
articulation and designation of threat, it is also important to move beyond this concern to point to how referent objects themselves, and the values in need of
protection, are given meaning. Crucially, these
different understandings have different implications for policy and
action. The central determinant of how an issue is dealt with politically is not whether it is included on security agendas,
but the way in which a group's core values, the threats to those values, and the means of preserving or

advancing them are understood regarding that issue. I have also suggested here that, in approaching security as a social construction, we
should be concerned not only with the designation of threat, but also with the meaning of security. Such
an approach requires contesting the logic of security presented by the Copenhagen School and others, in which security is
the realm of emergency, exclusion and 'panic politics' . The remainder of this chapter sets out where we might begin to look for, and
make sense of, security discourses in political practice if not limiting ourselves to the designation and depiction of threat. Looking for security If the meaning

of security is context-specific, and if this meaning is a crucial determinant of how issues are addressed in
practice, it is clearly important to elaborate on where we might look for security discourses and how we might go
about doing this. Here, I outline a framework for understanding both the nature of significant security 'interventions', and the means through which we might go
about analysing these interventions and locating them in broader frameworks of meaning. It proceeds in two stages. The first outlines the role of language, defined
in terms of spoken representations and texts, as the central (although not only) form of representation in the discursive construction of security. The second
outlines the type of representation that delineates answers to core constituent questions of security ( whose
security, from what threats, and
also by what means and by whom should it be advanced? ). Here I advance the argument that we should focus on both the
invocation and evocation of security: the former involving the direct articulation of particular visions of 'referent', 'threat', 'means' and 'agent'; and the latter
involving the indirect articulation of such visions through representations of sovereignty and identity. This section also briefly addresses the question of how we
might look at these representations through the application of discourse-analytical methods. Language and the construction of security Lene Hansen has argued:
"language is ontologically significant: itis only through construction in language that 'things' - objects, subjects, states,
living beings, and material -tinctures - are given meaning and endowed with a particular identity'.74 Beyond the role of language in defining
security discourses and constituting reality more broadly, language - in text and especially speech - is central to dynamics of contestation

and negotiation about security. As Pierre Bourdieu has argued, actors are engaged in constant competition through
language to position their own account as most consistent with 'reality' and with a group's core values , in
the process attempting to marginalize alternatives narratives of reality and community .75 And successful actors

strategically use language that has particular symbolic power in given contexts (such as 'security', 'threat' or 'national

interest') to render their own accounts of security convincing and others less so. As noted, international relations theorists such as Mattern,
Krebs and Jackson have pointed to the ways in which successful speakers often attempt to locate narratives in powerful discourses of reality and community in
order to maximize resonance and coerce/compel political opponents to accept the speaker's account of a contested story lest they be seen as unpatriotic, for
example.76 The call to 'support our troops' that accompanied the deployment of military personnel to Iraq in 2003 in states such as the United States, the United
Kingdom and Australia may be seen as an example. Such a call arguably undermined the extent to which opponents could contest the intervention in Iraq without
also being seen to target those who would be risking their lives, thus limiting the space for legitimate opposition.'7 Similar examples are replete in the case studies
that follow, not least in the denigration of international opponents of deforestation in Brazil, and in domestic opposition to the Australian government's position on
climate change cooperation. Language, in short, constitutes the principal means through which meaning is given to
security in the contexts I address here. Of course, language is not the only means through which meaning is given or communicated, and discourses of security
are not reducible to linguistic representations of security and threat. A range of theorists have pointed convincingly to the important role of visual representations
as 'security' representations,78 for example, with examples ranging from depictions of tortured inmates in the infamous Abu Ghraib prison in Iraq to Danish
cartoons depicting the prophet Mohammed that sparked global unrest in 20 05.79 And at a more fundamental level, discourses are composed of both linguistic and
non-linguistic practices. This is especially true of dominant discourses, which become established in and practised through a wide range of processes, practices and
institutions. The post-2001 'war on terror' discourse in the American context, for example, was composed of, and constituted by, linguistic representations of the
terrorist threat by President Bush, but also through security processes at airports, the Department of Homeland Security's threat advisory images, and of course the
establishment of a Department of Homeland Security itself. It was also articulated and furthered across American society, from the level of political elite to cultural
media such as films and television programmes.80 While recognizing that language (through both speech and text) is therefore only one form through which
meanings of security are composed and communicated, my focus here is on the role of language, for two principal reasons. First, language
is the site of
the most direct and important form of engagement with security , directly delineating answers to core
questions such as 'whose security, from what threats, by what means?' Such representations , as Weldes et al.
argue, provide the vehicle for the construction of social categories that , in turn, serve to underpin the

production (through linguistic and non-linguistic practices) of social facts.81 Second, and related to this, a focus on language allows
greater recognition of security as a site of contestation between actors with competing visions of what
security means and how it might be realized . While dominant discourses are indeed institutionalized in a range of processes and dynamics
within (and crucially on behalf of) particular communities, marginal discourses are necessarily less evident in the official institutions, rituals and fora of a particular
community. Indeed, these marginal frameworks of meaning and the actors articulating them are regularly actively marginalized from these processes. As such, a
focus on language provides the best means of identifying multiple security discourses. The following analysis examines how dominant security discourses come to
be accepted and institutionalized, in turn underpinning the most important forms of action on behalf of a particular community regarding particular instances of
environmental change. A final point to note here in suggesting that wecan best look for security discourses (and the competition between
discourses) in linguistic practices is that analysing these practices has both analytical and normative implications , providing a

basis for progressive change in three central ways. First, analysing discourses generally allows us to 'make strange' the most

powerful discourses: frameworks of meaning that are necessarily partial but are represented and
accepted as common sense. This is a claim articulated powerfully by Gramsci, Foucault and Bourdieu, among many others. Second, analysing
the linguistic composition of discourses allows us to recognize possibilities for the most powerful forms
of contestation of these discourses, which tend to orient around practices of immanent critique. While this is
elaborated in chapter two, particularly important here is the broad claim that language enables but also constrains, providing

justifications for action that can effectively be contested and contrasted with the outcomes of that
action. Finally, a focus on language also allows us insight into those actors whose voices are able to be heard,
and those who are marginalized or , indeed, silenced. While the latter often occurs through the nature of the official institutions, rituals and
fora of a particular community that are the product of dominant security dis-courses, a focus on language draws our attention to the

vital question of which actors can and do 'speak for' particular communities, reminding us in the process
of those unable to have their voices heard . Types of security representation As I have suggested, security is a site of
contestation and negotiation, one that takes on particular importance given the political significance of
security, in particular as the central concern for the central actors in world politics: states. It should hardly be surprising, then, that attempts to define and
redefine security -and to lay the groundwork for these visions to resonate - are relatively ubiquitous in contemporary politics. I suggest here that attempts to
delineate answers to core constituent questions of security - 'referent', 'agent', 'threats' and 'means of response' - can be found in a range of different types of
representation beyond the direct designation of security and threat. This section outlines the broader view adopted here of the types of representation that serve
to engage with, and define, security. It asks: to what extent can particular representations be viewed as security representations? To reiterate an earlier point, the
key to answering this question lies in the extent to which particular representations engage with the preservation of a group's core values, delineating answers to
questions such as: who is the referent object and agent of security; who or what threatens this referent object and its core values; and what means should be
employed to advance or preserve the values of this particular community? For approaches to security that take any account of the role of representation and
language in constructing or positioning security, it is not controversial to argue that invocations of 'threat' and 'security' can be viewed as security representations.
The former - as noted - is a particularly prevalent theme in post-aructural and Copenhagen School research on the construction of security. The dual argument here
in justifying this focus is that security is denned primarily in oppositional terms - we
define what needs to be secured by designating
that which threatens it - and that engaging with the question of the designation of threat takes us to the
heart of the politics of security.82 The designation of threat is, for these approaches, therefore the most important
security 'move' in simultaneously telling us who or what needs to be protected, and from whom or what
it needs protecting. It also potentially enables particular types of political response and the actors
undertaking them, chiefly associated with the logic of exceptionalism.83 In my analysis, I accept the importance of these articulations of threat and
danger to the construction of security, even while broadening the terms of analysis to include other (less dramatic or obvious) forms of security representation.
Serial Policy Failure
Supremacy of policy-making crowds out critical questioning – causes serial policy
failure
Biswas 7 (Shampa, Professor of Politics – Whitman College, “Empire and Global Public Intellectuals:
Reading Edward Said as an International Relations Theorist”, Millennium, 36(1), p. 117-125)

The most serious threat to the ‘intellectual vocation’ , he argues, is ‘professionalism’ and mounts a pointed attack on the
proliferation of ‘specializations’ and the ‘cult of expertise’ with their focus on ‘relatively narrow areas of knowledge’, ‘technical
formalism’, ‘impersonal theories and methodologies’, and most worrisome of all, their ability and willingness to be
seduced by power.17 Said mentions in this context the funding of academic programmes and research which came out of the exigencies
of the Cold War18, an area in which there was considerable traffic of political scientists (largely trained as IR and comparative politics scholars)
with institutions of policy-making. Looking at various influential US academics as ‘organic intellectuals’ involved in a dialectical relationship with
foreign policy-makers and examining the institutional relationships at and among numerous think tanks and universities that create convergent
perspectives and interests, Christopher Clement has studied US intervention in the Third World both during and after the Cold War made
possible and justified through various forms of ‘intellectual articulation’.19 This
is not simply a matter of scholars working for the state, but
indeed a larger
question of intellectual orientation. It is not uncommon for IR scholars to feel the need to
formulate their scholarly conclusions in terms of its relevance for global politics, where ‘relevance’ is measured entirely in
terms of policy wisdom. Edward Said’s searing indictment of US intellectuals – policy-experts and Middle East experts - in the context
of the first Gulf War20 is certainly even more resonant in the contemporary context preceding and following the 2003 invasion of Iraq. The
space for a critical appraisal of the motivations and conduct of this war has been considerably diminished by the expertise-framed national
debate wherein certain kinds of ethical
questions irreducible to formulaic ‘for or against’ and ‘costs and
benefits’ analysis can simply not be raised. In effect, what Said argues for, and IR scholars need to pay particular heed to, is an
understanding of ‘intellectual relevance’ that is larger and more worthwhile, that is about the posing of critical,
historical, ethical and perhaps unanswerable questions rather than the offering of recipes and solutions, that is
about politics (rather than techno-expertise) in the most fundamental and important senses of the vocation.21
AT: State Key to Warming
State focused solutions to climate change fail and cause violent intervention – the
state is incompatible with a sustainable alternative
Cudworth and Hobden 11 (Erika and Stephen, School of Humanities and Social Sciences, University
of East London, UK, "Beyond environmental security: complex systems, multiple inequalities and
environmental risks," EBSCO)

are in many ways foundational for securitisation approaches, but as Dalby (2002c, p.
Second, and ironically, territorial states
5) notes, charging states with responsibility for the environment may well be a case of foxes guarding
chickens. States have been drivers for modernisation, putting in place infrastructure dependant on
carbon fuels and high levels of resource use, in large part, by establishing a capitalist system (Latouche 1993). The
fundamental question is whether states, and the international organisations of which they are members, can construct new physical and
institutional infrastructures that move us away from an ever expanding use of resources. Much
of what we have seen in terms of the
internationalisation of environmental policy involves an assumption that economic growth can be compatible with
ecological sustainability and ‘there is a techno-institutional fix for the present problems’ (Hajer 1995, p. 32). The politics of
ecologism, however, imply radical departures from our currently normative economic, political and social
practices in wealthy Western/Northern states , and has been extremely pessimistic concerning the ability of states to deliver
change (Sachs et al. 1998). Eckersley (2004, p. 241), however, is confident that a decline in territorially based governance accompanying
globalisation, coupled with a radical institutional reform, will make possible the consideration of ecological concern. She suggests a move from
liberal democracies to ‘ecological democracies’, where those collectivities subject to ecological risk must be involved in or represented in
decision-making, which may involve or generate ecological risk. Yet it is most uncertain that globalisation has actually reduced the power of
states, and international political organisations remain limited in both authority and power. Rather reforms
undertaken by states
to be a process in which environmental questions are subsumed under a
and supranational institutions tend
bureaucratic rationality of resource managerialism (Luke 1999). The logic of capitalist development, albeit linked with
‘sustainability’, is foundational for environmental policy initiatives (Clack and York 2005). State-derived security, as Neocleous (2008, p. 185–
statist focus has resulted in a tendency to associate
186) suggests, is a ‘gift’ that we might like to return. In addition, this
issues of security
with military solutions. Deudney (1990, p. 465–469) has argued that the use of the term ‘security’ links the
environment too closely to questions of national survival . Such an association may lead to inappropriate
polices, especially the prioritising of military solutions over diplomatic methods. There is evidence that the leaky
nature of environmental insecurities has also led organisations concerned with transnational security to take seriously nonmilitary threats.
North Atlantic Treaty Organisation (NATO) had a team of researchers employed from the late 1990s with the task of ascertaining which
environmental matters might contribute to international conflict (as discussed by Dalby 2002a, p. 97). A report produced by a group of retired
US military personnel concluded that ‘climate change can act as a threat multiplier for instability in some of the most volatile regions of the
world, and it presents significant national security challenges for the United States’ (CNA 2009). Some suggest, therefore, that we have
witnessed a ‘greening’ of the
military wherein security organisations are increasingly deployed in realising
environmental goals (Matthew 2002, p. 118). Yet there may be unintended consequences of this link. Extending the notion of
security to the environmental sphere, according to Brock (1997, p. 21), might ‘actually broaden the range of
arguments for justifying military action’. Hence a link to security might have unexpected and
undesirable consequences: the prioritisation of coercive solutions over diplomatic approaches to
environmental issues, and the legitimation of other forms of intervention
Predictions Fail
Worst case predictions cause worst case policy making—recognition of our ignorance
makes us more secure than their fatalistic scenario planning
Schneier 10 [Bruce Schneier is an internationally renowned security technologist and author, MA CS
American University, 3/13/10, http://www.schneier.com/blog/archives/2010/05/worst-case_thin.html]
At a security conference recently, the moderator asked the panel of distinguished cybersecurity leaders what their nightmare scenario was. The
answers were the predictable array of large-scale attacks: against our communications infrastructure, against the power grid, against the
financial system, in combination with a physical attack. I didn’t get to give my answer until the afternoon, which was: "My nightmare scenario is
that people keep talking about their nightmare scenarios." There’s a certain blindness that comes from worst-case
thinking. An extension of the precautionary principle, it involves imagining the worst possible outcome and then
acting as if it were a certainty. It substitutes imagination for thinking, speculation for risk analysis, and
fear for reason. It fosters powerlessness and vulnerability and magnifies social paralysis . And it makes us
more vulnerable to the effects of terrorism. Worst-case thinking means generally bad decision making for
several reasons. First, it’s only half of the cost-benefit equation . Every decision has costs and benefits, risks and rewards. By
speculating about what can possibly go wrong, and then acting as if that is likely to happen, worst-case
thinking focuses only on the extreme but improbable risks and does a poor job at assessing outcomes .
Second, it’s based on flawed logic. It begs the question by assuming that a proponent of an action must
prove that the nightmare scenario is impossible. Third, it can be used to support any position or its
opposite. If we build a nuclear power plant, it could melt down. If we don’t build it, we will run short of
power and society will collapse into anarchy. If we allow flights near Iceland’s volcanic ash, planes will crash and people will
die. If we don’t, organs won’t arrive in time for transplant operations and people will die . If we don’t invade Iraq, Saddam
Hussein might use the nuclear weapons he might have. If we do, we might destabilize the Middle East ,
leading to widespread violence and death. Of course, not all fears are equal. Those that we tend to exaggerate are more
easily justified by worst-case thinking. So terrorism fears trump privacy fears, and almost everything else ;
technology is hard to understand and therefore scary; nuclear weapons are worse than conventional weapons; our children need to be
protected at all costs; and annihilating the planet is bad. Basically, any fear that would make a good movie plot is amenable to worst-case
thinking. Fourth and finally, worst-case thinking validates ignorance. Instead of focusing on what we know, it
focuses on what we don’t know -- and what we can imagine. Remember Defense Secretary Rumsfeld’s quote? "Reports that say
that something hasn’t happened are always interesting to me, because as we know, there are known knowns; there are things we know we
know. We also know there are known unknowns; that is to say we know there are some things we do not know. But there are also unknown
unknowns -- the ones we don’t know we don’t know." And this: "the absence of evidence is not evidence of absence." Ignorance
isn’t a
cause for doubt; when you can fill that ignorance with imagination, it can be a call to action . Even worse, it
can lead to hasty and dangerous acts. You can’t wait for a smoking gun, so you act as if the gun is about
to go off. Rather than making us safer, worst-case thinking has the potential to cause dangerous
escalation. The new undercurrent in this is that our society no longer has the ability to calculate probabilities .
Risk assessment is devalued. Probabilistic thinking is repudiated in favor of "possibilistic thinking ": Since we
can’t know what’s likely to go wrong, let’s speculate about what can possibly go wrong. Worst-case thinking leads to bad
decisions, bad systems design, and bad security. And we all have direct experience with its effects: airline security and the
TSA, which we make fun of when we’re not appalled that they’re harassing 93-year-old women or keeping first graders off airplanes. You can’t
be too careful! Actually, you can. You can refuse to fly because of the possibility of plane crashes. You can lock your children in the house
because of the possibility of child predators. You can eschew all contact with people because of the possibility of hurt. Steven Hawking wants to
avoid trying to communicate with aliens because they might be hostile; does he want to turn off all the planet’s television broadcasts because
they’re radiating into space? It isn’t hard to parody worst-case thinking, and at its extreme it’s a psychological condition. Frank Furedi,
a
sociology professor at the University of Kent, writes: "Worst-case thinking encourages society to adopt
fear as one of the dominant principles around which the public , the government and institutions should organize
their life. It institutionalizes insecurity and fosters a mood of confusion and powerlessness. Through
popularizing the belief that worst cases are normal, it incites people to feel defenseless and vulnerable
to a wide range of future threats." Even worse, it plays directly into the hands of terrorists, creating a
population that is easily terrorized -- even by failed terrorist attacks like the Christmas Day underwear bomber and the Times
Square SUV bomber. When someone is proposing a change, the onus should be on them to justify it over the status quo. But worst-case
thinking is a way of looking at the world that exaggerates the rare and unusual and gives the rare much
more credence than it deserves. It isn’t really a principle; it’s a cheap trick to justify what you already
believe. It lets lazy or biased people make what seem to be cogent arguments without understanding
the whole issue. And when people don’t need to refute counterarguments, there’s no point in listening
to them.
Predictions Fail – Complexity
Causal linear IR predictions are inherently incomplete and fail to capture the
complexity of reality – epistemic uncertainty is the defining principle of international
politics
Hendrick 9 – PhD from Bradford U, contributor to Oxford University Press (Diane, “Complexity Theory
and Conflict Transformation: An Exploration of Potential and Implications”,
http://www.brad.ac.uk/acad/confres/papers/pdfs/CCR17.pdf)

In international relations Neil E. Harrison makes the case for the value of complexity theory given the
unpredictability of events in world politics that has confounded expectations based on existing theories. While
there are various explanations proffered for this situation, Harrison sees the tendency of current theories of world politics to
work with models of the social world that present it, for analytical purposes, as a simple system as fundamentally
misleading. In contrast to realism, that sees political behaviour being driven by essential human characteristics within fixed structures,
complexity theory sees world politics as a self- organising complex system in which macroproperties emerge
from microinteractions. It is precisely the interactions among interdependent but individual agents within the system that account for
the surprising events that defy prediction through the simple models used at the moment. Harrison thus
takes the state as a system that is not closed but open to other natural and social systems: “defined as a political system, it
is open to technological, cultural and economic systems that influence political choices and processes.”
(Harrison, 2006 p. 8) The state is also influenced by other states and by numerous transboundary interactions between
major corporations, NGOs, terrorist groups, etc. In such complex systems it is not possible to trace linear
causal links: “Despite occasional attempts to bring in domestic politics the state is usually modelled as a unit with exogenous identity and
objective interests. This greatly reduces the range of possible causal explanations for any perceived social event, simplifying causal analysis and
hypothesis generation and testing.” (Harrison, 2006 p. 11) It is a disconcerting fact that outcomes may have multiple causes and
that in different contexts, historically or spatially, the same cause may lead to different outcomes. This cannot be captured by
the over-simplified models of international systems. Given the multiple, mutually influencing interactions within social systems it is necessary to
look to the evolution of the system rather than to individual events when seeking the causes of observed effects. Complexity theory focuses on
processes and relations between components, or in the case of social systems, agents, rather than the components themselves. In a similar
criticism to that of Walby, Harrison points to the tendency of theories in international relations to focus on one level of analysis and to present
competing theories based on these. Where systems are theorised, they are limited by being presented as nested. Harrison notes that the
impact of positive feedback in systems has been acknowledged: “ ‘(I)ntra-national and inter-national events all impinge on one another in a
cyclical and ongoing process within which the self-aggravating propensities frequently exceed the self-correcting ones by an unacceptably large
amount’ (Singer 1970, 165) thus national elites use rhetoric for domestic political consumption that can incite potential enemies, the public and
military desire the psychological comfort of discernible superiority, media amplify inter-nation conflicts, and the benefits of participation in the
ideological mainstream preserve the distribution of power and inhibit changes in the historic patterns that transform inevitable conflicts into
costly rivalries.” (Harrison, 2006 p. 28) While Walby refers to examples of the importance of the notion of path dependence with reference to
differences in development between countries, Harrison sees its relevance at the level of the international state system. Thus development
through time is not wholly random and there are limits or constraints created by the prior development of the system that restrict the possible
options for change. In this way the international system may change its structure without becoming another system and here Harrison brings
the example of the Cold War. While it is true that the Cold War was produced by historical interactions, it is still not possible to claim that it was
an inevitable effect of historical causes. The myriad microinteractions that occurred introduce unpredictability into
development, especially given the above-mentioned possibility of positive feedback. Harrison is optimistic with regard to the gains from the
application of complexity theory to world politics in theoretical but also in policy terms: “This ontological shift from simple to complex systems
opens new paths to knowledge and understanding yet incorporates much current knowledge; it validates novel research methods; and theories
founded in this approach will generate radically different solutions to policy problems.” (Harrison, 2006 p. 2)
Predictions Fail – State Intentions
It is not possible to discern the goals of other states
Rosato 15 – prof of political science @ University of Notre Dame
(Sebastian, The Inscrutable Intentions of Great Powers, International Security, Vol. 39, No. 3: Winter
2014/15, pp. 48–88) **Note: Rosato makes a distinction between goals (what a state wants) and
intentions (what a state plans to do to achieve its goals)

Great powers cannot reach confident assessments of the intentions of their peers based on their foreign policy
goals, ideology, or regime type. To be sure, states can learn something about other states’ intentions by scrutinizing their
domestic characteristics. They cannot, however, learn enough to judge their intentions with confidence . ¶ The
contention that great powers can confidently infer others’ intentions from their foreign policy goals is
flawed for two reasons. First, determining goals is hard; and second, goals and intentions are not
straightforwardly related.¶ States face formidable obstacles in trying to assess others’ goals. It is reason- able
to assume that rational states want security, but beyond that they can have many goals.47 For example, they may want to
increase their prosperity, spread their ideology, or shape the global institutional landscape . In addition, it is
hard to know which objectives are more important to them and how they will adjudicate among
different goals when these conflict. Also, goals can change. As Jervis points out, “[E]ven if the other state now
supports the status quo, it may become dissatisfied later. . . . Minds can be changed, new leaders can
come to power, values can shift, new opportunities and dangers can arise.” 48 Furthermore, states have
incentives to conceal or misrepresent their objectives.49 A great power that aspires to hegemony, for example, will try to
hide this from potential victims to lull them into a false sense of security. Consequently, there is substantial uncertainty when it
comes to divining states’ goals.¶ Given these difficulties, there are no accepted guidelines for determining states’ goals,
and therefore even the most well known historical examples are hotly debated. In the case of World
War I, defensive realists claim that German aggression was driven by insecurity, specifically by fears of
encirclement and the rise of Russia. Imperial Germany was essentially a security seeker, albeit one with aggressive intentions.50
Offensive realists dissent, claiming that Germany was a revisionist power bent on “dominating the
European continent.”51 Similar disagreement exists about the origins of the Cold War, with analysts
variously arguing that both superpowers preferred the status quo, that the United States was a
revisionist and the Soviet Union was a status quo power, that the Soviet Union was a revisionist and the
United States was a status quo power, and that both great powers wanted to change the status quo .52
These examples undermine propositions that rely on goals: if scholars armed with definitions and the
documentary record cannot agree about what states wanted long after the fact, it is unlikely that
great powers can do so in real time.¶ Even if states could figure out others’ goals, a significant problem
would remain: goals are not a dependable guide to intentions. A status quo power may have peaceful
intentions; that is, it may plan to keep things as they are without threatening or using force. But it may equally have aggressive
intentions, believing that it must destroy its peers to maintain the current situation . Thus the same goal—
preserving the status quo—can generate different plans of action . The argument also applies to revisionists. A revisionist power
may have aggressive intentions; that is, it may plan to overturn the existing order by force. Or it may have peaceful intentions, simply choosing
to sit tight and wait for its more powerful rivals to decline. Again, the same goal can be associated with different intentions.
AT: Security Inevitable
Even if security is inevitable, our thought exercise is more productive than their stable
production of the present – the alternative enables a different conception of security
that can overcome inevitable conflict
Burke 7 (Anthony, Senior Lecturer – School of Politics and Professor of International Relations – University of New South Wales, Beyond
Security, Ethics and Violence, p. 68-69)

This chapter is thus anexercise in thinking, which challenges the continuing power of political ontologies (forms of
truth and being) that connect security, sovereignty, belonging, otherness and violence in ways that for many appear
like enduring political facts, inevitable and irrefutable. Conflict, violence and alienation then arise not merely
from individual or collective acts whose conditions might be understood and policed; they condition politics as such,
forming a permanent ground, a dark substrata underpinning the very possibility of the present. Conflict
and alienation seem inevitable because of the way in which the modem political imagination has
conceived and thought security, sovereignty and ethics. Israel/ Palestine is chosen here as a particularly urgent and complex
example of this problem, but it is a problem with much wider significance. While I hold out the hope that security can be re-visioned away from
a permanent dependence on insecurity, exclusion and violence, and I believe it retains normative promise, this analysis takes a deliberate step
backward to examine the very real barriers faced by such a project. Security cannot properly be rethought without a deeper
understanding of, and challenge to, the political forms and structures it claims to enable and protect. If Ken Booth argues
that the state should be a means rather than an end of security, my objective here is to place the continuing power and depth of its status as an
end of security, and a fundamental source for political identity, under critical interrogation.' If the state is to become a means of security (one
among many) it will have to be fundamentally transformed. The chapter pursues this inquiry in two stages. The first outlines the historic
strength and effective redundancy of such an exciusivist vision of security in Israel, wherein Israel not only confronts military and political
antagonists with an 'iron wall' of armed force but maps this onto a profound clash of existential narratives, a problem with resonances in the
West's confrontation with radical Islamism in the war on terror. The second, taking up the remainder of the chapter, then explores a series of
potential resources in continental philosophy and political theory that might help us to think our way out of a security grounded in violence and
alienation. Through
a critical engagement with this thought, I aim to construct a political ethics based not in
relations between insecure and separated identities mapped solely onto nation-states, but in relations of responsibility and
interconnection that can negotiate and recognise both distinct and intertwined histories , identities and needs;
an ethics that might underpin a vision of interdependent (national and non-national) existence proper to an
integrated world traversed by endless flows of people , commerce, ideas, violence and future potential.
AT: Permutation
Permutation fails – orthodox IR crowds out alternative forms of knowledge production
Jackson 15 [Patrick Thaddeus Jackson (Professor of International Relations @ American University),
“Must International Studies Be a Science?”, Millennium: Journal of International Studies, June 2015, Vol.
43(3) 942–965, http://mil.sagepub.com/content/43/3/942.full]

So we clearly have these different forms of knowing in the field broadly understood. Should we continue to? Although
partisans of epistemic knowing suggest otherwise, I would argue that we would be quite impoverished if we were to
discard other forms of knowing from our scholarly endeavour. After all, epistemic knowledge of whisky in
important ways supervenes on other knowledges: technical knowledge of distilling and maturation, aesthetic knowledge of
the diverse experiences associated with consuming whisky, normative knowledge of distilled spirits as a separate category. Indeed, this is
perhaps inevitably the case with epistemic knowing, which strives to be doubly separate from things:
detached from the flow of events, and impersonally removed from individual perspectives. Detached,
impersonal knowing—itself made possible, as Sandra Harding reminds us, by the power of the (colonial) knower
to both expand its empirical grasp and to borrow from local knowledges as it sees fit 101—does achieve
distinctive results through its pursuit of a systematic, public, and worldly factuality. But those results do
not mean that every other mode of knowing ought to be subordinated to epistemic knowing, any more than
the results of technical or aesthetic or normative knowing ought to necessarily dominate the field. In the end there are no definitive
philosophical arguments supporting the claim of any form of knowing to legitimate domination of the
field of international studies. Practitioners might like to see more technical knowing; activists and critics might prefer more
normative knowing; artists, more aesthetic knowing. This is impossible to settle through discussion, because the knowledge-interests at play are
different, and the intention of advancing a knowledge-claim—which tells us how to evaluate that claim—might be one of several options. We
commit a category mistake if we insist on treating every knowledge-claim as though it were intended to
be a contribution to epistemic knowledge ; this is the mistake that fuels the false dichotomy between, for example, evolutionary
biology and the notion of the world as having been formed by a divine Creator. No sacred text of which I am aware intends to provide epistemic
knowledge about the universe, and Galileo’s famous quip that the Bible tells one how to go to Heaven, not how the heavens go, captures the
point nicely: these are two different language-games, two different intentions, two different kinds of knowing. There is no conflict here, unless
So in the end this is a question of
we force one by erroneously presuming that all the claimants are intending the same thing.
meta-methodology: what attitude toward methodological diversity should we adopt? Perhaps unsurprisingly,
my answer with respect to the four forms of knowing I have discussed in this essay is much the same as my answer to the four philosophical
ontologies I discussed in The Conduct of Inquiry in International Relations. In
the absence of any definitive philosophical or
logical argument for the superiority of any of these ways of knowing, and in light of the global
importance of the issues with which we grapple in international studies, I am not convinced that we
have any compelling grounds for closing the borders of the field around one form of knowing. The
practical knowledge that enables one to navigate a tricky conflict situation is not likely to be contained in
general studies of the relative effectiveness of different strategies , and local experience and expertise is
needed to implement even a strategy that has worked well elsewhere. Epistemic knowing also, famously,
can’t tell us how to live together on the planet , because that’s not its goal; for that, normative knowing is needed. And
aesthetic knowing can capture and highlight just those situational particulars and diverse experiences
missed by other forms of knowing. So my answer is that the field should contain space for all of these kinds of
knowing, and should resist the call to be exclusively a form of aesthetic expression, a mode of technical
practice, a normative enterprise—or an epistemic science. Instead of intending to produce one
homogenous body of knowledge, the field of international studies and we scholars within it should work to
become more comfortable with the irreducible plurality, plurivocality, and diversity of knowing. Such an
attitude can, I think, best be cultivated not merely by our paying lip-service to the existence of other
forms of knowing, but by our actively seeking to position ourselves as scholars in the spaces between
these diverse ways of knowing. Which does not mean that an individual piece of scholarly work can or
should inhabit those spaces. The logical distinctions between ways of knowing, and the dynamics of academic knowledge-production,
virtually guarantee that whatever we write, whatever claims we make, will be read as contributions to the advancement of one or another
knowledge-interest, and evaluated accordingly.102 And if we are honest with ourselves, we do have particular ends and aims for which we
advance knowledge-claims, a heading which we may not have deliberately chosen ourselves but which is always and already given in our very
ways of going on.103 The logical differences between ways of knowing, just like the logical differences between philosophical ontologies,
mandate that a claim can only be judged a good one if we know what standards to apply to it. So pieces
of scholarly work, or
particular knowledge-claims made in our scholarly work,104 inhabit particular ways of knowing, but that is no
reason why we ourselves as scholars need to subordinate everything to one kind of approach. The
diversity of styles and flavours of single malt whisky does not, after all, imply that the best thing to do is to dump
them all into a vat and consume the resulting ‘multi-method’ product. And in practice, given the uneven playing-
field on which such ‘multi-method’ blends are conducted, we are more likely to end up with blended Scotch that is mostly composed of neutral
grain spirit, which further diminishes the distinctiveness of any individual contribution. What we need instead is the
attitude of a participant in a whisky tasting, who can appreciate the distinctiveness of particular single malts in their singularity, but is not
unequivocally and for all time wedded to any one of them. The
trick is to understand each claim in its appropriate
intentional context, where appropriateness is a matter of discussion and debate with reference to the
form and content of the claim itself: multiple readings of a claim may well be possible, but that does
not absolve us of the difficult work of producing a compelling reading . And then we should not apply a
single standard of knowing to all knowledge-claims, but instead should accentuate the differences
between ways of knowing instead of dissolving them into a flavourless mess or incoherently eclectic
grab-bag.
Affirmative Answers
Security Turns
Threats Real
Threats are real and not constructed – action is key to prevent them
Knudsen 1– PoliSci Professor at Sodertorn (Olav, Post-Copenhagen Security Studies, Security Dialogue 32:3)
Moreover, I have a problem with the underlying implication that it is unimportant whether states 'really' face dangers from other states or
groups. In the Copenhagen school, threats are seen as coming mainly from the actors' own fears , or from
what happens when the fears of individuals turn into paranoid political action. In my view, this
emphasis on the subjective is a
misleading conception of threat , in that it discounts an independent existence for what- ever is
perceived as a threat. Granted, political life is often marked by misperceptions , mistakes, pure imaginations,
ghosts, or mirages, but such phenomena do not occur simultaneously to large numbers of politicians, and
hardly most of the time . During the Cold War, threats - in the sense of plausible possibilities of
danger - referred to 'real' phenomena, and they refer to 'real' phenomena now. The objects referred to are
often not the same, but that is a different matter. Threats have to be dealt with both ín terms of perceptions and in terms of the
phenomena which are perceived to be threatening. The
point of Waever’s concept of security is not the potential
existence of danger somewhere but the use of the word itself by political elites. In his 1997 PhD dissertation,
he writes, ’One can View “security” as that which is in language theory called a speech act: it is not interesting as a sign referring to
something more real - it is the utterance itself that is the act.’24 The deliberate disregard of objective factors is even more explicitly stated
in Buzan & WaeVer’s joint article of the same year.” As a consequence, t he phenomenon of threat is reduced to a matter
of pure domestic politics.” It seems to me that the security dilemma, as a central notion in security
studies, then loses its foundation. Yet I see that Waever himself has no compunction about referring to the security dilemma in
a recent article." This discounting of the objective aspect of threats shifts security studies to insignificant concerns. What has long
made 'threats' and ’threat perceptions’ important phenomena in the study of IR is the implication
that urgent action may be required . Urgency, of course, is where Waever first began his argument in
favor of an alternative security conception, because a convincing sense of urgency has been the chief
culprit behind the abuse of 'security' and the consequent ’politics of panic', as Waever aptly calls it. ”
Now, here - in the case of urgency - another baby is thrown out with the Waeverian bathwater. When real situations of
urgency arise, those situations are challenges to democracy; they are actually at the core of the
problematic arising with the process of making security policy in parliamentary democracy. But in
Waever’s world, threats are merely more or less persuasive, and the claim of urgency is just another
argument. I hold that instead of 'abolishing' threatening phenomena ’out there’ by reconceptualizing
them, as Waever does, we should continue paying attention to them, because situations with a
credible claim to urgency will keep coming back and then we need to know more about how they
work in the interrelations of groups and states ( such as civil wars, for instance), not least to find adequate
democratic procedures for dealing with them.
Security Good
Even if security and risk calculation are flawed, engaging in them creates discourse of
social welfare and promotes a democratic civic culture that checks political exclusion
and loss of value to life
Loader 7 (Ian Loader, Criminology Prof at Oxford, Civilizing Security, Pg. 5)
Faced with such inhospitable conditions, one can easily lapse into fatalistic despair, letting events simply come as they will, or else
seek refuge in the consolations offered by the total critique of securitization practices – a path that some critical
scholars in criminology and security studies have found all too seductive (e.g. Bigo 2002, 2006; Walters 2003). Or one can, as we have done,
supplement social criticism with the hard, uphill, necessarily painstaking work of seeking to specify what it
may mean for citizens to live together securely with risk ; to think about the social and political arrangements capable of
making this possibility more rather than less likely, and to do what one can to nurture practices of collective security
shaped not by fugitive market power or by the unfettered actors of (un)civil society, but by an inclusive, democratic politics.
Social analysts of crime and security have become highly attuned to, and warned repeatedly of, the illiberal,
exclusionary effects of the association between security and political community (Dillon 1996; Hughes 2007).
They have not, it should be said, done so without cause, for reasons we set out at some length as the book unfolds. But this sharp
sensitivity to the risks of thinking about security through a communitarian lens has itself come at a price,
namely, that of failing to address and theorize fully the virtues and social benefits that can flow from members of a
political community being able to put and pursue security in common . This, it seems to us, is a failure to heed
the implications of the stake that all citizens have in security; to appreciate the closer alignment of self-interest and
altruism that can attend the acknowledgement that we are forced to live , as Kant put it, inescapably side-by-
side and that individuals simultaneously constitute and threaten one another’s security; and to register the security-enhancing
significance and value of the affective bonds of trust and abstract solidarity that political communities
depend upon, express and sustain. All this, we think, offers reasons to believe that security offers a conduit,
perhaps the best conduit there is, for giving practical meaning to the idea of the public good, for
reinventing social democratic politics, even for renewing the activity of politics at all.
Fear Good
Fear motivates people to pursue constructive means to sustain peace and prevent
large-scale catastrophe.
Lifton 01 (Robert Jay, Distinguished Professor of Psychiatry and Psychology at John Jay College,
Illusions of the second nuclear age, World Policy Journal. New York: Spring
2001. Vol. 18, Iss. 1;  pg. 25, 6 pgs)

The trouble is that in other ways the dangers associated with nuclear weapons are greater than ever : the continuing
weapons-- centered policies in the United States and elsewhere; the difficulties in controlling nuclear weapons that exist under unstable
conditions (especially in Russia and other areas of the former Soviet Union);2 and the eagerness and potential capacity of certain nations and
"private" groups to acquire and possibly use the weapons. In that sense, the nuclear quietism is perilous. Or, to put the matter another way,
we no longer manifest an appropriate degree of fear in relation to actual nuclear danger . While fear in itself is
hardly to be recommended as a guiding human emotion, its absence in the face of danger can lead to catastrophe. We
human animals have built-in fear reactions in response to threat. These reactions help us to protect ourselves-to step back from the path of a
speeding automobile, or in the case of our ancestors, from the path of a wild animal. Fear
can be transmuted into constructive
planning and policies: whether for minimizing vulnerability to attacks by wild animals, or for more complex
contemporary threats. Through fear, ordinary people can be motivated to pursue constructive means for
sustaining peace, or at least for limiting the scope of violence. Similarly, in exchanges between world leaders
on behalf of preventing large-scale conflict , a tinge of fear-sometimes more than a tinge- can enable each to feel the
potential bloodshed and suffering that would result from failure. But with nuclear weapons, our psychological circuits
are impaired. We know that the weapons are around-and we hear talk about nuclear dangers somewhere "out there" -but our minds no longer
connect with the dangers or with the weapons themselves. That blunting of feeling extends into other areas. One of the many sins for which
advocates of large nuclear stockpiles must answer is the prevalence of psychic numbing to enormous potential suffering, the blunting of our
ethical standards as human beings. In
the absence of the sort of threatening nuclear rhetoric the United States
and Russia indulged in during the 1980s, we can all too readily numb ourselves to everything nuclear,
and thereby live as though the weapons pose no danger, or as though they don't exist.

Fear of nuclear weapons has prevented their use – deterrence has checked conflict.
Rajaraman 02 (Professor of Theoretical Physics at JNU, 2002 [R., “Ban battlefield nuclear weapons,”
4/22/2, The Hindu,
http://www.hinduonnet.com/thehindu/2002/04/22/stories/2002042200431000.htm[
There were a variety of different reasons behind each of these examples of abstinence from using nuclear weapons. But one major common
factor contributing to all of them has been an ingrained terror of nuclear devastation. The well documented images of Hiroshima and Nagasaki,
the awesome photographs of giant mushroom clouds emerging from nuclear tests in the Pacific and the numerous movies based on nuclear
Armageddon scenarios have all contributed to building up a deep rooted fear of nuclear weapons . This is
not limited just to the abhorrence felt by anti-nuclear activists. It permeates to one extent or another the psyche of all
but the most pathological of fanatics. It colours the calculations, even if not decisively, of the most
hardened of military strategists. The unacceptability of nuclear devastation is the backbone of all
deterrence strategies. There is not just a fear of being attacked oneself, but also a strong mental barrier against actually
initiating nuclear attacks on enemy populations, no matter how much they may be contemplated in war
games and strategies. As a result a taboo has tacitly evolved over the decades preventing nations, at
least so far, from actually pressing the nuclear button even in the face of serious military crises.
Fear is inevitable – the only question is whether state institutions use it as an
instrumental tool to consolidate power or concerned publics use it as a form of
political resistance
Kurasawa 4 (Fuyuki, Sociology Professor at York University, “Cautionary Tales: The Global Culture of
Prevention and the Work of Foresight” Constellations, Vol 11 No 4)

State and market institutions may seek to produce a culture of fear by deliberately stretching
interpretations of reality beyond the limits of the plausible so as to exaggerate the prospects of
impending catastrophes, or yet again, by intentionally promoting certain prognoses over others for
instrumental purposes. Accordingly, regressive dystopias can operate as Trojan horses advancing political agendas or commercial
interests that would otherwise be susceptible to public scrutiny and opposition. Instances of this kind of manipulation of the dystopian
imaginary are plentiful: the invasion of Iraq in the name of fighting terrorism and an imminent threat of use of ‘weapons of mass destruction’;
the severe curtailing of American civil liberties amidst fears of a collapse of ‘homeland security’; the neoliberal dismantling of the welfare state
as the only remedy for an ideologically constructed fiscal crisis; the conservative expansion of policing and incarceration due to supposedly
spiraling crime waves; and so forth. Alarmism constructs and codes the future in particular ways, producing or reinforcing certain crisis
narratives, belief structures, and rhetorical conventions. As much as alarmist ideas beget a culture of fear, the reverse is no less true. If fear-
mongering is a misappropriation of preventive foresight, resignation about the future represents a
problematic outgrowth of the popular acknowledgment of global perils. Some believe that the world to come is so
uncertain and dangerous that we should not attempt to modify the course of history; the future will look after itself for better or
worse, regardless of what we do or wish. One version of this argument consists in a complacent optimism perceiving the future as fated to be
better than either the past or the present. Frequently accompanying it is a self-deluding denial of what is plausible (‘the world will not be so bad
after all’), or a naively Panglossian pragmatism (‘things will work themselves out in spite of everything, because humankind always finds ways to
survive’).37 Much more common, however, is the opposite reaction, a fatalistic pessimism reconciled to the idea that the future will be
necessarily worse than what preceded it. This is sustained by a tragic chronological framework according to which humanity is doomed to
decay, or a cyclical one of the endless repetition of the mistakes of the past. On top of their dubious assessments of what is to come,
alarmism and resignation would, if widely accepted, undermine a viable practice of farsightedness.
Indeed, both of them encourage public disengagement from deliberation about scenarios for the future ,
a process that appears to be dangerous, pointless, or unnecessary. The resulting ‘depublicization’ of debate leaves
dominant groups and institutions (the state, the market, techno-science) in charge of sorting out the
future for the rest of us, thus effectively producing a heteronomous social order . How, then, can we support a
democratic process of prevention from below? The answer, I think, lies in cultivating the public capacity for critical
judgment and deliberation, so that participants in global civil society subject all claims about potential
catastrophes to examination, evaluation, and contestation .
Predictions Good
Policymakers will inevitably make predictions – failure to use explicit risk calculation
causes poor decision-making
Fitzsimmons 7 (Michael, Washington DC defense analyst, “The Problem of Uncertainty in Strategic
Planning”, Survival, Winter 06-07, online)
In defence of prediction Uncertainty is not a new phenomenon for strategists. Clausewitz knew that ‘many intelligence reports in war are
contradictory; even more are false, and most are uncertain’. In coping with uncertainty, he believed that ‘what one can reasonably ask of an
officer is that he should possess a standard of judgment, which he can gain only from knowledge of men and affairs and from common sense.
He should be guided by the laws of probability.’34 Granted, one can certainly allow for epistemological debates about the best ways of gaining
‘a standard of judgment’ from ‘knowledge of men and affairs and from common sense’. Scientific inquiry into the ‘laws of probability’ for any
given strate- gic question may not always be possible or appropriate. Certainly, analysis cannot and should not be presumed to trump the
intuition of decision-makers. Nevertheless, Clausewitz’s implication seems to be that the
burden of proof in any debates about
planning should belong to the decision-maker who rejects formal analysis, standards of evidence and
probabilistic reasoning. Ultimately, though, the value of prediction in strategic planning does not rest
primarily in getting the correct answer, or even in the more feasible objective of bounding the range of correct answers. Rather,
prediction requires decision- makers to expose, not only to others but to themselves, the beliefs they
hold regarding why a given event is likely or unlikely and why it would be important or unimportant .
Richard Neustadt and Ernest May highlight this useful property of probabilistic reasoning in their renowned study of the use of history in
decision-making, Thinking in Time. In discussing the importance of probing presumptions, they contend: The need is for tests prompting
questions, for sharp, straightforward mechanisms the decision makers and their aides might readily recall and use to dig into their own and
each others’ presumptions. And they need tests that get at basics somewhat by indirection, not by frontal inquiry: not ‘what is your inferred
causation, General?’ Above all, not, ‘what are your values, Mr. Secretary?’ ... If someone says ‘a fair chance’ ... ask, ‘if you were a betting man
or woman, what odds would you put on that?’ If others are present, ask the same of each, and of yourself, too. Then probe the differences:
why? This is tantamount to seeking and then arguing assumptions underlying different numbers placed on a subjective probability assessment.
We know of no better way to force clarification of meanings while exposing hidden differences ... Once differing odds have been quoted, the
question ‘why?’ can follow any number of tracks. Argument may pit common sense against common sense or analogy against analogy. What is
important is that the expert’s basis for linking ‘if’ with ‘then’ gets exposed to the hearing of other experts before the lay official has to say yes
or no.’35 There are at least three critical and related benefits of prediction in strate- gic planning. The first reflects Neustadt and May’s point –
prediction enforces a certain level of discipline in making explicit the assumptions, key variables and
implied causal relationships that constitute decision-makers’ beliefs and that might otherwise remain
implicit. Imagine, for example, if Shinseki and Wolfowitz had been made to assign probabilities to their
opposing expectations regarding post-war Iraq. Not only would they have had to work harder to justify
their views, they might have seen more clearly the substantial chance that they were wrong and had to
make greater efforts in their planning to prepare for that contingency . Secondly, the very process of
making the relevant factors of a decision explicit provides a firm , or at least transparent, basis for making
choices. Alternative courses of action can be compared and assessed in like terms. Third, the transparency and discipline of
the process of arriving at the initial strategy should heighten the decision-maker’s sensitivity toward
changes in the environment that would suggest the need for adjustments to that strategy . In this way,
prediction enhances rather than under-mines strategic flexibility . This defence of prediction does not imply that great
stakes should be gambled on narrow, singular predictions of the future. On the contrary, the central problem of uncertainty in plan- ning
remains that any given prediction may simply be wrong. Preparations for those eventualities must be made. Indeed, in many cases, relatively
unlikely outcomes could be enormously consequential, and therefore merit extensive preparation and investment. In order to navigate this
complexity, strategists must return to the dis- tinction between uncertainty and risk. While
the complexity of the international
security environment may make it somewhat resistant to the type of probabilistic thinking associated
with risk, a risk-oriented approach seems to be the only viable model for national-security strategic
planning. The alternative approach, which categorically denies prediction, precludes strategy . As Betts
argues, Any assumption that some knowledge, whether intuitive or explicitly formalized, provides guidance about what should be done is a
presumption that there is reason to believe the choice will produce a satisfactory outcome – that is, it is a prediction, however rough it may
be. If there is no hope of discerning and manipulating causes to produce intended effects, analysts as well as politicians and generals should all
quit and go fishing.36 Unless they are willing to quit and go fishing, then, strategists must sharpen their tools of risk assessment. Risk
assessment comes in many varieties, but identification of two key parameters is common to all of them: the consequences of a harmful event
or condition; and the likelihood of that harmful event or condition occurring. With no perspective on likelihood, a strategist can have no firm
perspective on risk. With no firm perspective on risk, strategists cannot purposefully discriminate among alternative choices. Without
purposeful choice, there is no strategy. * * * One of the most widely read books in recent years on the complicated relation- ship between
strategy and uncertainty is Peter Schwartz’s work on scenario-based planning, The Art of the Long View. Schwartz warns against the hazards
faced by leaders who have deterministic habits of mind, or who deny the difficult implications of uncertainty for strategic planning. To
overcome such tenden- cies, he advocates the use of alternative future scenarios for the purposes of examining alternative strategies. His view
of scenarios is that their goal is not to predict the future, but to sensitise leaders to the highly contingent nature of their decision-making.37
This philosophy has taken root in the strategic-planning processes in the Pentagon and other parts of the US government, and properly so.
Examination of alternative futures and the potential effects of surprise on current plans is essential. Appreciation of uncertainty also has a
number of organisational impli- cations, many of which the national-security establishment is trying to take to heart, such as encouraging
multidisciplinary study and training, enhancing information sharing, rewarding innovation, and placing a premium on speed and versatility.
The arguments advanced here seek to take nothing away from these imperatives of planning and operating in an uncertain environment. But
appreciation of uncertainty carries hazards of its own. Questioning assumptions is critical, but assumptions must be
made in the end. Clausewitz’s ‘standard of judgment’ for discriminating among alternatives must be applied. Creative, unbounded
speculation must resolve to choice or else there will be no strategy. Recent history suggests that unchecked scepticism regarding
the validity of prediction can marginalise analysis , trade significant cost for ambig- uous benefit, empower parochial
interests in decision-making, and undermine flexibility . Accordingly, having fully recognised the need to broaden their
strategic-planning aperture, national-security policymakers would do well now to reinvigorate their efforts in the messy but indispensable
business of predicting the future.

Policy-makers can make reasonably accurate predictions based on social science


methods, empiricism, and relatively objective facts
Chernoff 9 Fred, Prof. IR and Dir. IR – Colgate U., European Journal of International Relations,
“Conventionalism as an Adequate Basis for Policy-Relevant IR Theory”, 15:1, Sage

For these and other reasons, many social


theorists and social scientists have come to the conclusion that
prediction is impossible. Well-known IR reflexivists like Rick Ashley, Robert Cox, Rob Walker and Alex Wendt have attacked
naturalism by emphasizing the interpretive nature of social theory. Ashley is explicit in his critique of prediction, as is Cox, who says quite
simply, ‘It is impossible to predict the future’ (Ashley, 1986: 283; Cox, 1987: 139, cf. also 1987: 393). More recently, Heikki Patomäki has
argued that ‘qualitative changes and emergence are possible, but predictions are not’ defective and that the latter two presuppose an
unjustifiably narrow notion of ‘prediction’.14 A
determined prediction sceptic may continue to hold that there is
too great a degree of complexity of social relationships  (which comprise ‘open systems’) to allow any
prediction whatsoever. Two very simple examples may  circumscribe and help to refute a radical variety
of scepticism. First, we all make reliable social predictions and do so with great frequency.  We can
predict with high probability that a spouse, child or parent will react to  certain well-known stimuli that we
might supply, based on extensive past experience . More to the point of IR prediction – scepticism, we can imagine a
young child in the UK who (perhaps at the cinema) (1) picks up a bit of 19th-century British imperial lore thus gaining a sense
of the power of the crown, without knowing anything of current balances of power, (2) hears some stories about the US–UK
invasion of Iraq in the context of the aim of advancing democracy, and (3) hears a bit about communist China and
democratic Taiwan. Although the specific term ‘preventative strike’ might not enter into her lexicon,
it is possible to imagine the child, whose knowledge is thus limited, thinking that if democratic
Taiwan were threatened by China, the UK would  (possibly or probably) launch a strike on China to protect
it, much as the UK had done to help democracy in Iraq. In contrast to the child, readers of this journal and scholars who study the
world more thoroughly have factual information (e.g. about the relative military and economic
capabilities of the UK and China) and hold some cause-and-effect principles (such as that states do not usually
initiate actions that leaders understand will have an extremely high probability of undercutting their power with almost no chances of
success). Anyone who has adequate knowledge of world politics would predict that the UK will not launch a preventive attack against
China. In the real world, China knows that for the next decade and well beyond the UK will not
intervene militarily in its affairs . While Chinese leaders have to plan for many likely — and even a few somewhat unlikely —
future possibilities, they do not have to plan for various implausible contingencies: they do not have to structure forces geared to defend
against specifically UK forces and do not have to conduct diplomacy with the UK in a way that would be required if such an attack were a
real possibility. Any
rational decision-maker in China may use  some cause-and-effect (probabilistic)
principles along with knowledge of specific facts relating to  the Sino-British relationship to
predict (P2) that the UK will not land its forces on Chinese territory  — even in the event of a war over Taiwan (that
is, the probability is very close to zero). The statement P2 qualifies as a prediction based on DEF above and counts as knowledge for
Chinese political and military decision-makers. A Chinese diplomat or military planner who would deny that theory-based prediction would
have no basis to rule out extremely implausible predictions like P2 and would thus have to prepare for such unlikely contingencies as UK
action against China.
A reflexivist theorist sceptical of ‘prediction’ in IR might argue that the China example
distorts the notion by using a trivial prediction and treating it as a meaningful one. But  the critic’s
temptation to dismiss its value stems precisely from the fact that  it is so obviously true. The value to
China of knowing that the UK is not a military threat is significant.  The fact that, under current conditions, any
plausible cause-and-effect understanding of IR that one might adopt would yield P2, that the ‘UK will not attack China’, does not diminish
the value to China of knowing the UK does not pose a military threat. A critic might also argue that DEF and the China example allow non-
scientific claims to count as predictions. But we
note that while physics and chemistry offer precise ‘point
predictions’, other natural sciences, such as seismology, genetics or meteorology, produce predictions that
are often much less specific; that is, they describe the predicted ‘events’ in broader time frame and typically in probabilistic
terms. We often find predictions about the probability, for example, of a seismic event in the form ‘some time in the next three years’
rather than ‘two years from next Monday at 11:17 am’. DEF includes approximate and probabilistic propositions as predictions and is thus
able to catagorize as a prediction the former sort of statement, which is of a type that is often of great value to policy-makers. With
the
help of these ‘non-point predictions’  coming from the natural and the social sciences, leaders are able to choose
the courses of action (e.g. more stringent earthquake-safety building codes, or procuring an additional carrier battle group) that
are most likely to accomplish the leaders’ desired ends.  So while ‘point predictions’ are not what
political leaders require in most decision-making situations, critics of IR predictiveness often attack
the predictive capacity of IR theory for its inability to deliver them . The critics thus commit the straw
man fallacy by requiring a sort of prediction in IR (1) that few, if any, theorists claim  to be able to
offer, (2) that are not required by policy-makers for theory-based predictions to be valuable, and  (3)
that are not possible even in some natural sciences .15 The range of theorists included in ‘reflexivists’ here is very wide
and it is possible to dissent from some of the general descriptions. From the point of view of the central argument of this article, there are
two important features that should be rendered accurately. One is that reflexivists reject explanation–prediction symmetry, which allows
them to pursue causal (or constitutive) explanation without any commitment to prediction. The second is that almost all share clear
opposition to predictive social science.16 The reflexivist commitment to both of these conclusions should be evident from the foregoing
discussion.
Apocalyptic Rhetoric Good
Apocalyptic environment rhetoric key to influence activism – framing warming as a
narrative mobilizes moral deliberation and political responses that would be
impossible absent scenarios for catastrophe
Veldman 12 – PhD Candidate Religion and Nature at U of Florida
(Robin, National Foundation Fellow at the Integrative Graduate Education and Research Traineeship,
"Narrating the Environmental Apocalypse: How Imagining the End Facilitates Moral Reasoning Among
Environmental Activists" Ethics and the Environment, 17.1, Muse)
As we saw in the introduction, critics often argue that apocalyptic rhetoric induces feelings of hopelessness or fatalism. While it certainly does
for some people, in this section I will present evidence that apocalypticism also often goes hand in hand with activism.
Some of the strongest evidence of a connection between environmental apocalypticism and activism
comes from a national survey that examined whether Americans perceived climate change to be
dangerous. As part of his analysis, Anthony Leiserowitz identified several “interpretive communities,” which had consistent demographic
characteristics but varied in their levels of risk perception. The group who perceived the risk to be the greatest, which he
labeled “alarmists,” described climate change [End Page 5] using apocalyptic language, such as “Bad…bad…bad…like
after nuclear war…no vegetation,” “Heat waves, it’s gonna kill the world,” and “Death of the planet” (2005, 1440). Given such language, this
would seem to be a reasonable way to operationalize environmental apocalypticism. If such
apocalypticism encouraged fatalism, we would expect alarmists to be less likely to have engaged in
environmental behavior compared to groups with moderate or low levels of concern. To the contrary,
however, Leiserowitz found that alarmists “were significantly more likely to have taken personal action to
reduce greenhouse gas emissions” (ibid.) than respondents who perceived climate change to pose less of a threat. Interestingly,
while one might expect such radical views to appeal only to a tiny minority, Leiserowitz found that a respectable eleven percent of Americans
fell into this group (ibid). Further supporting Leiserowitz’s findings, in
a separate national survey conducted in 2008, Maibach, Roser-
Renouf, and Leiserowitz found that a group they labeled “the Alarmed” (again, due to their high levels of concern about climate change)
“are the segment most engaged in the issue of global warming. They are very convinced it is happening,
human-caused, and a serious and urgent threat. The Alarmed are already making changes in their own
lives and support an aggressive national response ” (2009, 3, emphasis added). This group was far more likely than people
with lower levels of concern over climate change to have engaged in consumer activism (by rewarding companies that support action to reduce
global warming with their business, for example) or to have contacted elected officials to express their concern. Additionally, the authors found
that “[w]hen asked which reason for action was most important to them personally, the Alarmed were
most likely to select preventing the destruction of most life on the planet (31%)” (2009, 31)—a finding suggesting
that for many in this group it is specifically the desire to avert catastrophe , rather than some other motivation, that
encourages pro-environmental behavior. Taken together, these and other studies (cf. Semenza et al. 2008 and
DerKarabetia, Stephenson, and Poggi 1996) provide important evidence that many of those who think
environmental problems pose a severe threat practice some form of activism, rather than giving way to
fatalistic resignation. National surveys give a good overview of the association between apocalypticism and activism among the general
public, but they do not [End Page 6] provide sufficient ethnographic detail. To complement this broader picture I now turn to case studies,
which provide greater insight into how adherents themselves understand what motivates their environmental behavior. When seeking a subset
of environmentalists with apocalyptic beliefs, the radical wing is an obvious place to look. For example, many Earth First!ers believe that the
collapse of industrial society is inevitable (Taylor 1994). At the same time, the majority are actively committed to preventing ecological disaster.
As Earth First! co-founder Howie Wolke acknowledged, the two are directly connected: “As ecological calamity unravels the living fabric of the
Earth, environmental radicalism has become both common and necessary” (1989, 29).3 This logic underlies efforts to preserve wilderness
areas, which many radical environmentalists believe will serve as reservoirs of genetic diversity, helping to restore the planet after industrial
society collapses (Taylor 1994). In addition to encouraging activism to preserve wilderness, apocalyptic beliefs also motivate practices such as
“monkeywrenching,” or ecological sabotage, civil disobedience, and the more conventional “paper monkeywrenching” (lobbying, engaging in
public information campaigns to shift legislative priorities, or using lawsuits when these tactics fail). Ultimately, while there are disagreements
over what strategies will best achieve their desired goals, for most radical environmentalists, apocalypticism and activism are
bound closely together. The connection between belief in impending disaster and environmental activism holds true for Wiccans as
well. During fieldwork in the southeastern United States, for example, Shawn Arthur reported meeting “dozens of Wiccans who professed their
apocalyptic millenarian beliefs to anyone who expressed interest, yet many others only quietly agreed with them without any further
elaboration” (2008, 201). For this group, the coming disaster was understood as divine retribution, the result of an angry Earth Goddess
preparing to punish humans for squandering her ecological gifts (Arthur 2008, 203). In light of Gaia’s impending revenge, Arthur found that
Wiccans advocated both spiritual and material forms of activism. For example, practices such as Goddess worship, the use of herbal remedies
for healing, and awareness of the body and its energies were considered important for initiating a more harmonious relationship with the earth
(Arthur 2008, 207). As for material activism, Arthur notes [End Page 7] that the notion of environmental apocalypse played a key role in
encouraging pro-environmental behavior: images
of immanent [sic] ecological crisis and apocalyptic change often
were utilized as motivating factors for developing an environmentally and ecologically conscious
worldview; for stressing the importance of working for the Earth through a variety of practices , including
environmental activism, garbage collecting, recycling, composting, and religious rituals; for learning sustainable living skills; and for developing
a special relationship with the world as a divine entity. (2008, 212) What these studies and my own experiences in the environmentalist milieu4
suggest is that people who make a serious commitment to engaging in environmentally friendly behavior, people who move beyond making
superficial changes to making substantial and permanent ones, are quite likely to subscribe to some form of the apocalyptic narrative. All this is
not to say that apocalypticism directly or inevitably causes activism, or that believing catastrophe is imminent is the only reason people become
activists. However, it is to say thatactivism and apocalypticism are associated for some people, and that this
association is not arbitrary, for there is something uniquely powerful and compelling about the
apocalyptic narrative. Plenty of people will hear it and ignore it, or find it implausible, or simply decide that if the situation really is so
dire there is nothing they can do to prevent it from continuing to deteriorate. Yet to focus only on the ability of apocalyptic
rhetoric to induce apathy, indifference or reactance is to ignore the evidence that it also fuels quite the
opposite—grave concern, activism, and sometimes even outrage. It is also to ignore the movement’s history. From Silent
Spring (Carson [1962] 2002) to The Limits to Growth (Meadows et al 1972) to The End of Nature (McKibben 1989), apocalyptic
arguments have held a prominent place within environmental literature , topping best-seller lists and spreading
the message far and wide that protecting the environment should be a societal priority . Thus, while it is not a
style of argument that will be effective in convincing everyone to commit to the environmental cause (see Feinberg and Willer 2011), there
does appear to be a close relationship between apocalyptic belief and activism among a certain minority. The next section explores the
implications of that relationship further. [End Page 8] The Apocalyptic Narrative as a Framework for Moral Deliberation In discussing how
apocalypticism functions within the environmental community, it will be helpful to analyze it as a type of narrative. I do so because the domain
of narrative includes both the stories that people read and write, as well as those they tell and live by. By using narratives as data, scholars can
analyze experiential and textual sources simultaneously (Polkinghorne 1988; Riessman 2000). To analyze environmental
apocalypticism as a type of narrative is not to suggest that apocalyptics’ claims about the future are fictional. Rather, it is to
highlight that the facts to which environmentalists appeal have been organized with particular goals in
mind, goals which have necessarily shaped the selection and presentation of those facts. Compelling
environmental writers do not simply list every known fact pertaining to the natural world, but instead select certain findings and place them
within a larger interpretive framework. Alone,
each fact has little meaning, but when woven into a larger narrative,
a message emerges. This process of narrativization is essential if a message is to be persuasive (Killingsworth
and Palmer 2000, 197), and has occurred not only in the rapidly expanding genre of environmental nonfiction, but in much scientific writing
about the environment as well (Harré, Brockmeier, and Mühlhäusler 1999, 69). What defines narratives as such is their beginning-middle-end
structure, their ability to “describe an action that begins, continues over a well-defined period of time, and finally draws to a definite close”
(Cronon 1992, 1367). Here I will focus on the last of these elements, the ending, because anything we can learn about how endings function
within narratives in general will be applicable to the apocalypse, the most final ending of all. An ending is essential in order for a story to be
complete, but there is more to it than this. Endings are also key because they establish a story’s moral, the lesson it is supposed to impart upon
the reader. In other words, to know
the moral of the story, auditors must know the consequences of the actions
depicted therein, so there can be no moral without an ending . To take a simple example, when we hear the story of the
shepherd boy who falsely claims that a wolf is attacking his flock of sheep in order to entertain himself at his community’s expense, what makes
the lesson clear is that when a wolf does attack his flock, the disenchanted town members refuse to come to his aid. By clearly illustrating how
telling lies can have [End Page 9] unpleasant consequences for the perpetrator, the ending reveals the moral that lying is wrong. As Cronon
explains, it is “[t]he difference between beginning and end [that] gives us our chance to extract a moral from the rhetorical landscape” (1992,
1370). Endings play a similar role in environmental stories . In Al Gore’s book Earth in the Balance (1992), for example, he
devotes over a third of the book’s pages to presenting scientific evidence that disaster is imminent.5 As he sums it up, “Modern industrial
civilization…is colliding violently with our planet’s ecological system. The ferocity of its assault on the earth is breathtaking, and the horrific
consequences are occurring so quickly as to defy our capacity to recognize them” (1992, 269). He builds this argument so carefully precisely
because if the ending does not seem credible, the moral he wants readers to draw from the story will not be compelling. If his readers are not
convinced that the ending to this story of ecological misbehavior will be a debacle of colossal proportions, they will not become convinced that
they need to dramatically alter their ecological behavior. Thus
the vision of future catastrophe that Gore presents provides a
crucial vantage point from which the present environmental situation can be understood as the result of
a grand moral failure, and Gore’s readers are made aware of their obligations in light of it. Gore himself appreciates the importance of
this recognition, arguing that “whether we realize it or not, we are now engaged in an epic battle to right the balance of our earth, and the tide
of this battle will turn only when the majority of people in the world become sufficiently aroused by a shared sense of urgent danger to join an
all-out effort” (1992, 269, emphasis added). Here, as in so many other stories, the ending must be in place for the moral to become clear. To say
that endings are essential in order for stories to have morals is already to hint that stories alter behavior, that they can encourage action in the
real world even as they invoke an imaginary one. This much is clear from Earth in the Balance (1992): Gore does not just want people to grasp a
moral, to perceive some ethic in the abstract—he wants them change their behavior in the here and now. In constructing a narrative with this
goal in mind, he is banking on the ability of powerful stories to motivate social change, to be, as Cronon puts it, “our chief moral compass in the
world” (1992, 1375). Mark Johnson’s insightful synthesis of cognitive science and philosophy helps explain further how this process of moral
guidance occurs. For [End Page 10] Johnson, narrative is fundamental to our experience of reality, “the most
comprehensive means we have for constructing temporal syntheses that bind together and unify our
past, present, and future into more or less meaningful patterns ” (1993, 174). Narratives are also critical to our
ability to reason morally, an activity which Johnson asserts is fundamentally imaginative. In this view, we use stories to
imagine ourselves in different scenarios, exploring and evaluating the consequences of different possible
actions in order to determine the right one. Moral deliberation is thus …an imaginative exploration of
the possibilities for constructive action within a present situation. We have a problem to solve here and
now (e.g., ‘What am I to do?’…. ‘How should I treat others?’), and we must try out various possible continuations of our
narrative in search of the one that seems best to resolve the indeterminacy of our present situation.
(1993, 180) Put another way, what cognitive science has revealed is that from an empirical perspective the process of moral deliberation entails
constructing narratives rooted in our unique history and circumstances, rather than applying universal principles (such as Kant’s categorical
imperative) to particular cases. That we use narratives to reason morally is not a result of conscious choice but of how human cognition works.
That is, insofar as we experience ourselves as temporal beings, a
narrative framework is necessary to organize, explain,
and ultimately justify the many individual decisions that over time become a life . Formal principles may
be useful in unambiguous textbook cases, but in real life “we can almost never decide (reflectively) how to
act without considering the ways in which we can continue our narrative construction of our situation ”
(Johnson 1993, 160). Empirically speaking, “our moral reasoning is situated within our narrative understanding” (Johnson 1993, 180, italics in
original). The
observation that people use narratives to reason morally may help explain the association
between environmental apocalypticism and activism. The function of the apocalyptic narrative may be that it
helps adherents determine how to act by providing a storyline from which they can imaginatively
sample, enabling them to assess the consequences of their actions. In order to answer the question, “Should I drive or walk
to the store?” for example, they can reason, “If I walk, that will reduce my carbon footprint, which will help keep the ice caps from melting,
saving humans and other species.” It
is their access to this narrative of impending [End Page 11] disaster that makes
such reasoning possible, for it provides a simple framework within which people can consider and
eventually arrive at some conclusion about their moral obligations .6 More broadly, it can guide entire lives by
providing a narrative frame of reference that imbues the individual’s experiences with meaning. For example, it is the context of looming
anthropogenic apocalypse which suggests that dedicating one’s life to achieving a healthier relationship with the natural world is a worthwhile
endeavor. Absent the apocalypse, choices such as limiting one’s travel to reduce greenhouse gas emissions,
becoming vegetarian, working in the environmental sector (often for less compensation), or growing one’s own
food could seem to be meaningless sacrifices. Within this context, on the other hand, such choices
become essential features of the quest to live a moral life. The apocalyptic narrative is but one of many ways to tell the
environmental story, yet it is one that seems particularly well-suited to encouraging pro-environmental behavior. First, the apocalyptic
ending discloses certain everyday decisions as moral decisions. Without the narrative context of
impending disaster, decisions such as whether to drive or walk to the store would be merely matters of convenience
or preference. In the context of potentially disastrous consequences for valued places, people, and
organisms, by contrast, such decisions become matters of right and wrong . Second, putting information
about the environment into narrative form enables apocalyptics to link complex global environmental
processes to their own lives, a perceptual technique Thomashow describes as “bringing the biosphere home” (2002). Developing
this skill is essential because without that felt sense of connection to their own lived experience, people
are much less likely to become convinced that it is incumbent upon them to act (2002, 2). Finally, the sheer
magnitude of the impending disaster increases the feeling of responsibility to make good on one’s moral
intuitions. By locating individuals within a drama of ultimate concern, the narrative frames their choices
as cosmically important, and this feeling of urgency then helps to convert moral deliberation into action .
With this conceptual overview in place, we can now examine more closely what the relationship between apocalypticism and moral reasoning
looks like in practice.

Environmental representations that effect all serve to motivate collective action


Brulle 2k (Robert, Environmental Science @ Drexel, Agency, Democracy, and Nature p. 277-279)
An example of a
partial metanarrative of nature is represented by the term biodiversity . This discursive
invention unified disparate discourses and groups that were concerned about destruction of the
natural environment due to deforestation, overfishing, introduction of exotic species, hunting, habitat
destruction, and extinction of species. It did notabsorb or destroy the other discourses. Rather, it
expanded the concerns of the various groups to see their common purposes . By so doing, it lead to an
increase in collective action. A similar environmental metanarrative would allow for "the construction
of a new common sensewhich changes the identity of the different groups sothat their differing practices are able to
complement one another(Torgerson 1999: 47). There is no one particular environmental discourse that could function as a
metanarrative. The development of the various environmental frames has resulted in a number of particular discourses which are unique
cultural responses to specific conditions of ecological degradation (Eder 1996b: 163). Instead ,
an environmental metanarrative
will consist of multiple forms of arguments to motivate action in different social orders . There is no
requirement that joint action be based on one set ofcultural beliefs; the only requirement is that there
exist good reasons to act in a particular manner. "Political unity," Schlosberg (1998: 87-101) notes, "does
not require that a political agreement be reached based on identical reasons. Rather, unity can be achieved
through recognition and inclusion of multiplicity and particularity. 7i The theory of communicative action can inform the creation of a meta-
narrative by describing both the types of arguments the metanarrative would have to make and the social conditions under which it would be
cre- ated. First, the metanarrative must provide aesthetic, moral, and cognitive reasons for collective action. In order to create a rational
agreement about what joint action should be followed, a discourse must establish that it accurately represents the
objective world, that the acceptance of a proposed action is in accordance with other existing cultural norms and beliefs, and that the
statement is an authentic representation of the speaker's inner se lf.4 Hence, the development of collective action depends on a discourse's
sus- taining the validity claims of truth, normative rightness, and authenticity. This means that the multipleand partial discourses on
the natural environment must be integrated to form a coherent discourse that can provide cognitive
scientific, normative, and aesthetic rationales for the preservation of nature (Eder l996a: 215). Thus, contributions are needed
from all the environmental discourses. The existing environmental discourses form the starting point for such an
effort. According to Killingsworth and Palmer (1992: 266), an ecological metanarrative "will ... draw energy and direc- tion from them and in
turn will influence their sense of purpose and their understanding of their relationships to other discourses. The continuous narrative of an
environmentalist culture will, above all, be the medium through which communicative action is realized and perpetuated." Thus, to be
successful, an environmental metanarrative must recognize the valid- ity of the great variety of viewpoints and rationales with regard to
protect- ing the natural environment, and this knowledge must inform collective action (Schlosberg 1998: 21). Thus, we will need to "enlarge
the range of voices in our conversation and with them the means of considering our rela- tionship with the natural world" (Killingsworth and
Palmer 1992: 79). An
ecological metanarrative would draw on the special management, scientific and legal
capacities of the Conservation, Wildlife Management, and Reform Environmentalist discourses to ensure scientific
competence and adequately address scientific questions . To develop new normative criteria, it would need to
encompass the moral fervor and commitment of the ecotheologists and the deep concerns over equity and justice of both the Ecofeminist
discourse and the Environmental justice discourse. To address our images of what constitutes the good life, it would need to incorporate the
aesthetic insights provided by the discourses of Preservation and Deep Ecology. The theory of communicative action also defines the process
through which this metanarrative would be created. Here Habermas's communica- tive ethics specifies that the process of validating a
discourse requires an open speech community in which the unforced force of the better argument prevails. This ethical relationship, a
presupposition of mutual communica- tions, requires basic recognition and acceptance of others and respect for the autonomy and integrity of
the others' identity and selfhood (Schlosberg 1998: 68). Thus, the theory of communicative action specifies that the cre- ation of an ecological
metanarrative must occur through open communi- cation based on solidarity and mutual respect. An
open dialogue among all
environmental organizations would createa democratic community, which couldthen debate and
developcoordinated actions to deal with ecological degradation . Torgerson (1999: 160) has labeled the arena in which
this dialogue would take place the "green" polit- ical sphere. It is not a specific institution or think tank. Rather, it defines a change in the
relationship of the various environmental communities that would enable them to engage one another in a community of dialogue (ibid.: 161).
The "green" public sphere would define a space in which "industrialist presuppositions do not prevail" (ibid.: 162). This public space would be
the arena in which environmental politics would take place and meaningful disagreements and debates about our society and the actions
necessary to create an ecologically sustainable society would be carried out. Creating an environmental metanarrative through this dialogue
would enable the creation of an environmental community capable of democrat- ically discussing proposed actions and then acting together
(ibid.: 107).

Eco-apocalyptic discourse solves extinction


Leeson-Schatz 12 – PhD candidate in English @ Binghamton
Joe, “The Importance of Apocalypse: The Value of End-Of- The-World Politics While Advancing
Ecocriticism,” Journal of Ecocriticism, 4.2
It is no longer a question that human interaction with the world is destroying the very ecosystems that sustain
life1. Nevertheless, within academic communities people are divided over which discursive tactic , ontological
position, or strategy for activism should be adopted. I contend that regardless of an ecocritic’s particular orientation that
ecocriticism most effectively produces change when it doesn’t neglect the tangible reality that surrounds any
discussion of the environment. This demands including human-‐induced ecocidal violence within all our accounts .
Retreating from images of ecological collapse to speak purely within inner-‐academic or policymaking circles
isolates our conversations away from the rest of the world—as it dies before our eyes. This is not to argue
that interrogating people’s discourse, tactics, ontological orientation, or anything else lacks merit. Timothy Luke, Chair and Distinguished
Professor of Political Science at the Virginia Polytechnic Institute, explains that Because nothing in Nature simply is given within society, such
terms must be assigned sig-‐ nificance by every social group that mobilizes them[.] ... Many styles of ecologically grounded criticism circulate in
present-‐day American mass culture, partisan debate, consumer society, academic discourse, and electoral politics as episodes of ecocritique,
contesting our politics of nature, economy, and culture in the contemporary global system of capitalist production and consumption. (1997: xi)
Luke reminds us that regardless of how ecocritics advance their agenda they always impact our environmental awareness and therefore alter
our surrounding ecology. In doing so he shows that both literal governmental policies and the symbolic universe they take place within
reconstruct the discourses utilized to justify policy and criticism in the first place. This is why films like The Day After Tomorrow and 2012 can
put forth realistic depictions of government response to environmental apocalypse. And despite being fictional, these films in turn can influence
the reality of governmental policy. Even the science-‐fiction of weather-‐controlling weapons are now only steps away from becoming reality2.
Oftentimes it takes images of planetary annihilation to motivate people into action after years of sitting
idly by watching things slowly decay. In reality it takes awareness of impending disaster to compel
policymakers to enact even piecemeal reform. On the screen it takes the actual appearance of ecological
apocalypse to set the plot in motion. What is constant is that “as these debates unfold, visions of what is the good or bad life ...
find many of their most compelling articulations as ecocritiques ... [that are] mobilized for and against various projects of power and economy
in the organization of our everyday existence” (Luke 1997: xi). We cannot motivate people to change the ecological
conditions that give rise to thoughts of theorization without reference to the concrete environmental destruction
ongoing in reality. This means that, even when our images of apocalypse aren’t fully accurate, our use of
elements of scientifically-‐established reality reconstructs the surrounding power structures in beneficial ways . When we
ignore either ecological metaphors or environmental reality we only get part of the picture.` In recent years, many ecocritics have
shied away from the very metaphors that compel a sense of urgency. They have largely done so out of the fear that its
deployment will get co-‐opted by hegemonic institutions. Such critics ignore how what we advocate alters our
understanding of ourselves to the surrounding ecology . In doing so, our advocacies render such co-optation meaningless
because of the possibility to redeploy our metaphors in the future. In the upcoming sections, I will provide an overview of how poststructuralist
thinkers like Michel Foucault and Martin Heidegger influence some ecocritics to retreat from omnicidal rhetoric. This retreat minimizes the
main objectives of their ecocriticism. I argue that rather than withdrawing from images of apocalypse that we should
utilize them in subversive ways to disrupt the current relationship people have to their ecology . Professor
of Sociology at York University, Fuyuki Kurasawa argues that “instead of bemoaning the contemporary preeminence of a dystopian
imaginary ... it can enable a novel form of transnational socio-political action ... that can be termed
preventive foresight. ... [I]t is a mode of ethico-political practice enacted by participants in the emerging
realm of global civil society ... [by] putting into practice a sense of responsibility for the future by
attempting to prevent global catastrophes” (454-‐455).

Invoking climatic apocalypse is politically empowering—experimenting with policy


mechanisms is not just a “quick fix”, it creates awareness of our agency that is more
valuable than theorizing
Clark 14—professor in the Lancaster Environment Center at Lancaster University (UK) (Nigel,
“Geopolitics and the disaster of the Anthropocene”, Volume 62, Issue Supplement S1, pages 19–37, June
2014, dml)

Whether or not ‘apocalyptic’ imagery serves to promote or incapacitate politicization has long been
debated in environmentalist circles and in critical social thought ( see Swyngedouw , 2007; cf. Yusoff and Gabrys, 2011). In another
register and another field, the question of whether ‘actual’ disasters provide opportunities for political
transformation, or whether they are primarily occasions for the entrenchment of pre-existing power
relations, has also been a matter of lively discussion (Cuny, 1983; Pelling and Dill, 2010; Kelman, 2012; Tironi, this volume).
Whereas Naomi Klein's (2008) bestselling inquiry into the machinations of ‘disaster capitalism’ comes down firmly on the side of the latter,
geographer Mark Pelling and anthropologist Kathleen Dill sift through a range of case studies to arrive, cautiously, at a more
hopeful prognosis . ‘Disaster shocks’, they propose ‘ open political space for the contestation or
concentration of political power and the underlying distributions of rights between citizens and
citizens and the state’ (2010: 34). Engaging in a more general sense with the political potential of the crisis or emergency, political
theorist Bonnie Honig comes to a similar conclusion. Taking issue with the rush of recent critical work that characteristically equates the state of
emergency with the suspension of civil liberties and the closure of political possibility, Honig argues for the fundamental ambivalence of
invoking emergency, observing that no
declaration of emergency can dictate how it will be received, interpreted
and acted upon. In contrast to claims that the ‘emergency brings an end to real politics’, she seeks out
and discovers new possibilities for political renewal and change : ‘hidden resources and alternative
angles of vision that might motivate action in concert in emergency settings’ (2009: xv; see also Aradau and van
Munster, 2011). But what might these political possibilities be? What is demanded of the political in the face of the threats and challenges
designated by the Anthropocene? In the final section, I want to sketch out some of the ways that responses to the current geologic
predicament of humankind are awakening to Michel Serres' call for a ‘geopolitics in the sense of the real Earth’ (1995: 44; see also Dalby, 2007).
More than a matter of confronting the consequences of our own actions, I want to suggest, a
growing conception of the inherent
instability of the Earth is beginning to impact upon our understanding of the composition of the
political; our sense of what it is we work with – or against – when we mobilize collectively. Towards an
Anthropocene geopolitics Resonating with other researchers in the field of science and technology studies, Sheila Jasanoff writes of
‘the indeterminacy and complexity of many novel risks, and their refusal to stay within neatly drawn geopolitical lines’ (2010: 19; see also
Petersen, this volume). It is timely, however, to ask what exactly the ‘geo’ in ‘geopolitical’ is doing in this scenario, and what claims about the
coming of an Anthropocene epoch might mean for such an understanding of ‘geopolitical lines’. Perhaps the most crucial lesson of the
Anthropocene is that the
Earth itself must be understood as much more than a mere surface or stage on
which political contests take place: it must acquire a volumetric or vertical dimension (Dalby, 2013; see also Elden, 2013). That is
to say, the ‘geopolitical’ can no longer simply refer to a horizontal and synchronous globality. But this requires something more than extending
the conventional concerns of geopolitical discourse and practice upwards into the atmosphere or downwards into the depths of the ocean or
Earth. It requires us to bring politics into an intensive engagement with the planet's own dynamics: its processes of sedimentation and
mobilization, its layering and folding, its periodicities and singularities. This means that the crucial borders or thresholds on the political agenda
are not only those which divide nations or other socially inscribed territorial divisions of the Earth's surface, but also the spatio-temporal
junctures at which one state or regime of an Earth system passes into another (Clark, 2011, see Weszkalnys, this volume on the Cenomanian
Turonian extinction or boundary event). Or to put it another way, politics must expand its concerns with the shaping and reshaping of territory
to embrace processes of stratification and destratification (see Deleuze and Guattari, 1987). When it comes to the threat of
crossing boundaries or thresholds in Earth systems , as Johan Rockström and his interdisciplinary team observes:
‘[c]urrent governance and management paradigms are often oblivious to or lack a mandate to act
upon these planetary risks’ (2009: unpag.).While the repeated failure of climate summits to achieve the binding commitments
necessary to ward off ‘dangerous’ or ‘extremely dangerous’ climate change is the most conspicuous manifestation of this shortfall, the
relative paucity of attention to other imminent or already-transgressed ‘planetary boundaries’ is no
less revealing (Anderson and Bows, 2011; Rockström et al., 2009). Recent calls for what has been variously termed ‘planetary
stewardship’ (Steffen et al., 2011); ‘Earth System governmentality’ (Lövbrand et al., 2009); and ‘global earth system governance’ (Dryzek
and Stevenson, 2011: 1873) express a growing recognition of the need for new or greatly strengthened
frameworks to meet the political challenge of maintaining Earth systems in socially desirable states.
Needless to say, normative reasoning is far from enough to conjure such architectures into existence. Any

conceivable success, political theorists John Dryzek and Hayley Stevenson remind us, must work through and from
existing experience (2011: 1873). But what kinds of experience might be relevant here? We have seen that critical
social thinkers can be as apprehensive about the successful operationalizing of strategies to ‘manage’
Earth systems as they are about inadequate planetary governance. While radical critics tend to champion a
generalized advancement of democratic or deliberative political processes, they are often less than
forthcoming about their own preferences for responding practically to the challenges posed by
dynamic Earth or life processes. There are, of course, no easy answers to the question of how to gain
experience of ‘governing’ the forces of the Earth. As Latour argues, novel situations configured by messy admixtures
of social and material ingredients present a new imperative to improvise or experiment (see Farías, this volume). When
it comes to situations with the scale and complexity of global climate change , however, he suggests we are
way out of our depth: ‘The problem is that while we know how to conduct a scientific experiment in the narrow confines of a
laboratory, we have no idea how to pursue collective experiments in the confusing atmosphere of a
whole culture’ (Latour, 2003: 31). But who exactly the ‘we’ is in this statement raises questions of its own – inviting us to consider the
historical and geographical depth of the human experience of living through environmental extremes. One of the motivations for thinking
through geological durations, after all, is to contextualize the events of the present in a much broader framework. As philosopher-geologist
Robert Frodeman explains, ‘[e]arthquakes, floods, hurricanes, and droughts are places where deep time erupts into more familiar temporal
rhythms’ (2003: 125). If
such threshold transitions or destratifications might be seen as ways in which the Earth
experiments with its human (and non-human) inhabitants, they are equally occasions which oblige human
populations to respond with experiments of their own . Many of those peoples who still live in relatively close proximity to
the rhythms and upheavals of the Earth have learned how best to shelter from extreme events, when to move to safer ground, how to channel
excess energies, what to cache or stockpile, and when to fight fire with fire (Clark, 2008; 2011). The
shaping of such practices and
the decisions out of which they are forged might well be seen as a form of geologic politics – though this is
not necessarily ‘politics’ which is played out in the patient, deliberative manner that social theorists such as Latour or Ulrich Beck (1995) prefer
(see Michael, this volume). As philosophers Gilles Deleuze and Felix Guattari (1987) suggest, our
engagement with the
organizational layerings and dynamics of our material worlds can be more than reactive or defensive .
There is always the possibility of constructive traversals of compositional strata , of intercession in the flows of
matter and energy, with no purpose other than the joy of experimentation and the pleasure of creating

new forms and structures . At the same time, Deleuze and Guattari counsel about the dangers of ‘a too-
sudden destratification’, warning that this ‘will sometimes end in chaos , the void and destruction ,
and sometimes lock us back into the strata’ (1987: 503). If this cautionary note applies in a general sense
to the planetary predicament that results from unrestricted consumption of fossil fuels, so too is it
apposite with regard to strategies for deliberate geotechnical interventions into Earth systems – not least
the kind of unauthorized geoengineering experiment that recently took place off the Canadian coast (Geere, 2012). While geoengineering
proposals have justifiably attracted critical scrutiny, they have in the process helped put practical experimentation with
dynamic Earth processes more explicitly on the academic and the political agenda (see Galarraga and
Szerszynski, 2012). Today, alongside speculative planet-scaled ‘smoke and mirrors’ geoengineering schemes (Humphreys, 2011), a host of

more moderately scaled and easily reversible strategies for intervening in Earth systems are
currently under experiment and review. These include localized alterations of the planet's albedo involving brightening of water
and transformations of vegetative cover or the built environment, a range of forms of biological and geological carbon capture such as soil
enhancement using charred organic matter, and a whole raft of proposals to protect and enhance ecosystems (Olson, 2012; see also Chris,
2013). Such strategies are of interest not because they promise quick solutions to climate change and
other Earth system threshold problems, but because they give an idea of the possible mix of techno-
physical and socio-political issues that may characterize emergent ‘geo-political’ agendas . They direct
our attention not only to the kind of material interventions over which collective decisions must be
made, but to the need for political constituencies to consider their own everyday practical or material
implication in the dynamics of Earth systems – to ask how they themselves might take matter-energy
flows into their own hands. And this implies that, just as critical social thinkers increasingly demand political
awareness on the part of Earth systems scientists and engineers, so too must we require of ourselves a willingness to
commit to some form of experimental intervention in Earth processes – with all the risks this
inevitably entails.
Policy Good/Alt Fails
Enviro Policy Key
Students interrogating environmental issues through policy debate is critical to
developing sustainable solutions
Cotgrave and Alkhaddar 6 – Alison Cotgrave has a PhD in Sustainability Literacy, she is currently the
Deputy Director of the School of the Built Environment and a researcher in construction education, she
is also a Fellow of the Higher Education Academy, Rafid Alkhaddar has a PhD in Civil Engineering and
currently teaches at the School of the Built Environment John Moores University in Liverpool as a
Professor of Water and Environmental Engineering (March 2006, “Greening the Curricula within
Construction Programmes,” Journal for Education in the Built Environment, Vol.1, Issue 1, March 2006
pp. 3-29, http://131.251.248.49/jebe/pdf/AlisonCotgrave1(1).pdf)

Environmental education Many writers have determined that the main aim of environmental education is to
change attitudes, that will in turn change behaviour . As long ago as 1976, Ramsey and Rickson identified that it has long
been known that the basis for many environmental problems is irresponsible behaviour. Without a doubt, one
of the most important influences on behaviour is attitude, that in turn is influenced by education .
Campbell Bradley et al. (1999) stress the need for trying to change young people’s environmental attitudes

because young people ultimately will be affected by, and will need to provide, solutions to environmental
problems arising from present day actions . As future policymakers, the youth of today will be
responsible for ‘fixing’ the environment and they will be the ones who must be persuaded to act now
in order to avoid paying a high price to repair damage to the environment in the future , if indeed it is repairable.
Therefore it appears that effective environmental education, which changes the attitudes of young
people, is crucial. The (then) Department for Education (DFE) report, commonly known as the ‘Toyne Report’ (DFE, 1993), concluded
that as education seeks to lead opinion, it will do so more effectively if it keeps in mind the distinctive nature of its mission, which is first and
foremost to improve its students’ understanding. Their concern may well be awakened as a result; but it must be a properly informed concern.
This does not necessarily mean treating the environment as a purely scientific issue, but does mean that the respective roles of science and
ethics need to be distinguished, and the complexities
of each need to be acknowledged. Failure to do this may
lead all too readily to an ‘environmentalism’ which, by depicting possibilities as certainties, can only
discredit itself in the long run and feed the complacency which it seeks to dispel . McKeown-Ice and Dendinger
(2000) have identified the fact that scientific knowledge and political intervention will not solve the environmental problem on their own, thus
implying that something additional is required to change behaviour. As has already been discussed, behaviour changes can only
occur if attitudes change and this can be achieved through education . As Fien (1997) identifies,
environmental education can play a key role by creating awareness, and changing people’s values,
skills and behaviour . Introducing environmental elements into the curriculum can therefore be seen
as a potentially effective way of transferring knowledge. This should in turn improve attitudes that
will lead to improvements in environmental behaviour. Graham (2000) believes that it is crucial that building
professionals not only participate in the creation of projects that have low environmental impact, but
equally it is important that they learn to conceive, nurture, promote and facilitate the kind of
paradigm changes seen as necessary to create a sustainable society . There are however limitations as to what
education can achieve on its own, for as Jucker (2002) believes, if we do not do everything we can to transform our political, economic and
social systems into more sustainable structures, we might as well forget the educational part.
Only through public research and debate can we deconstruct centers of power who
prevent change from happening through use of the media; this is especially true for
the environment
Dispensa, 3 (Media's social construction of environmental issues: Focus on global warming - a
comparative study Dispensa, Jaclyn Marisa;Brulle, Robert J The International Journal of Sociology and
Social Policy; 2003; 23, 10; ABI/INFORM Complete)
This research has demonstrated that different interests and openness of states allows for openness of media coverage, such as with global
warming. The U.S. tends to distort global warming in contrast to other countries’ presentation of the issue and in contrast
to scientists’ views on the issue. The U.S. economy is strongly tied into the fossil fuel industry; therefore it is
in their interest to present information to society members that global warming is not a serious problem.
We encourage further research in order to substantiate or not our conclusion. This should include the continued analyses of other U.S.
newspapers as well as newspapers worldwide. Additional research into the control of media is
needed to identify and expose
to the public the abusive powers that exist within the media power structure. This exposure will better
allow society members to criticize the information produced by the media as well as to research and
formulate their thoughts and opinions. Only then can members of society bring issues of importance
into the larger public arena rather than be dictated by the corporate and political forces on issues of
importance only to them.
Institutions Key
Institutional deliberation is valuable and motivate effective responses to climate risks
Marx et al 7 (Sabine M, Center for Research on Environmental Decisions (CRED) @ Columbia
University, Elke U. Weber, Graduate School of Business and Department of Psychology @ Columbia
University, Benjamin S. Orlovea, Department of Environmental Science and Policy @ University of
California Davis, Anthony Leiserowitz, Decision Research, David H. Krantz, Department of Psychology @
Columbia University, Carla Roncolia, South East Climate Consortium (SECC), Department of Biological
and Agricultural Engineering @ University of Georgia and Jennifer Phillips, Bard Centre for
Environmental Policy @ Bard College, “Communication and mental processes: Experiential and analytic
processing of uncertain climate information”, 2007,
http://climate.columbia.edu/sitefiles/file/Marx_GEC_2007.pdf)
Based on the observation that experiential and analytic processing systems compete and that personal experience and vivid descriptions are
often favored over statistical information, we suggest the following research and policy implications. Communications designed to create, recall
and highlight relevant personal experience and to elicit affective
responses can lead to more public attention to,
processing of, and engagement with forecasts of climate variability and climate change. Vicarious
experiential information in the form of scenarios, narratives, and analogies can help the public and
policy makers imagine the potential consequences of climate variability and change, amplify or
attenuate risk perceptions, and influence both individual behavioral intentions and public policy
preferences. Likewise, as illustrated by the example of retranslation in the Uganda studies, the translation of statistical
information into concrete experience with simulated forecasts, decisionmaking and its outcomes can greatly
facilitate an intuitive understanding of both probabilities and the consequences of incremental change
and extreme events, and motivate contingency planning. Yet, while the engagement of experience-based, affective
decision-making can make risk communications more salient and motivate behavior, experiential processing is also subject to its own biases,
limitations and distortions, such as the finite pool of worry and single action bias. Experiential processing works best with easily imaginable,
emotionally laden material, yet many aspects of climate variability and change are relatively abstract and require a certain level of analytical
understanding (e.g., long-term trends in mean temperatures or precipitation). Ideally, communication of climate
forecasts should
encourage the interactive engagement of both analytic and experiential processing systems in the course of making
concrete decisions about climate, ranging from individual choices about what crops to plant in a particular season to broad social
choices about how to mitigate or adapt to global climate change. One way to facilitate this interaction is through group and
participatory decision-making. As the Uganda example suggests, group processes allow individuals with a range
of knowledge, skills and personal experience to share diverse information and perspectives and work
together on a problem. Ideally, groups should include at least one member trained to understand statistical forecast information to
ensure that all sources of information—both experiential and analytic—are considered as part of the decision-making process. Communications
to groups should also try to translate statistical information into formats readily understood in the language, personal and cultural experience
of group members. In a somewhat iterative or cyclical process, the shared concrete information can then be re-
abstracted to an analytic level that leads to action. Risk and uncertainty are inherent dimensions of all
climate forecasts and related decisions. Analytic products like trend analysis, forecast probabilities, and
ranges of uncertainty ought to be valuable contributions to stakeholder decision-making . Yet decision
makers also listen to the inner and communal voices of personal and collective experience, affect and emotion, and cultural values. Both
systems—analytic and experiential—should be considered in the design of climate forecasts and risk
communications. If not, many analytic products will fall on deaf ears as decision makers continue to rely heavily on personal experience
and affective cues to make plans for an uncertain future. The challenge is to find innovative and creative ways to engage both systems in the
process of individual and group decision-making.
Tying individual action to institutional change is key to solving environmental issues
Eden, 96 (Sally Eden, Public participation in environmental policy: considering scientific, counter-
scientific and non-scientific contributions, Public Understanding of Science July 1996 vol. 5 no. 3 183-
204, http://pus.sagepub.com/content/5/3/183.full.pdf+html)
Under this globally agreed interpretation, environmental policy should aim to involve the public not only in its design, to
ensure thorough representation and consultation, but also in its implementation , to ensure that its targets are
met through the actions of all individuals. Of course, the two aspects are complementary: policy should be designed so that it
can be implemented by individuals, i.e. it must appreciate their views and constraints. Successful environmental policy
has therefore been linked to the notion of ‘concerned citizens’, coupling individual action to institutional
change in the name of environmental protection. 2 In the major UNCED statement, Agenda 21, there is considerable
reference to local and public participation, including participation by specific sections of the population, e.g. women, children, indigenous
peoples and farmers. Specifically, Chapter 8 notes the need for better dialogue and ‘mechanisms to facilitate the involvement of concerned
individuals, groups and organizations in decision-making at all levels’, and Chapter 28 says that all local authorities should consult with citizens
and local groups to build a consensus around a ‘local Agenda 21’ by 1996. This kind of policy emphasis has been interpreted to indicate that ‘a
new language of “empowerment”, “citizen participation” and “multi-stakeholder partnership”’ has been fostered by Agenda 21. 3 But such
‘empowerment’ is not clearly built into policy formulation. Often,
public participation is relegated to the discussion of
awareness and education, which implies passive absorption of information rather than active
consultation. There is therefore apparently a difference between empowering people to know and empowering them to act which often
goes unappreciated in environmental policy. For example, Chapter 36 of Agenda 21 begins by noting ‘a considerable lack of awareness of the
interrelated nature of all human activities and the environment, due to inaccurate or insufficient information’ (emphasis added), and primarily
concerns itself with how to promote more appropriate values and behaviour through awareness and education. Participation is limited to
specific structural changes within established channels of consultation and these reach out to non-governmental organizations (NGOs) rather
than to the general public.
Policy Key
Policy simulation is good – promotes action and can’t solve without politics
Hodson 10 Derek, professor of education – Ontario Institute for Studies @ University of Toronto,
“Science Education as a Call to Action,” Canadian Journal of Science, Mathematics and Technology
Education, Vol. 10, Issue 3, p. 197-206
**note: SSI = socioscientific issues
The final (fourth) level of sophistication in this issues-based approach is concerned with students findings ways of
putting their values and convictions into action, helping them to prepare for and engage in
responsible action, and assisting them in developing the skills , attitudes, and values that will enable them to
take control of their lives, cooperate with others to bring about change , and work toward a more just and
sustainable world in which power, wealth, and resources are more equitably shared. Socially and
environmentally responsible behavior will not necessarily follow from knowledge of key concepts and possession of the “right attitudes.” As
Curtin (1991) reminded us, it is important to distinguish between caring about and caring for. It is almost always much easier to proclaim that
one cares about an issue than to do something about it. Put simply, our values are worth nothing until we live them. Rhetoric and espoused
values will not bring about social justice and will not save the planet. We must change our actions. A politicized
ethic of care (caring
for) entails active involvement in a local manifestation of a particular problem or issue, exploration of the
complex sociopolitical contexts in which the problem/issue is located, and attempts to resolve conflicts
of interest. FROM STSE RHETORIC TO SOCIOPOLITICAL ACTION Writing from the perspective of environmental education, Jensen (2002)
categorized the knowledge that is likely to promote sociopolitical action and encourage pro-environmental behavior
into four dimensions: (a) scientific and technological knowledge that informs the issue or problem; (b)
knowledge about the underlying social, political, and economic issues , conditions, and structures and how
they contribute to creating social and environmental problems; (c) knowledge about how to bring about changes in society
through direct or indirect action; and (d) knowledge about the likely outcome or direction of possible actions and
the desirability of those outcomes. Although formulated as a model for environmental education, it is reasonable to suppose that
Jensen's arguments are applicable to all forms of SSI-oriented action. Little needs to be said about dimensions 1 and 2 in Jensen's framework
beyond the discussion earlier in the article. With regard to dimension 3, students
need knowledge of actions that are likely
to have positive impact and knowledge of how to engage in them. It is essential that they gain robust
knowledge of the social, legal, and political system(s) that prevail in the communities in which they live and develop
a clear understanding of how decisions are made within local, regional, and national government and within
industry, commerce, and the military. Without knowledge of where and with whom power of decision
making is located and awareness of the mechanisms by which decisions are reached , intervention is
not possible. Thus, the curriculum I propose requires a concurrent program designed to achieve a measure of political
literacy, including knowledge of how to engage in collective action with individuals who have different
competencies, backgrounds, and attitudes but share a common interest in a particular SSI. Dimension 3 also includes
knowledge of likely sympathizers and potential allies and strategies for encouraging cooperative action and group interventions. What Jensen
did not mention but would seem to be a part of dimension 3 knowledge is the nature of science-oriented knowledge that would
enable students to appraise the statements, reports, and arguments of scientists, politicians, and
journalists and to present their own supporting or opposing arguments in a coherent, robust, and
convincing way (see Hodson [2009b] for a lengthy discussion of this aspect of science education). Jensen's fourth category includes
awareness of how (and why) others have sought to bring about change and entails formulation of a vision of the kind of world in which we (and
our families and communities) wish to live. It is important for students to explore and develop their ideas, dreams, and aspirations for
themselves, their neighbors and families and for the wider communities at local, regional, national, and global levels—a clear overlap with
futures studies/education. An essential step in cultivating the critical scientific and technological literacy on
which sociopolitical action depends is the application of a social and political critique capable of challenging
the notion of technological determinism. We can control technology and its environmental and social impact. More significantly, we can
control the controllers and redirect
technology in such a way that adverse environmental impact is
substantially reduced (if not entirely eliminated) and issues of freedom, equality, and justice are kept in
the forefront of discussion during the establishment of policy .
Nuclear Policy Good
Students must debate nuclear policy --- next generation’s experts solve extinction
Douglas Shaw 9, associate dean for planning, research, and external relations & assistant professor of
international affairs at George Washington University's Elliott School of International Affairs & formerly
worked for the Arms Control and Disarmament Agency and Energy Department, “Reintroducing arms
control to higher education”, Bulletin of the Atomic Scientists, 5-26-09,
http://www.thebulletin.org/web-edition/op-eds/reintroducing-arms-control-to-higher-education
The first set of tensions involves the transformation of nonproliferation regime institutions. This comes, in part, from the temptation
to look backward in nuclear negotiations. The U.N. General Assembly has failed to control nuclear weapons. The
Comprehensive Test Ban Treaty and other commitments embedded in the Nuclear Non-Proliferation Treaty (NPT) are years
overdue. The "thirteen steps" on a practical path toward nuclear disarmament identified at the 2000 NPT Review Conference are a good
start, but there have been few new ideas to realize the potential of technological, political, and social developments. Higher
education must educate a next generation to look forward in examining these institutions. Prudent
and verifiable progress may include new fora for negotiations, new governmental structures, new issue
linkages, and new technologies and procedures for enhancing global confidence. The second set of
tensions involves universality. In reality, the exclusive superpower prerogative over the nuclear future ended long ago. Every human
being is threatened by nuclear weapons and has a legitimate stake in nuclear negotiations . But
vastly more people need to understand these topics in order to create a global order that can
control nuclear weapons permanently. Moving forward, useful negotiations will involve an increasing number of parties--
and this must extend beyond inviting the British, French, and Chinese to participate directly in U.S.-Russian strategic arms reduction
negotiations or the pursuit of a global nuclear weapons convention. More far-reaching and innovative
solutions must be put
forward. For example, we might consider how follow-on generations of nuclear safeguard enhancements might expand the use of
transparency. In addition, we might consider confidence-building measures that enhance global verification in the arms reduction process
or reinforce nuclear weapon states' negative security assurances. Peaceful uses of nuclear energy encompass a third group of tensions. The
prospect of "power too cheap to meter" has tantalized leaders into compromises about proliferation risk since the dawn of the nuclear
age. President Dwight D. Eisenhower's vision of "Atoms for Peace" led to these compromises being written into the NPT and the mandate
of the International Atomic Energy Agency. Internationalization of the nuclear fuel supply, the dilution of international safeguards to suit
any one state, the spread of nuclear power to additional countries, and the widening understanding of plutonium as an energy resource
may each have a purpose, but they also imply identifiable risks for the future. Going
forward, experts must be trained to
assess these current challenges to nuclear energy. They must also look further afield and learn to
examine the effects climate change and oil dependence will have on future proliferation
compromises since such new risks will undoubtedly accompany any "nuclear renaissance." Most
importantly, a fourth group of tensions involves deterrence stability on the way to zero. Deterrence isn't a reliable piece of hardware, so
we must be increasingly clear about why we have nuclear weapons, what we imagine destroying, how many we need available on short
notice, and how others will react to our choices. Currently, Al Qaeda aims to provoke the United States to overreact, and at the same time,
is attempting to convince the world the United States must be resisted. But in a key moment we may find that the fear nuclear weapons
are built to instill doesn't necessarily serve our interests. The
perceptions of allies and billions of innocent bystanders too
often are assumed irrelevant or even requiring a larger nuclear arsenal for "extended deterrence." Looking forward , it is
incumbent that the soundness and costs of each of these assumptions are continuously tested and
improved. The trade-offs between uncertain paths forward should be explicitly debated both by
today's experts and tomorrow's nascent explorers. These tensions of zero--institutional transformation,
universality, peaceful uses of nuclear energy, and deterrence--will never be cleanly resolved . But if
we're lucky, we will be managing them long after the legal abolition of nuclear weapons. Learning to
do so effectively is the work of a generation, and we are a generation behind in preparing our best
and brightest for this work. This suggests an intimidating, but attainable, goal for higher education
institutions.
Debate Solves
Debate solves state fear-mongering – scenario planning counteracts misinformation –
the alt creates an apathetic public
Kurasawa, Sociology Professor at York University, ‘4 (Fuyuki, “Cautionary Tales: The Global
Culture of Prevention and the Work of Foresight” Constellations, Vol 11 No 4)

State and market institutions may seek to produce a culture of fear by deliberately stretching
interpretations of reality beyond the limits of the plausible so as to exaggerate the prospects of
impending catastrophes, or yet again, by intentionally promoting certain prognoses over others for
instrumental purposes. Accordingly, regressive dystopias can operate as Trojan horses advancing political agendas or commercial
interests that would otherwise be susceptible to public scrutiny and opposition. Instances of this kind of manipulation of the dystopian
imaginary are plentiful: the invasion of Iraq in the name of fighting terrorism and an imminent threat of use of ‘weapons of mass destruction’;
the severe curtailing of American civil liberties amidst fears of a collapse of ‘homeland security’; the neoliberal dismantling of the welfare state
as the only remedy for an ideologically constructed fiscal crisis; the conservative expansion of policing and incarceration due to supposedly
spiraling crime waves; and so forth. Alarmism constructs and codes the future in particular ways, producing or reinforcing certain crisis
narratives, belief structures, and rhetorical conventions. As much as alarmist ideas beget a culture of fear, the reverse is no less true. If fear-
mongering is a misappropriation of preventive foresight, resignation about the future represents a
problematic outgrowth of the popular acknowledgment of global perils. Some believe that the world to come is so
uncertain and dangerous that we should not attempt to modify the course of history; the future will look after itself for better or
worse, regardless of what we do or wish. One version of this argument consists in a complacent optimism perceiving the future as fated to be
better than either the past or the present. Frequently accompanying it is a self-deluding denial of what is plausible (‘the world will not be so bad
after all’), or a naively Panglossian pragmatism (‘things will work themselves out in spite of everything, because humankind always finds ways to
survive’).37 Much more common, however, is the opposite reaction, a fatalistic pessimism reconciled to the idea that the future will be
necessarily worse than what preceded it. This is sustained by a tragic chronological framework according to which humanity is doomed to
decay, or a cyclical one of the endless repetition of the mistakes of the past. On top of their dubious assessments of what is to come,
alarmism and resignation would, if widely accepted, undermine a viable practice of farsightedness.
Indeed, both of them encourage public disengagement from deliberation about scenarios for the future ,
a process that appears to be dangerous, pointless, or unnecessary. The resulting ‘depublicization’ of debate leaves
dominant groups and institutions (the state, the market, techno-science) in charge of sorting out the
future for the rest of us, thus effectively producing a heteronomous social order . How, then, can we support a
democratic process of prevention from below? The answer, I think, lies in cultivating the public capacity for critical
judgment and deliberation, so that participants in global civil society subject all claims about potential
catastrophes to examination, evaluation, and contestation .

Simulation allows us to influence state policy and is key to agency


Eijkman 12 (The role of simulations in the authentic learning for national security policy development:
Implications for Practice / Dr. Henk Simon Eijkman. [electronic resource]
http://nsc.anu.edu.au/test/documents/Sims_in_authentic_learning_report.pdf. Dr Henk Eijkman is
currently an independent consultant as well as visiting fellow at the University of New South Wales at
the Australian Defence Force Academy and is Visiting Professor of Academic Development, Annasaheb
Dange College of Engineering and Technology in India. As a sociologist he developed an active interest in
tertiary learning and teaching with a focus on socially inclusive innovation and culture change. He has
taught at various institutions in the social sciences and his work as an adult learning specialist has taken
him to South Africa, Malaysia, Palestine, and India. He publishes widely in international journals, serves
on Conference Committees and editorial boards of edited books and international journal)
However, whether as an approach to learning, innovation, persuasion or culture shift, policy simulations derive their power
from two central features: their combination of simulation and gaming (Geurts et al. 2007). 1. The simulation element: the
unique combination of simulation with role-playing .The unique simulation/role-play mix enables participants to
create possible futures relevant to the topic being studied. This is diametrically opposed to the more
traditional, teacher-centric approaches in which a future is produced for them. In policy simulations,
possible futures are much more than an object of tabletop discussion and verbal speculation. ‘No other
technique allows a group of participants to engage in collective action in a safe environment to create
and analyse the futures they want to explore’ (Geurts et al. 2007: 536). 2. The game element: the interactive and tailor-
made modelling and design of the policy game. The actual run of the policy simulation is only one step, though a most important and visible
one, in a collective process of investigation, communication, and evaluation of performance. In the context of a post-graduate course in public
policy development, for example, a
policy simulation is a dedicated game constructed in collaboration with
practitioners to achieve a high level of proficiency in relevant aspects of the policy development process. To drill down to a
development simulations —as forms of interactive or participatory modelling— are
level of finer detail, policy
particularly effective in developing participant knowledge and skills in the five key areas of the policy development
process (and success criteria), namely: Complexity, Communication, Creativity, Consensus, and Commitment to action (‘the five Cs’). The
capacity to provide effective learning support in these five categories has proved to be particularly helpful in strategic decision-making (Geurts
et al. 2007). Annexure 2.5 contains a detailed description, in table format, of the synopsis below.
Alt Fails – Action Key
Prior questions can’t be settled --- action under uncertainty is necessary
Molly Cochran 99, Assistant Professor of International Affairs at Georgia Institute for Technology,
“Normative Theory in International Relations”, 1999, pg. 272

To conclude this chapter, while modernist and postmodernist


debates continue, while we are still unsure as to
what we can legitimately identify as a feminist ethical/political concern, while we still are unclear about
the relationship between discourse and experience , it is particularly important for feminists that we
proceed with analysis of both the material (institutional and structural) as well as the discursive. This holds not only for
feminists, but for all theorists oriented towards the goal of extending further moral inclusion in the
present social sciences climate of epistemological uncertainty. Important ethical/ political concerns
hang in the balance. We cannot afford to wait for the meta-theoretical questions to be conclusively
answered . Those answers may be unavailable. Nor can we wait for a credible vision of an alternative
institutional orderto appear before an emancipatory agenda can be kicked into gear. Nor do we have before
us a chicken and egg question of which comes first: sorting out the metatheoretical issues or working out
which practices contribute to a credible institutional vision. The two questions can and should be
pursued together, and can be via moral imagination. Imagination can help us think beyond discursive and material conditions
which limit us, by pushing the boundaries of those limitations in thought and examining what yields. In this respect, I believe international
ethics as pragmatic critique can be a useful ally to feminist and normative theorists generally.
Alt Fails – Transition Wars
Transition to the alternative guarantees war – radical changes in existing security
architecture collapse threat perception
Yoon 03 – Professor of International Relations at Seoul National University; former Foreign Minister of
South Korea (Young-Kwan, “Introduction: Power Cycle Theory and the Practice of International
Relations”, International Political Science Review 2003; vol. 24; p. 7-8)

In history, the effort to balance power quite often tended to start too late to protect the security of
some of the individual states. If the balancing process begins too late , the resulting amount of force
necessary to stop an aggressor is often much larger than if the process had been started much earlier . For
example, the fate of Czechoslovakia and Poland showed how non-intervention or waiting for the
“automatic” working through of the process turned out to be problematic. Power cycle theory could also
supplement the structure-oriented nature of the traditional balance of power theory by incorporating
an agent-oriented explanation. This was possible through its focus on the relationship between power and the role of a state in the
international system. It especially highlighted the fact that a discrepancy between the relative power of a state and its role in the system would
result in a greater possibility for systemic instability. In order to prevent this instability from developing into a war, practitioners of international
relations were to become aware of the dynamics of changing power and role, adjusting role to power. A statesperson here was not simply
regarded as a prisoner of structure and therefore as an outsider to the process but as an agent capable of influencing the operation of
equilibrium. Thus power cycle theory could overcome the weakness of theoretical determinism associated with the traditional balance of
power. The question is often raised whether government decision-makers could possibly know or respond to such relative power shifts in the
real world. According to Doran, when the “tides of history” shift against the state, the push and shove of world politics reveals these matters to
the policy-maker, in that state and among its competitors, with abundant urgency. (2) The Issue of Systemic Stability Power cycle theory is built
on the conception of changing relative capabilities of a state, and as such it shares the realist assumption emphasizing the importance of power
in explaining international relations. But its main focus is on the longitudinal dimension of power relations, the rise and decline of relative state
power and role, and not on the static power distribution at a particular time. As a result, power cycle theory provides a significantly different
explanation for stability and order within the international system. First of all, power cycle theory argues that what matters
most in explaining the stability of the international system or war and peace is not the type of particular
international system (Rosecrance, 1963) but the transformation from one system to another . For example, in the
1960s there was a debate on the stability of the international system between the defenders of
bipolarity such as Waltz (1964) and the defenders of multi-polarity such as Rosecrance (1966), and Deutsch and
Singer (1964). After analyzing five historical occasions since the origin of the modern state system, Doran concluded that what has
been responsible for major war was not whether one type of system is more or less conducive to war
but that instead systems transformation itself led to war (Doran, 1971). A non-linear type of structural
change that is massive, unpredicted, devastating to foreign policy expectation, and destructive of
security is the trigger for major war, not the nature of a particular type of international system .
Epistemology Answers
No Root Cause
Root cause logic is reductive and false —it subverts change
Gary Morson 7, Professor of Slavic Studies, Russian Literature and History at Northwestern, “Anna Karenina In Our Time:
Seeing More Wisely,” P. 152-4

If Levin resembled so may intellectuals in his time and ours, he might seek “root cause” (as we would call it today) of all
these failures. Much as the generals an historians satirized in War and Peace mistakenly seek the cause of historical events in a single

decision, an much as revolutionaries often reduce the complexities of social ills to a single conspiracy or
institution , so intellectuals often view complexity as a delusion to be explained away by a few simple
underlying laws. It is just this habit of thought that feeds utopianism, because if the diversity of evil
an misery had a single cause, then one could eliminate it by changing only one thing What could be
easier? Abolish private property, alter the way children are educated, pass laws to regulate morals according to a given code,
and evil will disappear or, at least, radically diminish. Behold, I make all new things But Levin learns that there is no single cause

for what has gone wrong. Looking back on the twentieth century , we may wonder whether the root cause of the
worst human misery is the belief that there is a root cause of human misery. In fact, many things happen
contingently, just “for some reason.”¶ Friction¶ When l.evin attends the elections, he tries to handle some business for his sister, but discovers that somehow it cannot be done. In
Dostoevsky, the reason would be "administrative ecstasy," the sheer delight bureaucrats take in making petitioners cringe, plead, or wait. But nothing of the sort happens here, and the
problem is not one of intent at all. No one has any interest in thwarting Levin, so he cannot understand what goes wrong.¶ When conspiracy theorists find they cannot accomplish
something as easily as expected, they typically ask cut bono? (who benefits?) ro discover the obstacle. Some person or group must have caused the failure. Defeat means sabotage. This
way of thinking presumes that behind every action there must be an intent,¶ whether conscious or unconscious. Such a view rules out the possibility that mere contingency or friction
accounts for the difficulty.¶ flic military theorist Carl von Clauscwitz deemed friction, in this special metaphorical sense, an essential concept in understanding armies. Without using this
word, Tolstoy regarded the same phenomenon as pertaining not just to war but to everything social. "If one has never personally experienced war," Clauscwitz explains,¶ one cannot
understand in what difficulties constantly mentioned really consist. . . . Everything looks simple; the knowledge required docs not look remarkable, the strategic options are so obvious
that by comparison the simplest problem of higher mathematics has an impressive scientific dignity. Once war has actually been seen the difficulties become clear; but it is extremely
difficult to describe the unseen, all-pervading element that brings about this change of perspective. Everything in war is very simple, but the simplest thing is difficult. 'Die difficulties
accumulate and end by producing a kind of friction that is inconceivable unless one has experienced war. (Clausewitz, 119)¶ The unseen, all-pervading element: For Tolstoy, similar
difficulties arise when dealing with bureaucracy, introducing changes in agriculture, and implementing reforms. A Tolstoyan perspective is easily imagined today. Social problems look so
simple: people in underdeveloped countries are poor, so give their governments foreign aid; workers arc unemployed, so hire them to perform needed government services; schools do
not educate, so raise teachers' salaries; the state regulatory commission keeps energy prices too high, so partially privatize the system: answers seem so obvious, but in practice reforms
rarely have the intended effect. They produce unintended consequences, which themselves have consequences; and, as Isaiah Berlin liked to point our, no one can foresee the
consequences of consequences of consequences. Experience may teach one to expect certain kinds of difficulties, but some can never be anticipated, lhcrc is always friction: "Countless
minor incidents —the kind you can never really foresee—combine to lower the general level of performance, so that one always falls far short of the intended goal" (Clauscwitz, 119).¶
No one is deliberately impeding Levin's efforts for his sister. By the same token, no one is trying to thwart his agricultural reforms. Sabotage is out of the question. "All this happened not
because anyone felt ill will toward Levin or his farm; on the contrary, he knew that they [rhe peasants] liked him [and] thought him a simple gentleman (their highest praise)" (340).¶
Friction defeats the reforms. But where does this friction come from and how might one best deal with it?¶ TTic Elemental Force¶ 'Ihe bailiff and peasants recognize in advance when a
plan is bound to fail, and at lasr l.evin, instead of growing angry, pays artention to what they say:¶ The bailiff listened attentively, and obviously made an effort to approve of his
employer's projects. But still he had that look Levin knew so well that always irritated him, a look of hopelessness and despondency. That look said: " Ihat's all very well, but as God wills."
Nothing mortified Levin so much as that tone. But it was common to all the bailiffs he had ever had. They had all taken up that attitude toward his plans, and so now he was not angered
by it but mortified, and felr all the more roused ro struggle against this, as it seemed, elemental force continually ranged against him, for which he could find no other expression than "as
God wills." (165)¶ Ihe elementalforce: this concept is central to both Tolstoy's great novels. Tolstoy uses a few similar terms for it. In War and Peace, he refers to an elemental force
shaping individual lives (W&P, 648) and to "the elemental life of the swarm" constituting the cumulative effect of countless people's small actions governed by no overarching law. In
Anna Karentna, he calls the elemental force a "brutal force" when its outcome is cruel. Ihe rough equivalent of friction for Clause-witz, the elemental force applies more widely.¶

Clauscwitz's explanation stops at friction, but Tolstoy takes the elemental force as a starting point for understanding why some plans arc more likely to fail than others.¶ In order
to grasp the course of events more easily, we tend to reduce the countless infinitesimal forces making up the

elemental force to a single cause . After all, it is impossible to enumerate innumerable actions. And so historians and social
scientists naturally look for some super-cause that sums up all those small actions. They may presume laws or

postulate narrative neatness. Tolstoy relentlessly exposed the logical fallacies in both forms of simplification , which, at some
point, either assume what is to be proven or proceed as if it were already proven . ¶ Historians, social
theorists, and biographers favor generalizations or symmetries permitting a clear analysis or simple story . They
find what they seek, their success demonstrates not that complexity has been adequately explained but that when a
discipline demands a certain sort of explanation it is bound to be “discovered.” In disciplines
pretending to be social sciences, it is repeatedly discovered that things are not as complex as they appear.
AT: Discourse Shapes Reality
Discourse doesn’t shape reality
Tuathail 96 (Gearóid, Professor of Government and International Affairs, Virginia Tech, The patterned
mess of history and the writing of critical geopolitics: a reply to Dalby, Political Geography 15:6/7, p 661-
5 http://www.nvc.vt.edu/toalg/Website/Publish/miscellaneous/DalbyResponse.htm)
While theoretical debates at academic conferences are important to academics, the discourse and concerns of foreign policy
decision makers are quite different, so different that they constitutes a distinctive problem-solving
theory-averse policy making sub-culture. There is a danger that academics assume that the discourses
they engage are more significant in the practice of foreign policy and the exercise of power than they
really are. This is not, however, to minimize the obvious importance of academia as a general institutional structure among many which
sustain certain epistemic communities in particular states. In general, I do not disagree with Dalby's fourth point about politics and discourse
except to note that his statement -- "Precisely because reality could be represented in particular ways political decisions could be taken, troops
and material moved and war fought" -- evades the important question of agency I noted in my review essay. The
assumption that it is
representations that make action possible is inadequate by itself . Political, military and economic
structures, institutions, discursive networks and leadership are all crucial in explaining social action and should be
theorized together with representational practices. Both here and earlier, Dalby's reasoning inclines towards a form of idealism. In
response to Dalby's fifth point (with its three subpoints), it is worth noting, first, that his book is about the CPD not the Reagan administration.
He analyzes certain CPD discourses not the geographical reasoning practices of the Reagan administration nor its public policy reasoning on
national security. Dalby's book is narrowly textual; the general contextuality of the Reagan administration is not dealt with in the book. Second,
let me simply note that I find that the distinction between critical theorists and post-structuralists is a little too rigidly and heroically drawn by
Dalby and others. Third, Dalby's interpretation of the reconceptualization of national security in Moscow as heavily influenced by dissident
peace researchers in Europe is highly idealist, an interpretation that ignores the structural and ideological crises facing the Soviet elite at that
time. Gorbachev's reforms and his new security discourse were also strongly self-interested, an ultimately futile attempt to save the Communist
Party and a discredited regime of power from disintegration. The issues raised by Simon Dalby in his comment are important ones for all those
interested in the practice of critical geopolitics. While I agree with Dalby that questions of discourse are extremely important ones for political
geographer's to engage, there
is a danger of fetishizing this concern with discourse so that we neglect the
institutional and the sociological, the materialist and the cultural, the political and the geographical
contexts within which particular discursive strategies become significant. Critical geopolitics, in other words,
should not be a prisoner of the sweeping ahistorical cant that sometimes accompanies "post-structuralism" nor convenient reading strategies
like the identity politics narrative; it needs to always be open to the patterned mess that is human history.
AT: Reps Cause War
Reps don’t cause war – their argument is empirically false
Reiter 95 DAN REITER is a Professor of Political Science at Emory University and has been an Olin post-
doctoral fellow in security studies at Harvard “Exploring the Powder Keg Myth” International Security
v20 No2 Autumn 1995 pp 5-34 JSTOR
A criticism of assessing the frequency of preemptive wars by looking only at wars themselves is that this misses the non-events, that is, instances in which preemption would be predicted but did not occur. However, excluding non-events should bias the results in favor of finding

Therefore, if preemptive wars seem infrequent


that preemptive war is an important path to war, as the inclusion of non-events could only make it seem that the event was less frequent.

within the set of wars alone, then this would have to be considered strong evidence in favor of the third,

most skeptical view of preemptive war even when the sample is rigged to make preemptive wars , because

seem frequent they are still rare events


(by including only wars), ¶ The rarity . Below, a few cases in which preemption did not occur are discussed to illustrate factors that constrain preemption.

of preemptive wars offers preliminary support for the view, that the preemption scenario third, most skeptical

does not tell us much about how war breaks out Closer examination . of the three cases of preemption, set forth below, casts doubt
on the validity of the two preemption hypotheses discussed earlier: that hostile images of the enemy increase the chances
of preemption , and that belief in the dominance of the offense increases the chances of preemption. In each case there are motives for war aside from fear of an imminent attack, indicating that such fears may not
be sufficient to cause war . In addition, in these cases of war the conditions hypothesized to stimulate preemption—
two

hostile images of the adversary and belief in the military advantages of striking first— are present to a very high degree . This implies that these
are insubstantial causal forces , as they are associated with the outbreak of war only when they are present to a very high degree. This reduces even further the
significance of these forces as causes of war. To illustrate this point, consider an analogy: say there is a hypothesis that saccharin causes cancer. Discovering that rats who were fed a lot of

saccharin and also received high levels of X-ray exposure, which we know causes cancer, had a higher risk for cancer does not, however, set off alarm bells about the risks of saccharin. Though there might be a relationship between saccharin consumption

and cancer, this is not demonstrated by the results of such a test.


AT: Epistemology First
There are no prior questions in IR – epistemology fails and prevents action
Owen 2 (David, Reader of Political Theory at the University of Southampton, Reader of Political Theory at the
Univ. of Southampton, Millennium Vol 31 No 3 p. 655-657)
Commenting on the ‘philosophical turn’ in IR, Wæver remarks that ‘[a] frenzy for words like “epistemology” and
“ontology” often signals this philosophical turn’, although he goes on to comment that these terms are often used loosely.4
However, loosely deployed or not, it is clear that debates concerning ontology and epistemology play a central role in the contemporary IR
theory wars. In one respect, this is unsurprising since it is a characteristic feature of the social sciences that periods of disciplinary disorientation
involve recourse to reflection on the philosophical commitments of different theoretical approaches, and there is no doubt that such reflection
can play a valuable role in making explicit the commitments that characterise (and help individuate) diverse theoretical positions. Yet, such a
philosophical turn is not without its dangers and I will briefly mention three before turning to consider a confusion that has, I will
suggest, helped to promote the IR theory wars by motivating this philosophical turn. The first danger with the philosophical turn is that it has
an inbuilt tendency to prioritize issues of ontology and epistemology over explanatory and/or
interpretive power as if the latter two were merely a simple function of the former. But while the
explanatory and/or interpretive power of a theoretical account is not wholly independent of its ontological and/or
epistemological commitments (otherwise criticism of these features would not be a criticism that had any value), it is by no means clear that it
is, in contrast, wholly dependent on these philosophical commitme nts. Thus, for example, one need not be sympathetic
to rational choice theory to recognise that it can provide powerful accounts of certain kinds of problems, such as the tragedy of the commons in
It may, of course, be the case that the advocates of rational choice
which dilemmas of collective action are foregrounded.
theory cannot give a good account of why this type of theory is powerful in accounting for this class of problems
(i.e., how it is that the relevant actors come to exhibit features in these circumstances that approximate the assumptions of rational choice
theory) and, if this is the case, it is a philosophical weakness—but this does not undermine the point that,
for a certain class of problems, rational choice theory may provide the best account available to us. In
other words, while the critical judgement of theoretical accounts in terms of their ontological and/or
epistemological sophistication is one kind of critical judgement, it is not the only or even necessarily
the most important kind. The second danger run by the philosophical turn is that because prioritisation of ontology
and epistemology promotes theory-construction from philosophical first principles, it cultivates a theory-driven
rather than problem-driven approach to IR. Paraphrasing Ian Shapiro, the point can be put like this: since it is the case that
there is always a plurality of possible true descriptions of a given action, event or phenomenon, the challenge is
to decide which is the most apt in terms of getting a perspicuous grip on the action, event or phenomenon in
question given the purposes of the inquiry; yet, from this standpoint, ‘theory-driven work is part of a reductionist
program’ in that it ‘dictates always opting for the description that calls for the explanation that flows
from the preferred model or theory’.5 The justification offered for this strategy rests on the mistaken belief
that it is necessary for social science because general explanations are required to characterise the
classes of phenomena studied in similar terms. However, as Shapiro points out, this is to misunderstand the
enterprise of science since ‘whether there are general explanations for classes of phenomena is a
question for social-scientific inquiry, not to be prejudged before conducting that inquiry’ .6 Moreover, this
strategy easily slips into the promotion of the pursuit of generality over that of empirical validity. The
third danger is that the preceding two combine to encourage the formation of a particular image of disciplinary debate in
IR—what might be called (only slightly tongue in cheek) ‘the Highlander view’—namely, an image of warring theoretical
approaches with each, despite occasional temporary tactical alliances, dedicated to the strategic achievement of sovereignty over the
disciplinary field. It encourages this view because the turn to, and prioritisation of, ontology and epistemology
stimulates the idea that there can only be one theoretical approach which gets things right , namely, the
theoretical approach that gets its ontology and epistemology right. This image feeds back into IR exacerbating the first
and second dangers, and so a potentially vicious circle arises.
AT: Biased Evidence
Academic public policy research prevents political elite cooption
Walt, IR Prof at Harvard, ’11 (Stephen, July 21, “International Affairs and the Public Sphere”
Institute for Public Knowledge, http://publicsphere.ssrc.org/walt-international-affairs-and-the-public-
sphere/)

There is today no shortage of global problems that social scientists should study in depth: ethnic and
religious conflict within and between states, the challenge of economic development, terrorism, the management of a fragile world
economy, climate change and other forms of environmental degradation, the origins and impact of great power rivalries, the
spread of weapons of mass destruction, just to mention a few. In this complex and contentious world, one might think that
academic expertise about global affairs would be a highly valued commodity. Scholars would strive to
produce useful knowledge, students would flock to courses that helped them understand the world in
which they will live and work, and policymakers and the broader public would be eager to hear what
academic experts had to say. One might also expect scholars of international relations to play a
prominent role in public debates about foreign policy, along with government officials, business interests,
representatives of special interest groups, and other concerned citizens. Social scientists are far from omniscient, but the rigor of
the scientific process and the core values of academia should give university-based scholars an
especially valuable role within the broader public discourse on world affairs . At its best, academic
scholarship privileges creativity, validity, accuracy, and rigor and places little explicit value on political
expediency. The norms and procedures of the academic profession make it less likely that scholarly
work will be tailored to fit pre-conceived political agendas . When this does occur, the self-correcting
nature of academic research makes it more likely that politically motivated biases or other sources of
error will be exposed. Although we know that scholarly communities do not always live up to this ideal picture, the existence of
these basic norms gives the academic world some important advantages over think tanks, media
pundits, and other knowledge-producing institutions.
AT: Serial Policy Failure
No serial policy failure – multiplicity of meanings and discourse check back any effect
words have on our policymaking
Harvey 97 (Frank, Associate Prof. Pol. Sci. – Dalhousie U., “The Future’s Back: Nuclear Rivalry,
Deterrence Theory, and Crisis Stability after the Cold War”, p. 138-139)
Linguistic Relativism. One approach of postmodernists is to point to the complex nature of language and meaning as a critique of positiv¬ism;
this critique is, in turn, relevant to the overwhelming amount of work in IR (Phillips 1977; Giddens 1979; George and Campbell 1990). Although
a comprehensive assessment of the linguistic relativism debate is beyond the scope of this project, it is possible to address the underlying
philosophical argument, which is fairly straightforward. Building on the work of Wittgenstein (1968), the linguistic variant
of the criticism contends that any attempt to reduce  everyday terms "to a singular essentialist meaning" is
problematic given "the multiplicity of meaning  to be found in social activity" (George and Campbell 1990, 273). By
implication, a concept, term, word, or symbol cannot correspond "to some  ...externally derived foundation or object" and
ulti¬mately is context-dependent. Similarly, Phillips argues that the validity of theory cannot be determined because
"There is no standard or objective reality  (always fixed, never changing) against which to com¬pare a universe of discourse ...
nothing exists outside of our language and actions which can be used to justify ... a statement's truth or falsity" (1977, 273). Of course, it is
not entirely clear how this "multiplicity of meaning" is sufficient to render meaningless an approach that
assumes the existence of an objective reality . An important distinction must be drawn between the
assertion that these discrepancies might have a significant impact on scientific theorizing and the
assertion that they do have such an effect.  In most cases, errors of interpretation and generalization
produced by linguistic nuances are relatively insignificant and ultimately have very little impact on the
generalizability of social theories. There are numerous words, symbols, concepts, and ideas, for
example, that are commonly understood, regardless of other linguistic variations, but the implications of
this standardized concep¬tual framework are frequently overlooked and ignored in the post¬modern
critique. In any case, it is contingent upon the theorist to specify the precise meaning of any variable or symbol that is central to a theory.
Although definitions may vary — possibly partly, but not entirely, as a conse¬quence of language — scholars nevertheless are more
likely than not to understand and agree on the underlying meaning of most words, symbols and phrases .
The point is that theorists generally do have a common starting point and often suspend , at least
temporarily, coun¬terproductive debates over meaning in order to shift emphasis towards the strength and
logical consistency of the theory itself , a more important issue that has nothing to do with language .
Evaluating the internal consistency of the central assumptions and propositions of a theory, that is, criticising from within, is likely to be more
conducive to theoretical progress than the alternative, which is to reject the idea of theory building entirely. Finally, the
lack of purity
and precision, another consequence of linguistic relativism, does not necessarily imply irrelevance of purpose or
approach. The study of international relations may not be exact,  given limitations noted by Wittgenstein and
others, but precision is a practical research problem, not an insurmountable barrier to progress . In fact, most
observers who point to the context-dependent nature of language are critical not so much of the social sciences but of the incorrect application
of scientific techniques to derive overly precise measurement of weakly developed concepts. Clearly, our understanding of the
causes of international conflict — and most notably war — has improved considerably as a consequence of
applying sound scientific methods and valid operationalizations  (Vasquez 1987, 1993). The alternative approach,
implicit in much of the postmodern literature, is to fully accept the inadequacy of positivism, throw one's hands up in failure , given
the complexity of the subject, and repudiate the entire enterprise. The most relevant question is whether we would know more or less about
international relations if we pursued that strategy.
Aff Outweighs
Extinction First
Moral equality means even a small risk of preventing extinction outweighs structural
violence – future generations
Bostrom, 2012 (Mar 6, Nick, director of the Future of Humanity Institute at Oxford, recipient of the
2009 Gannon Award, “We're Underestimating the Risk of Human Extinction,” interview with Ross
Andersen, freelance writer in D.C., http://www.theatlantic.com/technology/archive/2012/03/were-
underestimating-the-risk-of-human-extinction/253821/)

Some have argued that we ought to be directing our resources toward humanity's existing problems, rather
than future existential risks, because many of the latter are highly improbable. You have responded by suggesting that
existential risk mitigation may in fact be a dominant moral priority over the alleviation of present suffering .
Can you explain why? Bostrom: Well suppose you have a moral view that counts future people as being worth as
much as present people. You might say that fundamentally it doesn't matter whether someone exists at the
current time or at some future time, just as many people think that from a fundamental moral point of view, it doesn't
matter where somebody is spatially---somebody isn't automatically worth less because you move them to the moon or to Africa or
something. A human life is a human life . If you have that moral point of view that future generations matter in proportion to their
population numbers, then you get this very stark implication that existential risk mitigation has a much higher utility than
pretty much anything else that you could do . There are so many people that could come into existence in the
future if humanity survives this critical period of time---we might live for billions of years , our descendants
might colonize billions of solar systems, and there could be billions and billions times more people than exist
currently. Therefore, even a very small reduction in the probability of realizing this enormous good will tend
to outweigh even immense benefits like eliminating poverty or curing malaria , which would be tremendous under
ordinary standards.
AT: Structural Violence
War turns structural violence
Goldstein 1—Prof PoliSci @ American University, Joshua, War and Gender , P. 412
First, peace activists face a dilemma in thinking about causes of war and working for peace. Many peace scholars and activists support the
approach, "if you want peace, work for justice ". Then if one believes that sexism contributes to war, one can work for gender
justice specifically (perhaps among others) in order to pursue peace. This approach brings strategic allies to the peace movement (women,
labor, minorities), but rests on the assumption that injustices cause war. The evidence in this book suggests
that causality runs at least as strongly the other way. War is not a product of capitalism, imperialism,
gender, innate aggression, or any other single cause, although all of these influences wars' outbreaks
and outcomes. Rather, war has in part fueled and sustained these and other injustices . So, "if you want peace,
work for peace." Indeed, if you want justice (gener and others), work for peace. Causality does not run just upward through the
levels of analysis from types of individuals, societies, and governments up to war. It runs downward too. Enloe suggests that changes in
attitudes toward war and the military may be the most important way to "reverse women's
oppression/" The dilemma is that peace work focused on justice brings to the peace movement energy, allies and moral grounding, yet, in
light of this book's evidence, the emphasis on injustice as the main cause of war seems to be empirically inadequate.

Вам также может понравиться